203 final

Ace your homework & exams now with Quizwiz!

The ____________ represents ventricular depolarization.

QRS complex

The ____________ represents ventricular contraction. It is the period of ventricular depolarization to repolarization.

QT interval

____________ is a condition in which arterioles in fingers develop spasms, causing intermittent skin pallor or cyanosis and then rubor.

Raynaud disease

____________ is narrowing of the tricuspid valve, murmur over tricuspid area.

Tricuspid stenosis

symptoms of pericardial tamponade

-fatigue -dyspnea -confusion/agitation (sudden onset)

____________ is chest pain due to ischemia of the myocardium. Can occur with activity, or at rest due to coronary artery spasms.

Angina pectoris

Left-sided HF leading to Right-sided HF

LS HF-> pulmonary congestion-> increased pressure in blood vessels of lung, pulmonary hypertension-> increased right ventricular overload-> RS HF

How do you differentiate a bacterial induced myocarditis from an acute MI?

bacterial induced myocarditis will be more global acute MI is more focal

o Angiotensin II receptor blockers may

be used in patients who are ACE inhibitor intolerant.

Hormonal effects of natriuretic peptides (counterregulatory mechanism)

1) inhibition of aldosterone and renin secretion 2) interference with ADH release

Mechanisms that regulate CO

1) preload 2) afterload 3) myocardial contractility 4) heart rate

Two subgroups of HF causes

1) primary causes 2) precipitating causes

Two classifications of HF

1) systolic 2) diastolic

____________ is the backflow of blood from the aorta into the left ventricle. Murmur with patient leaning forward, click heard in aortic area.

Aortic regurgitation

____________ is narrowing of the aortic valve, Murmur at aortic area.

Aortic stenosis

The ____________ is at the right 2nd intercostal space, you hear S2.

Aortic valve

You should compare the ____________ to the carotid pulse, and use the diaphragm and the bell when auscultating the carotid pulse.

Apical pulse

In the cardiac assessment, the ____________ should be inspected and palpated for symmetry, skin integrity, color, temp, brachial/radial pulses, fingernails, and cap refill.

Upper extremities

Which fungi is associated with myocarditis?

Candida albicans

aphasia

an abnormal neurologic condition in which language function is disordered or absent because of an injury to certain areas of the cerebral cortex

Late signs of increased intracranial pressure are? *unilateral pupil dilation, hypotension, bradycardia*

Late signs of increased intracranial pressure are? *unilateral pupil dilation, hypotension, bradycardia*

S3 and S4 are best heard when the patient is lying in the ____________ position.

Left lateral

Diagnostic studies, HF

-Endomyocardial biopsy -Echocardigraphy -chest xray -ECG, stress test -cardiac catheterization -labwork, BNP, ABG

Ventricular failure causes...

-low BP -low CO -poor renal perfusion -poor exercise tolerance -heart dysrhythmias

EKG: pericardial tamponade

-sinus tachy

What is the mnemonic for the 5 heart sounds?

All People Eat Too Much

____________ is dilation of an artery caused by weakness in the arterial wall.

Aneurysms

PA Mean

Pulmonary Artery Mean Pressure 10 - 20 mmHg

The ____________ is used as a vertical landmark, and the angle of louis is used to find the second intercostal space.

Sternum

thrombotic stroke

a stroke resulting from thrombosis or narrowing of the blood vessel

The ____________ are used to determine the "midclavicular line" for cardio assessment.

Clavicles

____________ may be heard in patients with valvular disease.

Clicks or snaps

myocyte degeneration with abundant lymphocytic infiltration

lymphocytes myocyte degeneration (****ton of lymphocytes)

Clients may complain of *projectile vomiting without nausea*

Clients may complain of *projectile vomiting without nausea*

In the cardiac assessment, the skin should be assessed for ____________, and ____________.

Color and Turgor

Elevate head of bed to? *10-30 degrees; to promote jugular venous outflow*

Elevate head of bed to? *10-30 degrees; to promote jugular venous outflow*

____________ is the backflow of blood from the left ventricle into the left atrium. Murmur at apex radiating to L axillae.

Mitral regurgitation

____________ is narrowing of the mitral valve, a murmur will be heard at the apex when the pt is left-side lying.

Mitral stenosis

The ____________ is at the left 5th intercostal space, you hear S1. It is the apex and the PMI.

Mitral valve

____________ is when the mitral valve prolapses into the left atrium, clicks heard at lower left sternal border.

Mitral valve prolapse

____________ are harsh, blowing sounds caused by disruption of blood flow.

Murmurs

____________ occurs when myocardial ischemia is sustained, resulting in tissue death. Pain lasts longer than 5 minutes, and may radiate to the left arm, or jaw.

Myocardial infarction

Myocardial infarction pain is not relieved by rest or ____________.

Nitroglycerin

When the patient is lying at a 45° angle, you see bilateral pulsations over the carotid pulse. This is a ____________ finding.

Normal

In the cardiac assessment, the eyes should be assessed for ____________, and no ____________.

Normal color, protrusion

Nursing interventions would be to? *Decrease environmental stimuli, maintain body temperature, limit fluid intake, monitor intake and output.*

Nursing interventions would be to? *Decrease environmental stimuli, maintain body temperature, limit fluid intake, monitor intake and output.*

Purpose and Types of compensatory mechanisms

Overloaded heart tries to maintain adequate CO 1) SNS activation 2) Neurohormonal response 3) dilation 4) hypertrophy

The ____________ wave represents atrial depolarization.

P

The ____________ represents the electrical current traveling across the atria, to the AV node.

PR interval

What are the symptoms of a DVT?

Pain Swelling

What are the 7 things used to describe a murmur?

1. Loudness 2. Place in cardiac cycle 3. Quality of sound 4. Pitch 5. Where it is best heard 6. Does it radiate 7. Does it change with position/respirations

Dyspnea (symptom of HF)

Common manifestation Caused by increased pulmonary pressure secondary to interstitial and alveolar edema. Mild exertion, at rest Adaptive behaviors, sleeping with more pillows

dysarthria

a disturbance in the muscular control of speech, resulting from interference in the control and execution over the muscles of speech, usually caused by damage to a central or peripheral motor nerve

Morphine

-Reduces preload and after load -Relieves dyspnea and anxiety

Compensatory mechanisms

-SNS, chatecholamines -Neurohormonal response -Ventricular dilatation -ventricualr hypertrophy

complications of acute pericarditis

-pericardial effusion -tamponade -constrictive pericarditis

common causes of acute pericarditis: non-infectious

-post-MI -uremia -neoplastic disease -CT disease (lupus) -drug induced

primary causes of heart failure that are not linked to specific genes and gene mutation

1. Hyperthyroidism and 2. rheumatic heart disease

physical findings of constrictive pericarditis

*pericardial knock -peripheral edema -ascites -hepatomegaly -JVD *kussmaul's sign

common causes of acute pericarditis: infectious

*viral=most common!! -TB -pyogenic bacteria

RAAS inhibitors

-ACE inhibitors -Angiotensin II receptor blockers -Aldosterone antagonists Monitor K+ levels for hyperkalemia

Primary causes of HF

-CAD, MI -HTN, hypertensive crisis -Rheumatic heart disease -congenital heart defects -pulmonary hypertension -cardiomyopathy (viral, postpartum, drug abuse) -hyperthyroidism -valvular disorder (mitral stenosis) -myocarditis

Dilation (compensatory mechanism)

-Enlargement of heart chambers -Pressure in the chambers, left ventricle, increases over time -Stretch heart muscle fibers initially causes increased CO -over time overstretched elastic fibers and cannot contract effectively

Ventricular failure

-HF usually both ventricles fail -one may precede the other in dysfunction -one may fail while the other pumps normally

Natriuretic peptides, counterregulatory mechanisms (2 types)

-Have renal, cardiovascular and -hormone effects -Help counter effects of SNS and RAAS -Released in response to increased blood volume in heart 1) Atrial natriuretic peptide (ANP), released from atria 2) Brain, b-type, natriuretic peptide (BNP), released from ventricles Effects: diuresis, vasodilation, lowered BP

temporary treatment of pericardial tamponade

-IV fluids -IV inotropes (2nd line)

symptoms of constrictive pericarditis

-LE edema -ab fullness -fatigue -dyspnea

EKG: acute pericarditis

-PR seg depression in ST seg elevation leads -PR elevation in ST seg depression in AVR

Contributing factors, HF

-advanced age -diabetes -tobacco use -obesity -high serum cholesterol

chest pain (symptom of HF)

-angina, chest pain -decreased coronary artery perfusion from decreased CO

Pulmonary edema, clinical manifestations

-anxious, pale, cyanotic -cool, clammy skin -dyspnea, orthopnea, tachypnea -accessory muscles -cough, blood dinges sputum -crackles, wheezes, rhonchi -tachycardia -hypo/hypertension

Etiology, HF

-anything that interferes with CO -Preload -HR -Contractility -Afterload -Direct damage to the heart -conditions that increase the workload of the heart: anemia, infection, thyroid toxicosis, dysrhythmias

symptoms of pericardial effusion

-aysmp -chest discomfort -dyspnea -fatigue

ventricular remodeling

-caused by SNS activation and neurohormonal response -hypertrophy of ventricular myocytes -produces large abnormally shaped contractile cells -increases wall tension -increased O2 consumption -impaired contractility -ventricles become less effective at pumping -can lead to dysrhythmias and death

Ventricular assist device (VAD)

-circulatory assist device, HF -bridge to transplant -can be a destination therapy, long term

Intraaortic Balloon Pump (IABP)

-circulatory assist device, HF -counter pulsation device -strict bed rest -increase coronary perfusion -decrease aortic pressure -increase forward blood flow -temporary

HF, Low BP

-decreased CO -poor renal perfusion -activates RAAS -vasoconstriction, retention of Na and H2O -elevates BP -compensatory mechanism -poor renal perfusion can lead to risk of renal failure

Behavior changes (symptom of HF)

-decreased cerebral ciruculation -restlessness, confusion, decreased attention span or memory -secondary to poor gas exchange -often in late stages

cardiac cath: constrictive pericarditis

-diastolic equalization in all 4 chambers -square root sign

Skin changes (symptom of HF)

-dusky skin, increased capillary O2 extraction -swollen shiny lower extremity, diminished or absent hair, pigment changes

symptoms of acute pericarditis

-dyspnea -pleuritic chest pain (worse on inspiration & cough) -worse laying down, better sitting up

pericardial knock

-early diastolic knock -from abrupt cessation of ventricular filling imposed by rigid pericardium

physical findings of pericardial tamponade

-exagg pulsus paradoxus *beck's triad (hypotension, JVD, muffled heart sounds)

3 functions of pericardium

-fixes heart/limits motion -prevents too much dilation during rise of SV -barrier to infection

Weight changes (symptom of HF)

-fluid retention partially due to renal failure -abdominal fullness, ascites -hepatomegaly-> anorexia and nausea -

Primary risk factors for HF

-hypertension -CAD

causes of constrictive pericarditis

-idiopathic (viral) -post-cardiac surgery -post-radiation -TB

Diastolic HF definition

-inability of the ventricles to relax and fill during diastole -high filling pressures, stiff ventricles -decreased filling of the ventricle -> decreased stroke volume -> decreased CO -results in pulmonary and systemic vascular engorgement

ventricular interdependence and pulsus paradoxus: normal

-inc RV volume during inspiration causes IV septum of shift left -pericardium allows outward expansion of RV to accommodate inc venous return -causes slight red in LV SV during inspiration=normal pulsus paradoxus

ventricular interdependence and pulsus paradoxus: cardiac tamponade

-inc in RV volume during inspiration causes EXAGGERATED shift of IV septum left -inc pericardial press inc resistance to outward expansion of ventricles -get EXAGG red in LV SV during inspiration -get EXAGG pulses paradoxus

Long term effect of SNS activation

-increase failing heart's workload and need for O2 -vasoconstriction casuses increase in preload-> increases venous return to heart-> volume overload worsens ventricular performance

cardiac cath: pericardial tamponade

-invasive -all chambers=equal diastolic pressure

Neurohormonal response (compensatory mechanism) Proinflammatory cytokines

-local but over time leads to systemic inflammatory response -released by heart cells due to cardiac inj -tumor necrosis factor and interleukin-1 -depress heart function, causes: *hypertrophy *contractile dysfunction *cell death

EKG: pericardial effusion

-low voltage! -electrical alternans (every other beat has lower voltage)

causes of pericardial effusion

-malignancy -post-cardiac surgery/procedure -comp of acute pericarditis -aortic dissection -chest trauma

ADHF clinical manifestations

-manifest as pulmonary edema -life threatening -alveoli filled with fluid -suffocation -Left HF -pink frothy sputum

Diastolic HF causes

-most common: left ventricular hypertrophy -myocardial ischemia -valve disease (aortic, mitral) -cardiomyopathy

with a large pericardial effusion you will hear...

-muffled heart sounds -ewarts sign (dullness to percussion on angle of L scapula)

Left-sided HF side effects

-prevents normal forward flow of blood -blood backs up into atrium and pulmonary veins -increased pulmonary pressure-> fluid leakage from pulmonary capillaries into interstitum and alveoli -pulmonary congestion and edema

Causes of Right-sided HF

-primary cause is left-sided heart failure -right ventricular infection (acute) -pulmonary embolism (acute) -right ventricular dilation and hypertrophy

3 determinants of symptom onset/progression in pericardial effusion

-rate of accum -volume of fluid -compliance of pericardium

acute pericarditis treatment

-rest *NSAIDS

Maintenance goals of HF treatment•

-treat underlying cause -maximize CO -alleviate SXS -improve ventricular function -improve quality of life -preserve target organ fx (heart, lung, kidney, brain) -improve mortality and morbidity, maximize viable time

treatment of pericardial effusion

-treat underlying cause -observation (asymp) -pericardiocentesis

SNS activation (compensatory mechanism)

-triggerend by low CO -least effective -release epinephrine and norepinephrine -increased HR -increased myocardial contractility and vasoconstriction

cachexia

-wasting, muscle loss, atrophy -weakness, fatigue

What is the normal range for creatine?

0.7- 1.4 mg

Onset of HF (2 types)

1) abrupt onset (i.e. from an MI) 2) subtle process from slow progressive changes

Renal effects of natriuretic peptides (counterregulatory mechanism)

1) increased GFR 2) excretion of sodium

• Important nursing responsibilities in the care of a patient with HF include

1) teaching the patient about the physiologic changes that have occurred, (2) assisting the patient to adapt to both the physiologic and psychologic changes, and (3) integrating the patient and the patient's family or support system in the overall care plan. (4) Many patients with HF are at high risk for anxiety and depression. (5) Patients should be taught to evaluate the action of the prescribed drugs and to recognize the manifestations of drug toxicity. (6) teach patients to take own pulse rate, esp. if taken digitalis and beta blockers (7) instruct on home bp monitoring (8) teach signs of hyper/hypokalemia if taking diuretics (9) have PT instruct on energy conservation (10) sodium reduction diet, medications, exercise, slowing progression of disease

Cardiovascular effects of natriuretic peptides (counterregulatory mechanism)

1) vasodilation 2) increased BP

What is the grading scale for pitting edema?

1+ 2mm 2+ 4mm 3+ 6mm 4+ 8mm

Neurohormonal response, compensatory HF (4)

1. -Kidneys release renin, RAAS-> fluid retention, vasoconstriction 2. -posterior pituitary releases ADH-> Na and H2O retention 3. -endothelin released by vascular endothelial cell-> vasoconstriction 4. -proinflammatory cytokines

etiology of heart risk factors

1. CAD 2. age. 3. HTN 4. diabetes, 5. tobacco use, 6. obesity, 7. high serum cholesterol

A client is admitted to the emergency department with chest pain and myocardial infarction is being ruled out. Vital signs: at 11:00 a.m.: P 92, RR 24, BP 140/88; 11:15 A.M.; P 96, RR 26, BP 128/82; 11:30 A.M.; P 104, RR 28, BP 104/68; 11:45 A.M.; P 118, RR 32, BP 88/58. The nurse alerts the physician, as these changes are most consistent with: 1. Cardiogenic shock 2. Cardiac Tamponade 3. Pulmonary embolism 4. Dissecting thoracic aortic aneurysm

1. Cardiogenic Shock (Cardiogenic shock occurs with severe damage (greater than 40%) to the left ventricle. Classic signs include hypotension, rapid pulse that becomes weaker, decreased urine output, and cool, clammy skin. Respiratory rate increases as the body develops metabolic acidosis from shock.)

What are 5 important questions in assessing her home health care needs?

1. Does J.F. have social and family support? 2. Does J.F. have health insurance that covers the cost of TPN and long-term use of antibiotics? 3. Does J.F. have a refrigerator to store the necessary supplies for TPN and antibiotics? 4. Can J.F. monitor her blood glucose levels? 5. How many stairs does J.F. need to climb to get in and out of her house? Bedroom? 6. Does J.F. have a clean home?

symptoms of digitalis toxicity pg 777

1. Poor appetite, 2. low heart rate 3. altered mental status, and 4. seeing halos around lights 5. Anorexia, 6. nausea, 7. vomiting, 8. blurred or yellow vision, and 9. cardiac dysrhythmias are all signs of digitalis toxicity

The client is admitted into the ED with diaphoresis, pale clammy skin, and BP of 90/70. Which intervention should the nurse implement first? 1. Start an IV with an 18-gauge catheter. 2. Administer dopamine intravenous infusion. 3. Obtain ABGs 4. Insert an indwelling urinary catheter.

1. Start an IV with an 18-gauge catheter. (There are many types of shock, but the common intervention which should be done first in all types of shock is to establish an IV line with a large-bore catheter. The low BP and cold, clammy skin indicate shock.)

The nurse and an unlicensed assistive personnel (UAP) are caring for a group of clients on a medical floor. Which action by the UAP warrants intervention by the nurse? 1. The UAP places a urine specimen in a biohazard bag in the hallway. 2. The UAP uses the alcohol foam hand cleanser after removing gloves. 3. The UAP puts soiled linen in a platic bag in the client's room. 4. The UAP obtains a disposable stethoscope for a client in an isolation room.

1. The UAP places a urine specimen in a biohazard bag in the hallway. (Specimens should be put into biohazard bags prior to leaving the client's room.)

The client with myocardial infarction is going into cardiogenic shock. Becasue of myocardial ischemia, the nurse carefully assesses the client for: 1. Ventricular dysrhythmias 2. Bradycardia 3. Rising diastolic blood pressure 4. Falling central venous pressure

1. Ventricular dysrhythmias ( Classic signs of cardiogenic shock as they relate to this question include low blood pressure and tachycardia. The CVP would rise as the backward effects of the left ventricular failure became apparent. Dysrhythmias commonly occur as a result of decreased oxygenation to the myocardium

Key complications after transplantation include

1. acute rejection, 2. risk for sudden cardiac death, 3. infection, and 4. cardiac transplant vasculopathy. 5. Life-time immunosuppression plus corticosteroids are necessary. 6. Endomyocardial biopsies are typically used to detect rejection.

precipitating causes of heart failure include

1. anemia, 2. hypothyroidism, and 3. infection

When preparing to administer the vasodilator nitroprusside (Nitropress), the nurse should

1. assess the patient's blood pressure before the administration. 2. ensure a slow rate of IV administration to prevent a sudden reduction in blood pressure. 3. The blood pressure should be assessed again every 5 to 10 minutes during administration to avoid hypotension

1. When explaining respiratory failure to the patient's family, what should the nurse use as an accurate description? a. The absence of ventilation b. Any episode in which part of the airway is obstructed c. Inadequate gas exchange to meet the metabolic needs of the body d. An episode of acute hypoxemia caused by a pulmonary dysfunction

1. c. Respiratory failure results when the transfer of oxygen or carbon dioxide function of the respiratory system is impaired and, although the definition is determined by PaO2 and PaCO2 levels, the major factor in respiratory failure is inadequate gas exchange to meet tissue oxygen (O2) needs. Absence of ventilation is respiratory arrest and partial airway obstruction may not necessarily cause respiratory failure. Acute hypoxemia may be caused by factors other than pulmonary dysfunction.

Primary causes of heart failure linked to specific genes and gene mutations include

1. coronary artery disease, 2. cardiomyopathy, and 3. hypertension

Treatment strategies for HF should include the following:

1. decreasing intravascular volume with the use of diuretics or ultrafiltration. 2. Decreasing venous return (preload) to reduce the amount of volume returned to the left ventricle (LV). 3. Decreasing afterload (the resistance against which the LV must pump) to improve CO and decrease pulmonary congestion. 4. Gas exchange is improved by the administration of intravenous (IV) morphine sulfate and supplemental oxygen. 5. Inotropic therapy and hemodynamic monitoring may be needed in patients who do not respond to conventional drug therapy. 6. reduction of anxiety is an important nursing function, since anxiety may increase the sympathetic nervous system (SNS) response and further increase myocardial workload.

The edema of chronic HF is often treated by .

1. dietary restriction of sodium. 2. Fluid restrictions are not commonly prescribed for the patient with mild to moderate HF. In moderate to severe HF and renal insufficiency, fluids are limited to less than two liters per day. 3. Patients should weigh themselves daily. If a patient experiences a weight gain of 3 pounds over 2 days or 3 to 5 pounds over a week, the primary care provider should be called

The Joint Commission has identified these three core measures for heart failure patients.

1. left ventricular function documentation, 2. prescription for ACE inhibitor at discharge, and 3. education materials

Absolute contraindications for heart transplant include

1. physiologic age over 65, 2. presence of advanced cerebral or vascular disease, and 3. severe pulmonary disease that may depend on the patient being ventilator-dependent after transplant. Indications for heart transplant include recurrent life-threatening dysrhythmias not responsive to maximal interventions and cardiac abnormalities that severely limit normal function.

• Nursing management throughout the posttransplant period focuses on

1. promoting patient adaptation to the transplant process, 2. monitoring cardiac function, 3. managing lifestyle changes, and 4. providing ongoing teaching of the patient and family.

What is the normal range for BUN?

10-20 mg

14. When caring for the patient with heart failure, which medications or treatments require careful monitoring of the patient's serum potassium level to prevent further cardiac dysfunction (select all that apply)? a. Enalapril (Vasotec) b. Furosemide (Lasix) c. Inamrinone (Inocor) d. Spironolactone (Aldactone) e. Metoprolol CR/XL (Toprol XL)

14. b, d. Furosemide is a diuretic that eliminates potassium and spironalactone is a potassium-sparing diuretic that retains potassium. The other treatments and medications are used for patients with HF but they do not directly affect serum potassium levels.

10. Which changes of aging contribute to the increased risk for respiratory failure in older adults (select all that apply)? a. Alveolar dilation b. Increased delirium e strength c. Changes in vital signs d. Increased infection risk e. Decreased respiratory muscl f. Diminished elastic recoil within the airways

10. a, d, e, f. Changes from aging that increase the older adult's risk for respiratory failure include alveolar dilation, increased risk for infection, decreased respiratory muscle strength, and diminished elastic recoil in the airways. Although delirium can complicate ventilator management, it does not increase the older patient's risk for respiratory failure. The older adult's blood pressure (BP) and heart rate (HR) increase but this does not affect the risk for respiratory failure. The ventilatory capacity is decreased and the larger air spaces decrease the surface area for gas exchange, which increases the risk.

10. Which diagnostic test is most useful in differentiating dyspnea related to pulmonary effects of heart failure from dyspnea related to pulmonary disease? a. Exercise stress testing b. Cardiac catheterization c. B-type natriuretic peptide (BNP) levels d. Determination of blood urea nitrogen (BUN)

10. c. B-type natriuretic peptide (BNP) is released from the ventricles in response to increased blood volume in the heart and is a good marker for heart failure. If BNP is elevated, shortness of breath is due to heart failure; if BNP is normal, dyspnea is due to pulmonary disease. BNP opposes the actions of the renin-angiotensin-aldosterone system, resulting in vasodilation and reduction in blood volume. Exercise stress testing and cardiac catheterization are more important tests to diagnose coronary artery disease and although the blood urea nitrogen (BUN) may be elevated in heart failure, it is a reflection of decreased renal perfusion. (See Table 32-6.)

11. Which medication is currently approved only for use with African American patients for hypertension and angina? a. Captopril (Capoten) b. Nitroglycerin (Nitro-Bid) c. Spironolactone (Aldactone) d. Isosorbide dinitrate and hydralazine (BiDil)

11. d. Isosorbide dinitrate and hydralazine (BiDil) is currently used only with African American patients for hypertension and angina. Captopril (Capoten) is used only for hypertension by all patients. Nitroglycerin (Nitro-Bid) is used with hydralazine (Apresoline) for patients who cannot tolerate renin-angiotensin-aldosterone system inhibitors for heart failure management. Spironolactone (Aldactone) is used for hypertension.

11. The nurse assesses that a patient in respiratory distress is developing respiratory fatigue and the risk of respiratory arrest when the patient displays which behavior? a. Cannot breathe unless he is sitting upright b. Uses the abdominal muscles during expiration c. Has an increased inspiratory-expiratory (I/E) ratio d. Has a change in respiratory rate from rapid to slow

11. d. The increase in respiratory rate required to blow off accumulated CO2 predisposes to respiratory muscle fatigue. The slowing of a rapid rate in a patient in acute distress indicates tiring and the possibility of respiratory arrest unless ventilatory assistance is provided. A decreased inspiratory-expiratory (I/E) ratio, orthopnea, and accessory muscle use are common findings in respiratory distress but do not necessarily signal respiratory fatigue or arrest.

12. A patient has a PaO2 of 50 mm Hg and a PaCO2 of 42 mm Hg because of an intrapulmonary shunt. Which therapy is the patient most likely to respond best to? a. Positive pressure ventilation b. Oxygen administration at a FIO2 of 100% c. Administration of O2 per nasal cannula at 1 to 3 L/min d. Clearance of airway secretions with coughing and suctioning

12. a. Patients with a shunt are usually more hypoxemic than patients with a V/Q mismatch because the alveoli are filled with fluid, which prevents gas exchange. Hypoxemia resulting from an intrapulmonary shunt is usually not responsive to high O2 concentrations and the patient will usually require positive pressure ventilation. Hypoxemia associated with a V/Q mismatch usually responds favorably to O2 administration at 1 to 3 L/min by nasal cannula. Removal of secretions with coughing and suctioning is generally not effective in reversing an acute hypoxemia resulting from a shunt.

12. Priority Decision: A patient is admitted to the emergency department with acute decompensated heart failure (ADHF). Which IV medication would the nurse expect to administer first? a. Digoxin (Lanoxin) b. Morphine sulfate c. Nesiritide (Natrecor) d. Bumetanide (Bumex)

12. c. Nesiritide (Natrecor) is a recombinant form of a natriuretic peptide that decreases preload and afterload by reducing pulmonary artery wedge pressure (PAWP) and systolic BP which decreases the workload of the heart for short-term emergency treatment of acute decompensated heart failure (ADHF). Digoxin (Lanoxin) requires a loading dose and time to work, so it is not recommended for emergency treatment of ADHF. Morphine sulfate relieves dyspnea but has more adverse events and mortality. Bumetanide (Bumex) will decrease fluid volume but also will decrease potassium levels and activate the sympathetic nervous system and renin-angiotensin-aldosterone system, which can exacerbate HF symptoms.

13. A patient with a massive hemothorax and pneumothorax has absent breath sounds in the right lung. To promote improved V/Q matching, how should the nurse position the patient? a. On the left side b. On the right side c. In a reclining chair bed d. Supine with the head of the bed elevated

13. a. When there is impaired function of one lung, the patient should be positioned with the unaffected lung in the dependent position to promote perfusion to the functioning tissue. If the diseased lung is positioned dependently, more V/Q mismatch would occur. The head of the bed may be elevated or a reclining chair may be used, with the patient positioned on the unaffected side, to maximize thoracic expansion if the patient has increased work of breathing.

13. The patient with chronic heart failure is being discharged with a diuretic, a renin-angiotensin-aldosterone system (RAAS) inhibitor, and a β-adrenergic blocker. When received from the pharmacy, which medication should not be included for this patient? a. Losartan (Cozaar) b. Carvedilol (Coreg) c. Dopamine (Intropin) d. Hydrochlorothiazide (HCTZ)

13. c. Dopamine (Intropin) is a β-adrenergic agonist that is a positive inotrope given IV, not orally, and used for acute HF. Losartan (Cozaar) is an angiotensin II receptor blocker used for patients who do not tolerate angiotensin-converting enzyme inhibitors. Carvedilol (Coreg) is the β-adrenergic blocker that blocks the sympathetic nervous system's negative effects on the failing heart. Hydrochlorothiazide (HCTZ) is the diuretic.

A patient who has been involved in a motor-vehicle crash is admitted to the ED with cool, clammy skin, tachycardia, and hypotension. All of these orders are written. Which one will the nurse act on first? a. Insert two 14-gauge IV catheters. b. Administer oxygen at 100% per non-rebreather mask. c. Place the patient on continuous cardiac monitor. d. Draw blood to type and crossmatch for transfusions.

B

14. A patient in hypercapnic respiratory failure has a nursing diagnosis of ineffective airway clearance related to increasing exhaustion. What is an appropriate nursing intervention for this patient? a. Inserting an oral airway b. Performing augmented coughing c. Teaching the patient huff coughing d. Teaching the patient slow pursed lip breathing

14. b. Augmented coughing is done by applying pressure on the abdominal muscles at the beginning of expiration. This type of coughing helps to increase abdominal pressure and expiratory flow to assist the cough to remove secretions in the patient who is exhausted. An oral airway is used only if there is a possibility that the tongue will obstruct the airway. Huff coughing prevents the glottis from closing during the cough and works well for patients with chronic obstructive pulmonary disease (COPD) to clear central airways. Slow pursed lip breathing allows more time for expiration and prevents small bronchioles from collapsing.

15. Priority Decision: A patient with chronic heart failure is treated with hydrochlorothiazide, digoxin, and lisinopril (Prinivil). To prevent the risk of digitalis toxicity with these drugs, what is most important that the nurse monitor for this patient? a. Heart rate (HR) b. Potassium levels c. Blood pressure (BP) d. Gastrointestinal function

15. b. Hypokalemia is one of the most common causes of digitalis toxicity because low serum potassium levels enhance ectopic pacemaker activity. When a patient is receiving potassium-losing diuretics, such as hydrochlorothiazide or furosemide, it is essential to monitor the patient's serum potassium levels to prevent digitalis toxicity. Monitoring the heart rate (HR) assesses for complications related to digoxin but does not prevent toxicity.

15. The patient with a history of heart failure and acute respiratory failure has thick secretions that she is having difficulty coughing up. Which intervention would best help to mobilize her secretions? a. Administer more IV fluid b. Perform postural drainage c. Provide O2 by aerosol mask d. Suction airways nasopharyngeally

15. c. For the patient with a history of heart failure, current acute respiratory failure, and thick secretions, the best intervention is to liquefy the secretions with either aerosol mask or using normal saline administered by a nebulizer. Excess IV fluid may cause cardiovascular distress and the patient probably would not tolerate postural drainage with her history. Suctioning thick secretions without thinning them is difficult and increases the patient's difficulty in maintaining oxygenation. With copious secretions, this could be done after thinning the secretions.

16. Priority Decision: After endotracheal intubation and mechanical ventilation have been started, a patient in respiratory failure becomes very agitated and is breathing asynchronously with the ventilator. What is it most important for the nurse to do first? a. Evaluate the patient's pain level, ABGs, and electrolyte values b. Sedate the patient to unconsciousness to eliminate patient awareness c. Administer the PRN vecuronium (Norcuron) to promote synchronous ventilations d. Slow the rate of ventilations provided by the ventilator to allow for spontaneous breathing by the patient

16. a. It is most important to assess the patient for the cause of the restlessness and agitation (e.g., pain, hypoxemia, electrolyte imbalances) and treat the underlying cause before sedating the patient. Although sedation, analgesia, and neuromuscular blockade are often used to control agitation and pain, these treatments may contribute to prolonged ventilator support and hospital days.

16. The health care provider prescribes spironolactone (Aldactone) for the patient with chronic heart failure. What diet modifications related to the use of this drug should the nurse include in the patient teaching? a. Decrease both sodium and potassium intake b. Increase calcium intake and decrease sodium intake c. Decrease sodium intake and increase potassium intake d. Decrease sodium intake and the use of salt substitutes for seasoning

16. a. Spironolactone is a potassium-sparing diuretic and when it is the only diuretic used in the treatment of heart failure, moderate to low levels of potassium intake should be maintained to prevent development of hyperkalemia. Sodium intake is usually reduced to at least 2400 mg/day in patients with heart failure but salt substitutes cannot be freely used because most contain high concentrations of potassium. Calcium intake is not increased.

17. The nurse monitors the patient receiving treatment for acute decompensated heart failure with the knowledge that marked hypotension is most likely to occur with the IV administration of which medication? a. Furosemide (Lasix) b. Nitroglycerin (Tridil) c. Milrinone (Primacor) d. Nitroprusside (Nipride)

17. d. Although all of these drugs may cause hypotension, nitroprusside is a potent dilator of both arteries and veins and may cause such marked hypotension that an inotropic agent (e.g., dobutamine) administration may be necessary to maintain the BP during its administration. Furosemide may cause hypotension because of diuretic-induced depletion of intravascular fluid volume. Nitroglycerin is a vasodilator and can decrease BP but not as severely as nitroprusside. It primarily dilates veins and increases myocardial oxygen supply. Milrinone has a positive inotropic effect in addition to direct arterial dilation.

17. What is the primary reason that hemodynamic monitoring is instituted in severe respiratory failure? a. To detect V/Q mismatches b. To continuously measure the arterial BP c. To evaluate oxygenation and ventilation status d. To evaluate cardiac status and blood flow to tissues

17. d. Hemodynamic monitoring with a pulmonary artery catheter is instituted in severe respiratory failure to determine the amount of blood flow to tissues and the response of the lungs and heart to hypoxemia. Continuous BP monitoring may be performed but BP is a reflection of cardiac activity, which can be determined by the pulmonary artery catheter findings. Arterial blood gases (ABGs) are important to evaluate oxygenation and ventilation status and V/Q mismatches.

18. Patients with acute respiratory failure will have drug therapy to meet their individual needs. Which drugs will meet the goal of reducing pulmonary congestion (select all that apply)? a. Morphine b. Furosemide (Lasix) c. Nitroglycerin (Tridil) d. Albuterol (Ventolin) e. Ceftriaxone (Rocephin) f. Methylprednisolone (Solu-Medrol)

18. a, b, c. Morphine and nitroglycerin (e.g., Tridil) will decrease pulmonary congestion caused by heart failure; IV diuretics (e.g., furosemide [Lasix]) are also used. Inhaled albuterol (Ventolin) or metaproterenol (Alupent) will relieve bronchospasms. Ceftriaxone (Rocephin) and azithromycin (Zithromax) are used to treat pulmonary infections. Methylprednisolone (Solu-Medrol), an IV corticosteroid, will reduce airway inflammation. Morphine is also used to decrease anxiety, agitation, and pain.

Edema (symptom of HF)

Common sign, pitting edema, weight gain more than 3 lb in 2 days -Dependent body parts (peripheral edema) -Liver (hepatomegaly) -Abdominal cavity (acites) -Lungs (Pulmonary edema and effusion)

19. In caring for a patient in acute respiratory failure, the nurse recognizes that noninvasive positive pressure ventilation (NIPPV) may be indicated for which patient? a. Is comatose and has high oxygen requirements b. Has copious secretions that require frequent suctioning c. Responds to hourly bronchodilator nebulization treatments d. Is alert and cooperative but has increasing respiratory exhaustion

19. d. Noninvasive positive pressure ventilation (NIPPV) involves the application of a face mask and delivery of a volume of air under inspiratory pressure. Because the device is worn externally, the patient must be able to cooperate in its use and frequent access to the airway for suctioning or inhaled medications must not be necessary. It is not indicated when high levels of oxygen are needed or respirations are absent.

The client is diagnosed with neurogenic shock. Which signs/symptoms should the nurse assess in this client? 1. Cool moist skin 2. Bradycardia 3. Wheezing 4. Decreased bowel sounds

2. Bradycardia (The client will have bradycardia instead of tachycardia, which is seen in other forms of shock.)

The elderly female client with vertebral fractures who has been self-medicating with ibuprofen, a NSAID, presents to the ED complaining of abdominal pain, is pale and clammy, and has a P of 110 and a BP of 92/60. Which type of shock should the nurse suspect? 1. Cardiogenic shock 2. Hypovolemic shock 3. Neurogenic shock 4. Septic shock

2. Hypovolemic shock (These client's signs/symptoms make the nurse suspect the client is losing blood, which leads to hypovolemic shock, which is the most common type of shock and is characterized by decreased intravascular volume. The client's taking of NSAID medication puts her at risk for hemorrhage because NSAIDs inhibit prostaglandin production in the stomach, which increases the risk of developing ulcers, which can erode the stomach lining and lead to hemorrhaging.)

The client diagnosed with septicemia has the following health-care provider orders. Which HCP order has the highest priority? 1. Provide clear liquid diet 2. Initiate IV antibiotic therapy 3. Obtain a STAT chest x-ray 4. Peform hourly glucometer checks

2. Initiate IV antibiotic therapy (An IV antibiotic is the priority medication for the client with an infection, which is the definition of spesis-a systemic bacterial infection of the blood. A new order for an IV antibiotic should be implemented within 1 hr of receiving the order.)

2. Which descriptions are characteristic of hypoxemic respiratory failure (select all that apply)? a. Referred to as ventilatory failure b. Primary problem is inadequate O2 transfer c. Risk of inadequate O2 saturation of hemoglobin exists d. Body is unable to compensate for acidemia of increased PaCO2 e. Most often caused by ventilation-perfusion (V/Q) mismatch and shunt f. Exists when PaO2 is 60 mm Hg or less, even when O2 is administered at 60%

2. b, c, e, f. Hypoxemic respiratory failure is often caused by ventilation-perfusion (V/Q) mismatch and shunt. It is called oxygenation failure because the primary problem is inadequate oxygen transfer. There is a risk of inadequate oxygen saturation of hemoglobin and it exists when PaO2 is 60 mm Hg or less, even when oxygen is administered at 60%. Ventilatory failure is hypercapnic respiratory failure. Hypercapnic respiratory failure results from an imbalance between ventilatory supply and ventilatory demand and the body is unable to compensate for the acidemia of increased PaCO2.

20. The patient progressed from acute lung injury to acute respiratory distress syndrome (ARDS). He is on the ventilator and receiving propofol (Diprivan) for sedation and fentanyl (Sublimaze) to decrease anxiety, agitation, and pain in order to decrease his work of breathing, O2 consumption, carbon dioxide production, and risk of injury. What intervention is recommended in caring for this patient? a. A sedation holiday b. Monitoring for hypermetabolism c. Keeping his legs still to avoid dislodging the airway d. Repositioning him every 4 hours to decrease agitation

20. a. A sedation holiday is needed to assess the patient's condition and readiness to extubate. A hypermetabolic state occurs with critical illness. With malnourished patients, enteral or parenteral nutrition is started within 24 hours; with well-nourished patients it is started within 3 days. With these medications, the patient will be assessed for cardiopulmonary depression. Venous thromboembolism prophylaxis will be used but there is no reason to keep the legs still. Repositioning the patient every 2 hours may help to decrease discomfort and agitation.

The nurse determines that treatment of heart failure has been successful when the patient experiences a. weight loss and diuresis. b. warm skin and less fatigue. c. clear lung sounds and decreased HR. d. absence of chest pain and improved level of consciousness (LOC).

20. c. Successful treatment of heart failure is indicated by an absence of symptoms of pulmonary edema and hypoxemia, such as clear lung sounds and a normal HR. Weight loss and diuresis, warm skin, less fatigue, and improved LOC may occur without resolution of pulmonary symptoms. Chest pain is not a common finding in heart failure unless coronary artery perfusion is impaired.

21. Although ARDS may result from direct lung injury or indirect lung injury as a result of systemic inflammatory response syndrome (SIRS), the nurse is aware that ARDS is most likely to occur in the patient with a host insult resulting from a. sepsis. b. oxygen toxicity. c. prolonged hypotension. d. cardiopulmonary bypass.

21. a. Although ARDS may occur in the patient who has virtually any severe illness and may be both a cause and a result of systemic inflammatory response syndrome (SIRS), the most common precipitating insults of ARDS are sepsis, gastric aspiration, and severe massive trauma.

21. Which statement by the patient with chronic heart failure should cause the nurse to determine that additional discharge teaching is needed? a. "I will call my health clinic if I wake up breathless at night." b. "I will look for sodium content on labels of foods and over-the-counter medicines." c. "I plan to organize my household tasks so I don't have to constantly go up and down the stairs." d. "I should weigh myself every morning and go on a diet if I gain more than 2 or 3 pounds in 2 days."

21. d. Further teaching is needed if the patient believes a weight gain of 2 to 3 pounds in 2 days is an indication for dieting. In a patient with heart failure, this type of weight gain reflects fluid retention and is a sign of heart failure that should be reported to the health care provider. The other options show patient understanding of the heart failure management teaching. (See Table 35-10.)

The client in shock has the following vital signs: T 99.8° F, P 132 beats/min, R 32 breaths/min, and BP 80/58 mm Hg. Calculate the pulse pressure.

22 mm Hg Pulse pressure is the difference between the systolic and diastolic pressures: 80 (systolic) - 58 (diastolic) = 22 (pulse pressure)

22. What are the primary pathophysiologic changes that occur in the injury or exudative phase of ARDS (select all that apply)? a. Atelectasis b. Shortness of breath c. Interstitial and alveolar edema d. Hyaline membranes line the alveoli e. Influx of neutrophils, monocytes, and lymphocytes

22. a, c, d. The injury or exudative phase is the early phase of ARDS when atelectasis and interstitial and alveoli edema occur and hyaline membranes composed of necrotic cells, protein, and fibrin line the alveoli. Together, these decrease gas exchange capability and lung compliance. Shortness of breath occurs but it is not a physiologic change. The increased inflammation and proliferation of fibroblasts occurs in the reparative or proliferative phase of ARDS, which occurs 1 to 2 weeks after the initial lung injury.

22. The evaluation team for cardiac transplantation is evaluating patients. Which patient is most likely to receive the most benefit from a new heart? a. A 24-year-old man with Down syndrome who has received excellent care from parents in their 60s b. A 46-year-old single woman with a limited support system who has alcohol-induced cardiomyopathy c. A 60-year-old man with inoperable coronary artery disease who has not been compliant with lifestyle changes and rehabilitation programs d. A 52-year-old woman with end-stage coronary artery disease who has limited financial resources but is emotionally stable and has strong social support

22. d. The 52-year-old woman does not have any contraindications for cardiac transplantation, even though she lacks the indication of adequate financial resources. The postoperative transplant regimen is complex and rigorous and patients who have not been compliant with other treatments or who might not have the means to understand the care would not be good candidates. A history of drug or alcohol abuse is usually a contraindication to heart transplant.

23. The nurse plans long-term goals for the patient who has had a heart transplant with the knowledge that what is the most common cause of death in heart transplant patients during the first year? a. Infection b. Heart failure c. Embolization d. Malignant conditions

23. a. Because of the need for long-term immunosuppressant therapy to prevent rejection, the patient with a transplant is at high risk for infection, a leading cause of death in transplant patients. Acute rejection episodes may also cause death in patients with transplants but many can be treated successfully with augmented immunosuppressive therapy. Malignancies occur in patients with organ transplants after taking immunosuppressants for a number of years.

23. In patients with ARDS who survive the acute phase of lung injury, what manifestations are seen when they progress to the fibrotic phase? a. Chronic pulmonary edema and atelectasis b. Resolution of edema and healing of lung tissue c. Continued hypoxemia because of diffusion limitation d. Increased lung compliance caused by the breakdown of fibrotic tissue

23. c. In the fibrotic phase of ARDS, diffuse scarring and fibrosis of the lungs occur, resulting in decreased surface area for gas exchange and continued hypoxemia caused by diffusion limitation. Although edema is resolved, lung compliance is decreased because of interstitial fibrosis. Long-term mechanical ventilation is required. The patient has a poor prognosis for survival.

24. In caring for the patient with ARDS, what is the most characteristic sign the nurse would expect the patient to exhibit? a. Refractory hypoxemia b. Bronchial breath sounds c. Progressive hypercapnia d. Increased pulmonary artery wedge pressure (PAWP)

24. a. Refractory hypoxemia, hypoxemia that does not respond to increasing concentrations of oxygenation by any route, is a hallmark of ARDS and is always present. Bronchial breath sounds may be associated with the progression of ARDS. PaCO2 levels may be normal until the patient is no longer able to compensate in response to the hypoxemia. Pulmonary artery wedge pressure (PAWP) that is normally elevated in cardiogenic pulmonary edema is normal in the pulmonary edema of ARDS.

25. The nurse suspects the early stage of ARDS in any seriously ill patient who manifests what? a. Develops respiratory acidosis b. Has diffuse crackles and rhonchi c. Exhibits dyspnea and restlessness d. Has a decreased PaO2 and an increased PaCO2

25. c. Early signs of ARDS are insidious and difficult to detect but the nurse should be alert for any early signs of hypoxemia, such as dyspnea, restlessness, tachypnea, cough, and decreased mentation, in patients at risk for ARDS. Abnormal findings on physical examination or diagnostic studies, such as adventitious lung sounds, signs of respiratory distress, respiratory alkalosis, or decreasing PaO2, are usually indications that ARDS has progressed beyond the initial stages.

26. A patient with ARDS has a nursing diagnosis of risk for infection. To detect the presence of infections commonly associated with ARDS, what should the nurse monitor? a. Gastric aspirate for pH and blood b. Quality, quantity, and consistency of sputum c. Subcutaneous emphysema of the face, neck, and chest d. Mucous membranes of the oral cavity for open lesions

26. b. Ventilator-associated pneumonia (VAP) is one of the most common complications of ARDS. Early detection requires frequent monitoring of sputum smears and cultures and assessment of the quality, quantity, and consistency of sputum. Prevention of VAP is done with strict infection control measures, ventilator bundle protocol, and subglottal secretion drainage. Blood in gastric aspirate may indicate a stress ulcer and subcutaneous emphysema of the face, neck, and chest occurs with barotrauma during mechanical ventilation. Oral infections may result from prophylactic antibiotics and impaired host defenses but are not common.

Cardiac decompensation

Compensatory mechanisms can no longer maintain adequate CO, inadequate tissue perfusion

27. The best patient response to treatment of ARDS occurs when initial management includes what? a. Treatment of the underlying condition b. Administration of prophylactic antibiotics c. Treatment with diuretics and mild fluid restriction d. Endotracheal intubation and mechanical ventilation

27. a. Because ARDS is precipitated by a physiologic insult, a critical factor in its prevention and early management is treatment of the underlying condition. Prophylactic antibiotics, treatment with diuretics and fluid restriction, and mechanical ventilation are also used as ARDS progresses.

28. When mechanical ventilation is used for the patient with ARDS, what is the rationale for applying positive endexpiratory pressure (PEEP)? a. Prevent alveolar collapse and open up collapsed alveoli b. Permit smaller tidal volumes with permissive hypercapnia c. Promote complete emptying of the lungs during exhalation d. Permit extracorporeal oxygenation and carbon dioxide removal outside the body

28. a. Positive end-expiratory pressure (PEEP) used with mechanical ventilation applies positive pressure to the airway and lungs at the end of exhalation, keeping the lung partially expanded and preventing collapse of the alveoli and helping to open up collapsed alveoli. Permissive hypercapnia is allowed when the patient with ARDS is ventilated with smaller tidal volumes to prevent barotrauma. Extracorporeal membrane oxygenation and extracorporeal CO2 removal involve passing blood across a gas-exchanging membrane outside the body and then returning oxygenated blood to the body.

29. The nurse suspects that a patient with PEEP is experiencing negative effects of this ventilatory maneuver when which of the following is assessed? a. Increasing PaO2 b. Decreasing blood pressure c. Decreasing heart rate (HR) d. Increasing central venous pressure (CVP)

29. b. PEEP increases intrathoracic and intrapulmonic pressures, compresses the pulmonary capillary bed, and reduces blood return to both the right and left sides of the heart. Increased PaO2 is an expected effect of PEEP. Preload (CVP) and cardiac output (CO) are decreased, often with a dramatic decrease in BP.

The nurse is caring for a client daignosed with septic shock. Which assessment data warrant immediate intervention by the nurse. 1. Vital signs T 100.4F, P 104, R 26, and BP 102/60. 2. A white blood cell count of 18,000/mm 3. A urinary output of 90mL in the last 4 hours. 4. The client complains of being thirsty

3. A urinary output of 90mL in the last 4 hours. (The client must have a urinary output of at least 30mL/hr, so 90mL in the last 4 hours indicates impaired renal perfusion, which is a sign of worsening shock and warrants immediate intervention.)

The client has recently experienced a MI. Which action by the nurse helps prevent cardiogenic shock? 1. Monitor the client's telemetry 2. Turn the client every 2 hours 3. Administer oxygen via nasal cannula 4. Place the client in the Tendelenburg position

3. Administer oxygen via nasal cannula (Providing adequate oxygenation of the heart muscle and decreasing the cardiac workload can prevent cardiogenic shock.)

The nurse in the ED administered an intramuscular antibiotic in the left gluteal muscle to the client with pneumonia who is being discharged home. Which intervention should the nurse implement? 1. Ask the client about drug allergies 2. Obtain a sterile sputum specimen 3. Have the client wait for 30 minutes 4. Place a warm washcloth on the client's left hip

3. Have the client wait for 30 minutes (Anytime a nurse administers a medication for the first time, the client shoulde be observed for a possible anaphylactic reaction, especially with antibiotics.)

3. When teaching the patient about what was happening when experiencing an intrapulmonary shunt, which explanation is accurate? a. This occurs when an obstruction impairs the flow of blood to the ventilated areas of the lung. b. This occurs when blood passes through an anatomic channel in the heart and bypasses the lungs. c. This occurs when blood flows through the capillaries in the lungs without participating in gas exchange. d. Gas exchange across the alveolar capillary interface is compromised by thickened or damaged alveolar membranes.

3. c. Intrapulmonary shunt occurs when blood flows through the capillaries in the lungs without participating in gas exchange (e.g., acute respiratory distress syndrome [ARDS], pneumonia). Obstruction impairs the flow of blood to the ventilated areas of the lung in a V/Q mismatch ratio greater than 1 (e.g., pulmonary embolus). Blood passes through an anatomic channel in the heart and bypasses the lungs with anatomic shunt (e.g., ventricular septal defect). Gas exchange across the alveolar capillary interface is compromised by thickened or damaged alveolar membranes in diffusion limitation (e.g., pulmonary fibrosis, ARDS).

30. Prone positioning is considered for a patient with ARDS who has not responded to other measures to increase PaO2. The nurse knows that this strategy will a. increase the mobilization of pulmonary secretions. b. decrease the workload of the diaphragm and intercostal muscles. c. promote opening of atelectatic alveoli in the upper portion of the lung. d. promote perfusion of nonatelectatic alveoli in the anterior portion of the lung.

30. d. When a patient with ARDS is supine, alveoli in the posterior areas of the lung are dependent and fluid-filled and the heart and mediastinal contents place more pressure on the lungs, predisposing to atelectasis. If the patient is turned prone, air-filled nonatelectatic alveoli in the anterior portion of the lung receive more blood and perfusion may be better matched to ventilation, causing less V/Q mismatch. Lateral rotation therapy is used to stimulate postural drainage and help mobilize pulmonary secretions.

The nurse caring for a client with sepsis writes the client diagnosis of "alteration of comfort R/T chills and fever." Which intervention should be included in the plan of care? 1. Ambulate the client in the hallway every shift 2. Monitor urinalysis, creatinine level, and BUN level 3. Apply SCDs to the lower extremities 4. Administer an antipyretic medication every 4 hours PRN

4. Administer an antipyretic medication every 4 hours PRN (Antipyretic medication will help decrease the client's fever, which directly addresses the etiology of the client's nursing diagnosis.)

Cardiac compensation

Compensatory mechanisms succeed and maintain adequate CO, perfuse tissues

Right-sided heart failure caused by disease is termed ____________.

Cor pulmonale

The client diagnosed with septicemia is receiving a broad-spectrum antibiotic. Which lab data require the nurse to notify the health-care provider? 1. The client's potassium level is 3.8 mEq/L 2. The urine culture indicates high sensitivity to the antibiotic. 3. The client's pulse oximeter reading is 94% 4. The culture and sensitivity is resistant to the client's antibiotic.

4. The culture and sensitivity is resistant to the client's antibiotic. (A sensitivity report indicating a resistance to the antibiotic being administered indicates the medication the client is receiving is not appropriate for the treatment of the infectious organism, and the HCP needs to be notified so the antibiotic can be changed.)

4. When the V/Q lung scan result returns with a mismatch ratio that is greater than 1, which condition should be suspected? a. Pain b. Atelectasis c. Pulmonary embolus d. Ventricular septal defect

4. c. There will be more ventilation than perfusion (V/Q ratio greater than 1) with a pulmonary embolus. Pain and atelectasis will cause a V/Q ratio less than 1. A ventricular septal defect causes an anatomic shunt as the blood bypasses the lungs.

4. The acronym FACES is used to help educate patients to identify symptoms of heart failure. What does this acronym mean? a. Frequent activity leads to cough in the elderly and swelling b. Factors of risk: activity, cough, emotional upsets, salt intake c. Follow activity plan, continue exercise, and know signs of problems d. Fatigue, limitation of activities, chest congestion/cough, edema, shortness of breath

4. d. FACES is used to teach patients signs and symptoms of worsening heart failure. F = Fatigue; A = Activity limitations; C = Congestion/cough; E = Edema; S = Shortness of breath. The other options are not correct.

5. Which physiologic mechanism of hypoxemia occurs with pulmonary fibrosis? a. Anatomic shunt b. Diffusion limitation c. Intrapulmonary shunt d. V/Q mismatch ratio of less than 1

5. b. Diffusion limitation in pulmonary fibrosis is caused by thickened alveolar-capillary interface, which slows gas transport.

5. What is the pathophysiologic mechanism that results in the pulmonary edema of left-sided heart failure? a. Increased right ventricular preload b. Increased pulmonary hydrostatic pressure c. Impaired alveolar oxygen and carbon dioxide exchange d. Increased lymphatic flow of pulmonary extravascular fluid

5. b. In left-sided heart failure, blood backs up into the pulmonary veins and capillaries. This increased hydrostatic pressure in the vessels causes fluid to move out of the vessels and into the pulmonary interstitial space. When increased lymphatic flow cannot remove enough fluid from the interstitial space, fluid moves into the alveoli, resulting in pulmonary edema and impaired alveolar oxygen and carbon dioxide exchange. Initially the right side of the heart is not involved.

5 yr Mortality rate, HF

50% after diagnosis It is a terminal illness

6. Which patient with the following manifestations is most likely to develop hypercapnic respiratory failure? a. Rapid, deep respirations in response to pneumonia b. Slow, shallow respirations as a result of sedative overdose c. Large airway resistance as a result of severe bronchospasm d. Poorly ventilated areas of the lung caused by pulmonary edema

6. b. Hypercapnic respiratory failure is associated with alveolar hypoventilation with increases in alveolar and arterial carbon dioxide (CO2) and often is caused by problems outside the lungs. A patient with slow, shallow respirations is not exchanging enough gas volume to eliminate CO2. Deep, rapid respirations reflect hyperventilation and often accompany lung problems that cause hypoxemic respiratory failure. Pulmonary edema and large airway resistance cause obstruction of oxygenation and result in a V/Q mismatch or shunt typical of hypoxemic respiratory failure.

7. Which arterial blood gas (ABG) results would most likely indicate acute respiratory failure in a patient with chronic lung disease? a. PaO2 52 mm Hg, PaCO2 56 mm Hg, pH 7.4 b. PaO2 46 mm Hg, PaCO2 52 mm Hg, pH 7.36 c. PaO2 48 mm Hg, PaCO2 54 mm Hg, pH 7.38 d. PaO2 50 mm Hg, PaCO2 54 mm Hg, pH 7.28

7. d. In a patient with normal lung function, respiratory failure is commonly defined as a PaO2 ≤60 mm Hg or a PaCO2 >45 mm Hg or both. However, because the patient with chronic pulmonary disease normally maintains low PaO2 and high PaCO2, acute respiratory failure in these patients can be defined as an acute decrease in PaO2 or an increase in PaCO2 from the patient's baseline parameters, accompanied by an acidic pH. The pH of 7.28 reflects an acidemia and a loss of compensation in the patient with chronic lung disease.

The nurse assesses the patient with chronic biventricular heart failure for paroxysmal nocturnal dyspnea (PND) by questioning the patient regarding a. the presence of difficulty breathing at night. b. frequent awakening to void during the night. c. the presence of a dry, hacking cough when resting. d. the use of two or more pillows to help breathing during sleep.

7. d. Paroxysmal nocturnal dyspnea (PND) is awakening from sleep with a feeling of suffocation and a need to sit up to be able to breathe and patients learn that sleeping with the upper body elevated on several pillows helps to prevent PND. Orthopnea is an inability to breathe effectively when lying down and nocturia occurs with heart failure as fluid moves back into the vascular system during recumbency, increasing renal blood flow.

8. Priority Decision: The nurse reviews the following vital signs recorded by an unlicensed assistive personnel (UAP) on a patient with acute decompensated heart failure: BP 98/60, HR 102 bpm, RR 24, Temp 98.2°F (36.7° C), SpO2 84% on 2 L/min via nasal cannula. a. Which of these findings is of highest priority? b. What should the nurse do next?

8. a. SpO2 of 84% on 2 L/min via nasal cannula indicates impaired oxygen saturation. The patient is having trouble with gas exchange. Airway and breathing are the priority (follow ABCs). b. The nurse should place the patient in high Fowler's position, assess the patient immediately, recheck SpO2, auscultate breath sounds, assess level of consciousness (LOC), check the oxygen connection and rate setting (2 L/min), and talk with the patient about her or his breathing.

8. The patient is being admitted to the intensive care unit (ICU) with hypercapnic respiratory failure. Which manifestations should the nurse expect to assess in the patient (select all that apply)? a. Cyanosis b. Metabolic acidosis c. Morning headache d. Respiratory acidosis e. Use of tripod position f. Rapid, shallow respirations

8. c, d, e, f. Morning headache, respiratory acidosis, the use of tripod position, and rapid, shallow respirations would be expected. The other manifestations are characteristic of hypoxemic respiratory failure.

CorPP

Coronary Perfusion Pressure 60 - 80 mmHg (DiasBP - PCWP)

9. Which assessment finding should cause the nurse to suspect the early onset of hypoxemia? a. Restlessness b. Hypotension c. Central cyanosis d. Cardiac dysrhythmias

9. a. Because the brain is very sensitive to a decrease in oxygen delivery, restlessness, agitation, disorientation, and confusion are early signs of hypoxemia, for which the nurse should be alert. Mild hypertension is also an early sign, accompanied by tachycardia. Central cyanosis is an unreliable, late sign of hypoxemia. Cardiac dysrhythmias also occur later.

9. A patient with chronic heart failure has atrial fibrillation and a left ventricular ejection fraction (LVEF) of 18%. To decrease the risk of complications from these conditions, what drug does the nurse anticipate giving? a. Diuretics b. Anticoagulants c. β-Adrenergic blockers d. Potassium supplements

9. b. Thrombus formation occurs in the heart when the chambers do not contract normally and empty completely. Both atrial fibrillation and very low left ventricular output (LVEF <20%) lead to thrombus formation, which is treated with anticoagulants to prevent the release of emboli into the circulation as well as antidysrhythmics or cardioversion to control atrial fibrillation.

A patient in compensated septic shock has hemodynamic monitoring with a pulmonary artery catheter and an arterial catheter. Which information obtained by the nurse indicates that the patient is still in the compensatory stage of shock? a. The cardiac output is elevated. b. The central venous pressure (CVP) is increased. c. The systemic vascular resistance (SVR) is high. d. The PAWP is high.

A

A patient outcome that is appropriate for the patient in shock who has a nursing diagnosis of decreased cardiac output related to relative hypovolemia is a. urine output of 0.5 ml/kg/hr. b. decreased peripheral edema. c. decreased CVP. d. oxygen saturation 90% or more.

A

A patient with a myocardial infarction (MI) and cardiogenic shock has the following vital signs: BP 86/50, pulse 126, respirations 30. Hemodynamic monitoring reveals an elevated PAWP and decreased cardiac output. The nurse will anticipate a. administration of furosemide (Lasix) IV. b. titration of an epinephrine (Adrenalin) drip. c. administration of a normal saline bolus. d. assisting with endotracheal intubation.

A

A therapeutic measure used to prevent hypertrophic scarring during the rehabilitative phase of burn recover is: A applying pressure garments B. repositioning the patient every 2 hours C. performing active ROM at least every 4 hours D. massaging the new tissue with water-based moisturizers

A

All of these collaborative interventions are ordered by the health care provider for a patient stung by a bee who develops severe respiratory distress and faintness. Which one will the nurse administer first? a. Epinephrine (Adrenalin) b. Normal saline infusion c. Dexamethasone (Decadron) d. Diphenhydramine (Benadryl)

A

The injury the is least likely to result in a full-thickness burn is: A. sunburn B. scald injury C. chemical burn D. electrical burn

A

The nurse evaluates that fluid resuscitation for a 70 kg patient in shock is effective on finding that the patient's a. urine output is 40 ml over the last hour. b. hemoglobin is within normal limits. c. CVP has decreased. d. mean arterial pressure (MAP) is 65 mm Hg.

A

When assessing the hemodynamic information for a newly admitted patient in shock of unknown etiology, the nurse will anticipate administration of large volumes of crystalloids when the a. cardiac output is increased and the central venous pressure (CVP) is low. b. pulmonary artery wedge pressure (PAWP) is increased, and the urine output is low. c. heart rate is decreased, and the systemic vascular resistance is low. d. cardiac output is decreased and the PAWP is high.

A

While assessing a patient in shock who has an arterial line in place, the nurse notes a drop in the systolic BP from 92 mm Hg to 76 mm Hg when the head of the patient's bed is elevated to 75 degrees. This finding indicates a need for a. additional fluid replacement. b. antibiotic administration. c. infusion of a sympathomimetic drug. d. administration of increased oxygen.

A

Catecholamines, Compensatory mechanism, HF

Epinephrine and norepinephrine -increase HR and myocardial contractility -peripheral vasoconstriction -increase in preload -increase venous return and peripheral vascular resistance on already failing heart -increases workload and O2 demand of heart

At a clinic visit, the nurse provides dietary teaching for a 56-year-old woman who was recently hospitalized with an exacerbation of chronic heart failure. The nurse determines that teaching is successful if the patient makes which statement? A "I will limit the amount of milk and cheese in my diet." B "I can add salt when cooking foods but not at the table." C "I will take an extra diuretic pill when I eat a lot of salt." D "I can have unlimited amounts of foods labeled as reduced sodium ."

A "I will limit the amount of milk and cheese in my diet." Milk products should be limited to 2 cups per day for a 2500-mg sodium-restricted diet. Salt should not be added during food preparation or at the table. Diuretics should be taken as prescribed (usually daily) and not based on sodium intake. Foods labeled as reduced sodium contain at least 25% less sodium than regular.

A70-year-old woman with chronic heart failure and atrial fibrillation asks the nurse why warfarin (Coumadin) has been prescribed for her to continue at home. Which response by the nurse is accurate? A "The medication prevents blood clots from forming in your heart." B "The medication dissolves clots that develop in your coronary arteries." C "The medication reduces clotting by decreasing serum potassium levels." D "The medication increases your heart rate so that clots do not form in your heart."

A "The medication prevents blood clots from forming in your heart." Chronic heart failure causes enlargement of the chambers of the heart and an altered electrical pathway, especially in the atria. When numerous sites in the atria fire spontaneously and rapidly, atrial fibrillation occurs. Atrial fibrillation promotes thrombus formation within the atria with an increased risk of stroke and requires treatment with cardioversion, antidysrhythmics, and/or anticoagulants. Warfarin is an anticoagulant that interferes with hepatic synthesis of vitamin K-dependent clotting factors.

The client with which laboratory result is at risk for hemorrhagic shock? A. International normalized ratio (INR) 7.9 B. Partial thromboplastin time (PTT) 12.5 seconds C. Platelets 170,000/mm3 D. Hemoglobin 8.2 g/dL

A. International normalized ratio (INR) 7.9 Prolonged INR indicates that blood takes longer than normal to clot; this client is at risk for bleeding. PTT of 12.5 seconds and a platelet value of 170,000/mm3 are both normal and pose no risk for bleeding. Although a hemoglobin of 8.2 g/dL is low, the client could have severe iron deficiency or could have received medication affecting the bone marrow.

A patient admitted with heart failure appears very anxious and complains of shortness of breath. Which nursing actions would be appropriate to alleviate this patient's anxiety (select all that apply)? A Administer ordered morphine sulfate. B Position patient in a semi-Fowler's position. C Position patient on left side with head of bed flat. D Instruct patient on the use of relaxation techniques. E Use a calm, reassuring approach while talking to patient.

A Administer ordered morphine sulfate. B Position patient in a semi-Fowler's position. D Instruct patient on the use of relaxation techniques. E Use a calm, reassuring approach while talking to patient. Morphine sulfate reduces anxiety and may assist in reducing dyspnea. The patient should be positioned in semi-Fowler's position to improve ventilation that will reduce anxiety. Relaxation techniques and a calm reassuring approach will also serve to reduce anxiety.

The patient has heart failure (HF) with an ejection fraction of less than 40%. What core measures should the nurse expect to include in the plan of care for this patient (select all that apply)? A Left ventricular function is documented. B Controlling dysrhythmias will eliminate HF. C Prescription for digoxin (Lanoxin) at discharge D Prescription for angiotensin-converting enzyme (ACE) inhibitor at discharge E Education materials about activity, medications, weight monitoring, and what to do if symptoms worsen

A Left ventricular function is documented. D Prescription for angiotensin-converting enzyme (ACE) inhibitor at discharge E Education materials about activity, medications, weight monitoring, and what to do if symptoms worsen The Joint Commission has identified these three core measures for heart failure patients. Although controlling dysrhythmias will improve CO and workload, it will not eliminate HF. Prescribing digoxin for all HF patients is no longer done because there are newer effective drugs and digoxin toxicity occurs easily related to electrolyte levels and the therapeutic range must be maintained.

A patient with a recent diagnosis of heart failure has been prescribed furosemide (Lasix) in an effort to physiologically do what for the patient? A Reduce preload. B Decrease afterload. C Increase contractility. D Promote vasodilation.

A Reduce preload. Diuretics such as furosemide are used in the treatment of HF to mobilize edematous fluid, reduce pulmonary venous pressure, and reduce preload. They do not directly influence afterload, contractility, or vessel tone.

The patient with chronic heart failure is being discharged from the hospital. What information should the nurse emphasize in the patient's discharge teaching to prevent progression of the disease to ADHF? A Take medications as prescribed. B Use oxygen when feeling short of breath. C Only ask the physician's office questions. D Encourage most activity in the morning when rested.

A Take medications as prescribed. The goal for the patient with chronic HF is to avoid exacerbations and hospitalization. Taking the medications as prescribed along with nondrug therapies such as alternating activity with rest will help the patient meet this goal. If the patient needs to use oxygen at home, it will probably be used all the time or with activity to prevent respiratory acidosis. Many HF patients are monitored by a care manager or in a transitional program to assess the patient for medication effectiveness and monitor for patient deterioration and encourage the patient. This nurse manager can be asked questions or can contact the health care provider if there is evidence of worsening HF.

A deformity or injury of the endocardium leads to what?

A clot formation on the endocardium

When assessing a patient with a partial-thickness burn, the nurse would expect to find (select all that apply): A. blisters B. exposed fascia C. exposed muscles D. intact nerve endings E. red, shiny, wet appearance

A, D, E

A 24 year old female pt has acute infective endocarditis. While obtaining a health history, the nurse should ask about which of the following: Select all that apply a. Renal dialysis b. IV drug abuse c. Recent dental work d. Cardiac cath e. Recent UTI

A,B,C,D,E

When teaching fire safety to parents at a school function, which advice does the school nurse offer about the placement of smoke and carbon monoxide detectors? A. "Every bedroom should have a separate smoke detector." B. "Every room in the house should have a smoke detector." C. "If you have a smoke detector, you don't need a carbon monoxide detector." D. "The kitchen and the bedrooms are the only rooms that need smoke detectors."

A. "Every bedroom should have a separate smoke detector." Teach all people to use home smoke detectors and carbon monoxide detectors and to ensure these are in good working order. The number of detectors needed depends on the size of the home. Recommendations are that each bedroom has a separate smoke detector, there should be at least one detector in the hallway of each story, and at least one detector is needed for the kitchen, each stairwell, and each home entrance. Carbon monoxide detectors are instrumental in picking up other types of carbon monoxide gas, such as from a defective heating unit.

A client is exhibiting signs and symptoms of early shock. What is important for the nurse to do to support the psychosocial integrity of the client? (Select all that apply.) A. Ask family members to stay with the client. B. Call the health care provider. C. Increase IV and oxygen rates. D. Remain with the client. E. Reassure the client that everything is being done for him or her.

A. Ask family members to stay with the client. D. Remain with the client. E. Reassure the client that everything is being done for him or her. Having a familiar person nearby may provide comfort to the client. The nurse should remain with the client who is demonstrating physiologic deterioration. Offering genuine reassurance supports the client who is anxious. The health care provider should be notified, and increasing IV and oxygen rates may be needed, but these actions do not support the client's psychosocial integrity.

A patient has an admitting diagnosis of acute left-sided infective endocarditis. The nurse explains to the patient that this diagnosis is best confirmed with a. blood cultures b. a complete blood count c. a cardiac catherization d. a transesophageal echocardiogram

A. Blood cultures

A client is admitted to the hospital with two of the systemic inflammatory response syndrome variables: temperature of 95° F (35° C) and high white blood cell count. Which intervention from the sepsis resuscitation bundle does the nurse initiate? A. Broad-spectrum antibiotics B. Blood transfusion C. Cooling baths D. NPO status

A. Broad-spectrum antibiotics Broad-spectrum antibiotics must be initiated within 1 hour of establishing diagnosis. A blood transfusion is indicated for low red blood cell count or low hemoglobin and hematocrit; transfusion is not part of the sepsis resuscitation bundle. Cooling baths are not indicated because the client is hypothermic, nor is this part of the sepsis resuscitation bundle. NPO status is not indicated for this client, nor is it part of the sepsis resuscitation bundle.

The client is admitted to the hospital with two of the systemic inflammatory response system (SIRS) variables: temperature of 95° F (35° C) and high white blood cell count. Which intervention from the sepsis resuscitation bundle should be initiated? A. Broad-spectrum antibiotics B. Blood transfusion C. Cooling baths D. NPO status

A. Broad-spectrum antibiotics (Broad-spectrum antibiotics must be initiated within 1 to 3 hours.)

What is the best method to prevent autocontamination for a client with burns? A. Change gloves when handling wounds on different areas of the body. B. Ensure that the client is in isolation therapy. C. Restrict visitors. D. Watch for early signs of infection.

A. Change gloves when handling wounds on different areas of the body. Gloves should be changed when wounds on different areas of the body are handled and between handling old and new dressings. Isolation therapy methods and restricting visitors are used to prevent cross-contamination, not autocontamination. Watching for early signs of infection does not prevent contamination.

The nurse is discharging a patient admitted with a transient ischemic attack (TIA). For which medications might the nurse expect to provide discharge instructions (select all that apply)? A. Clopidogrel (Plavix) B. Enoxaparin (Lovenox) C. Dipyridamole (Persantine) D. Enteric-coated aspirin (Ecotrin) E. Tissue plasminogen activator (tPA)

A. Clopidogrel (Plavix) C. Dipyridamole (Persantine) D. Enteric-coated aspirin (Ecotrin) Aspirin is the most frequently used antiplatelet agent. Other drugs to prevent clot formation include clopidogrel (Plavix), dipyridamole (Persantine), ticlopidine (Ticlid), combined dipyridamole and aspirin (Aggrenox), and anticoagulant drugs, such as oral warfarin (Coumadin). Tissue plasminogen activator is a fibrinolytic medication used to treat ischemic stroke not prevent TIAs or strokes.

A female patient who had a stroke 24 hours ago has expressive aphasia. The nurse identifies the nursing diagnosis of impaired verbal communication. An appropriate nursing intervention to help the patient communicate is to a. ask questions that the patient can answer with "yes" or "no." b. develop a list of words that the patient can read and practice reciting. c. have the patient practice her facial and tongue exercises with a mirror. d. prevent embarrassing the patient by answering for her if she does not respond.

A. Communication will be facilitated and less frustrating to the patient when questions that require a "yes" or "no" response are used. When the language areas of the brain are injured, the patient might not be able to read or recite words, which will frustrate the patient without improving communication. Expressive aphasia is caused by damage to the language areas of the brain, not by the areas that control the motor aspects of speech. The nurse should allow time for the patient to respond.

The nursing assistant is concerned about the postoperative client with blood pressure (BP) of 90/60, heart rate of 80, and respirations of 22. What should the supervising nurse do? A. Compare these vital signs with the last several readings B. Request the surgeon to see the client C. Increase the rate of intravenous fluids D. Reassess vital signs using different equipment

A. Compare these vital signs with the last several readings (Vital sign trends must be taken into consideration; BP of 90/60 may be normal for this client.)

The nursing assistant is concerned about a postoperative client with blood pressure (BP) of 90/60 mm Hg, heart rate of 80 beats/min, and respirations of 22 breaths/min. What does the supervising nurse do? A. Compare these vital signs with the last several readings. B. Request that the surgeon see the client. C. Increase the rate of intravenous fluids. D. Reassess vital signs using different equipment.

A. Compare these vital signs with the last several readings. Vital sign trends must be taken into consideration; a BP of 90/60 mm Hg may be normal for this client. Calling the surgeon is not necessary at this point, and increasing IV fluids is not indicated. The same equipment should be used when vital signs are taken postoperatively.

A patient with carotid atherosclerosis asks the nurse to describe a carotid endarterectomy. Which response by the nurse is accurate? a. "The obstructing plaque is surgically removed from an artery in the neck." b. "The diseased portion of the artery in the brain is replaced with a synthetic graft." c. "A wire is threaded through an artery in the leg to the clots in the carotid artery and the clots are removed." d. "A catheter with a deflated balloon is positioned at the narrow area, and the balloon is inflated to flatten the plaque."

A. In a carotid endarterectomy, the carotid artery is incised and the plaque is removed. The response beginning, "The diseased portion of the artery in the brain is replaced" describes an arterial graft procedure. The answer beginning, "A catheter with a deflated balloon is positioned at the narrow area" describes an angioplasty. The final response beginning, "A wire is threaded through the artery" describes the mechanical embolus removal in cerebral ischemia (MERCI) procedure.

Which factors indicate that a client's burn wounds are becoming infected? (Select all that apply.) A. Dry, crusty granulation tissue B. Elevated blood pressure C. Hypoglycemia D. Edema of the skin around the wound E. Tachycardia

A. Dry, crusty granulation tissue D. Edema of the skin around the wound E. Tachycardia Pale, boggy, dry, or crusted granulation tissue is a sign of infection, as is swelling or edema of the skin around the wound. Tachycardia is a systemic sign of infection. Hypotension, not elevated blood pressure, and hyperglycemia, not hypoglycemia, are systemic signs of infection.

A client with burn injuries states, "I feel so helpless." Which nursing intervention is most helpful for this client? A. Encouraging participation in wound care B. Encouraging visitors C. Reassuring the client that he or she will be fine D. Telling the client that these feelings are normal

A. Encouraging participation in wound care Encouraging participation in wound care will offer the client some sense of control. Encouraging visitors may be a good distraction, but will not help the client achieve a sense of control. Reassuring the client that he or she will be fine is neither helpful nor therapeutic. Telling the client that his or her feelings are normal may be reassuring, but does not address the client's issue of feeling helpless.

The client with which problem is at highest risk for hypovolemic shock? A. Esophageal varices B. Kidney failure C. Arthritis and daily acetaminophen use D. Kidney stone

A. Esophageal varices Esophageal varices are caused by portal hypertension; the portal vessels are under high pressure and are prone to rupture, causing massive upper gastrointestinal tract bleeding and hypovolemic shock. As the kidneys fail, fluid is typically retained, causing fluid volume excess, not hypovolemia. Nonsteroidal anti-inflammatory drugs such as naproxen and ibuprofen, not acetaminophen, predispose the client to gastrointestinal bleeding and hypovolemia. Although a kidney stone may cause hematuria, there is not generally massive blood loss or hypovolemia.

The nurse recognizes that the client with which problem is at highest risk for hypovolemic shock? A. Esophageal varices B. Kidney failure C. Arthritis and daily acetaminophen use D. Kidney stone

A. Esophageal varices (Esophageal varices are caused by portal hypertension; the portal vessels are under high pressure and are prone to rupture, causing massive upper GI bleeding and hypovolemic shock)

A client with partial-thickness burns of the face and chest caused by a campfire is admitted to the burn unit. The nurse plans to carry out which health care provider request first? A. Give oxygen per facemask. B. Infuse lactated Ringer's solution at 150 mL/hr. C. Give morphine sulfate 4 to 10 mg IV for pain control. D. Insert a 14 Fr retention catheter.

A. Give oxygen per facemask. Facial burns are frequently associated with upper airway inflammation. Administration of oxygen will assist in maintaining the client's tissue oxygenation at an optimal level. Although fluid hydration and pain control are important, the nurse's first priority is the client's airway. Monitoring output is important, but the nurse's first priority is the client's airway.

The nurse is reviewing the health history for an older adult client recently admitted to the burn unit with severe burns to the upper body from a house fire. The nurse plans to contact the health care provider if the client's history reveals which condition? A. Heart failure B. Diverticulitis C. Hypertension D. Emphysema

A. Heart failure A client's health history, including any pre-existing illnesses, must be known for appropriate management. Obtain specific information about the client's history of cardiac or kidney problems, chronic alcoholism, substance abuse, and diabetes mellitus. Any of these problems can influence fluid resuscitation. The stress of a burn injury can make a mild disease process worsen. In older clients, especially those with cardiac disease, a complicating factor in fluid resuscitation may be heart failure or myocardial infarction. Diverticulitis, hypertension and emphysema are important to be aware of in guiding treatment options. However, heart failure is the main concern when attempting to optimize this older client's fluid resuscitation.

Which modifiable risk factors for stroke would be most important for the nurse to include when planning a community education program? A. Hypertension B. Hyerlipidemia C. Alcohol consumption D. Oral contraceptive use

A. Hypertension Hypertension is the single most important modifiable risk factor, but it is still often undetected and inadequately treated. The public is often more aware of hyperlipidemia and oral contraceptive use as risk factors for stroke. Alcohol is also a modifiable risk factor.

When analyzing the client's assessment data, the nurse recognizes that which client is at risk for hemorrhagic shock? A. INR 7.9 B. PTT 12.5 C. Platelets 170,000 D. Hemoglobin 8.2g

A. INR 7.9 (Prolonged INR indicates that blood takes longer than normal to clot; this client is at risk for bleeding)

To position a client's burned upper extremities appropriately, how does the nurse position the client's elbow? A. In a neutral position B. In a position of comfort C. Slightly flexed D. Slightly hyperextended

A. In a neutral position The neutral (extended) position is the correct placement of the elbow to prevent contracture development. Placing the elbow in a position of comfort is not the best placement because the client then usually wants to flex the joint, which increases the risk for contracture development. The slightly flexed position increases the risk for contracture development. The slightly hyperextended position is not indicated and can be painful.

Computed tomography (CT) of a 68-year-old male patient's head reveals that he has experienced a hemorrhagic stroke. What is the priority nursing intervention in the emergency department? A. Maintenance of patient's airway. B. Positioning to promote cerebral perfusion. C. Control of fluid and electrolyte imbalances. D. Administration of tissue plasminogen activator (tPA)

A. Maintenance of patient's airway. Maintenance of a patent airway is the priority in the acute care of a patient with a hemorrhagic stroke. It supersedes the importance of fluid and electrolyte imbalance and positioning. tPA is contraindicated in hemorrhagic stroke.

The nurse reviews the medical record of a client with hemorrhagic shock, which contains the following information: Pulse 140 beats/min and thready, ABG respiratory acidosis, Blood pressure 60/40 mm Hg, Lactate level 7 mOsm/L, Respirations 40/min and shallow. All of these provider prescriptions are given for the client. Which does the nurse carry out first? A. Notify anesthesia for endotracheal intubation. B. Give Plasmanate 1 unit now. C. Give normal saline solution 250 mL/hr. D. Type and crossmatch for 4 units of packed red blood cells (PRBCs).

A. Notify anesthesia for endotracheal intubation. Establishing an airway is the priority in all emergency situations. Although administering Plasmanate and normal saline, and typing and crossmatching for 4 units of PRBCs are important actions, airway always takes priority.

A newly admitted client has deep partial-thickness burns. The nurse expects to see which clinical manifestations? A. Painful red and white wounds B. Painless, brownish yellow eschar C. Painful reddened blisters D. Painless black skin with eschar

A. Painful red and white wounds A painful red and white wound bed characterizes deep partial-thickness burns; blisters are rare. Painless, brownish yellow eschar characterizes a full-thickness burn. A painful reddened blister is seen with a superficial partial-thickness burn. Painless black skin with eschar is seen in a deep full-thickness burn.

The nurse is caring for a client with a burn injury who is receiving silver sulfadiazine (Silvadene) to the burn wounds. Which best describes the goal of topical antimicrobials? A. Reduction of bacterial growth in the wound and prevention of systemic sepsis B. Prevention of cross-contamination from other clients in the unit C. Enhanced cell growth D. Reduced need for a skin graft

A. Reduction of bacterial growth in the wound and prevention of systemic sepsis Topical antimicrobials such as silver sulfadiazine are an important intervention for infection prevention in burn wounds. Topical antimicrobials such as silver sulfadiazine do not prevent cross-contamination from other clients in the unit. They do not enhance cell growth, nor do they minimize the need the need for a skin graft.

The patient with diabetes mellitus has had a right-sided stroke. Which nursing intervention should the nurse plan to provide for this patient related to expected manifestations of this stroke? A. Safety measures B. Patience with communication C. Mobility assistance on the right side D. Place food in the left side of patient's mouth

A. Safety measures A patient with a right-sided stroke has spatial-perceptual deficits, tends to minimize problems, has a short attention span, is impulsive, and may have impaired judgment. Safety is the biggest concern for this patient. Hemiplegia occurs on the left side of this patient's body. The patient with a left-sided stroke has hemiplegia on the right, is more likely to have communication problems, and needs mobility assistance on the right side with food placed on the left side if the patient needs to be fed after a swallow evaluation has taken place.

The client recovering from an open reduction of the femur suddenly feels light-headed, with increased anxiety and agitation. Which key vital sign differentiates a pulmonary embolism from early sepsis? A. Temperature B. Pulse C. Respiration D. Blood pressure

A. Temperature (A sign of early sepsis is low-grade fever.)

A client recovering from an open reduction of the femur suddenly feels light-headed, with increased anxiety and agitation. Which key vital sign differentiates a pulmonary embolism from early sepsis? A. Temperature B. Pulse C. Respiration D. Blood pressure

A. Temperature A sign of early sepsis is low-grade fever. Both early sepsis and thrombus may cause tachycardia, tachypnea, and hypotension.

A 40-year-old patient has a ruptured cerebral aneurysm and subarachnoid hemorrhage. Which intervention will be included in the care plan? a. Apply intermittent pneumatic compression stockings. b. Assist to dangle on edge of bed and assess for dizziness. c. Encourage patient to cough and deep breathe every 4 hours. d. Insert an oropharyngeal airway to prevent airway obstruction.

A. The patient with a subarachnoid hemorrhage usually has minimal activity to prevent cerebral vasospasm or further bleeding and is at risk for venous thromboembolism (VTE). Activities such as coughing and sitting up that might increase intracranial pressure (ICP) or decrease cerebral blood flow are avoided. Because there is no indication that the patient is unconscious, an oropharyngeal airway is inappropriate.

For a patient who had a right hemisphere stroke the nurse establishes a nursing diagnosis of a. risk for injury related to denial of deficits and impulsiveness. b. impaired physical mobility related to right-sided hemiplegia. c. impaired verbal communication related to speech-language deficits. d. ineffective coping related to depression and distress about disability.

A. The patient with right-sided brain damage typically denies any deficits and has poor impulse control, leading to risk for injury when the patient attempts activities such as transferring from a bed to a chair. Right-sided brain damage causes left hemiplegia. Left-sided brain damage typically causes language deficits. Left-sided brain damage is associated with depression and distress about the disability.

Which clients are at immediate risk for hypovolemic shock? (Select all that apply.) A. Unrestrained client in motor vehicle accident B. Construction worker C. Athlete D. Surgical intensive care client E. 85-year-old with gastrointestinal virus

A. Unrestrained client in motor vehicle accident D. Surgical intensive care client E. 85-year-old with gastrointestinal virus The client who is unrestrained in a motor vehicle accident is prone to multiple trauma and bleeding. Surgical clients are at high risk for hypovolemic shock owing to fluid loss and hemorrhage. Older adult clients are prone to shock; a gastrointestinal virus results in fluid losses. Unless injured or working in excessive heat, the construction worker and the athlete are not at risk for hypovolemic shock; they may be at risk for dehydration.

Which clients are at immediate risk for hypovolemic shock? Select all that apply. A. Unrestrained in motor vehicle accident B. Construction worker C. Athlete D. Surgical intensive care client E. 85-year-old with gastrointestinal virus

A. Unrestrained in motor vehicle accident D. Surgical intensive care client E. 85-year-old with gastrointestinal virus (Unless injured or working in excessive heat, this client is not at risk for hypovolemic shock; he may be at risk for dehydration.)

Which vital sign change in a client with hypovolemic shock indicates to the nurse that the therapy is effective? A. Urine output increase from 5mL/hr to 25mL/hr B. Pulse pressure decrease from 35 mm Hg to 28 mm Hg C. Respiratory rate increase from 22 breaths/min to 28 breaths/min D. Core body temperature increase from 98.2F (36.8C) to 98.8F (37.1C)

A. Urine output increase from 5mL/hr to 25mL/hr (During shock, the kidneys and baroreceptors sense an ongoing decrease in MAP and trigger the release of renin, antidiuretic hormone (ADH), aldosterone, epinephrine, and norepinephrine to start kidney compensation, which is very sensitive to changes in fluid volume. Renin, secreted by the kidney, causes decreased urine output. ADH increases water reabsorption in the kidney, further reducing urine output. These actions compensate for shock by attempting to prevent further fluid loss. This response is so sensitive that urine output is a very good indicator of fluid resuscitation adequacy. If the therapy is not effective, urine output does not increase. Reference: p. 819, Safe and Effective Care Environment)

1. In which order will the nurse take these actions when doing a dressing change for a partial-thickness burn wound on a patient's chest? (Put a comma and a space between each answer choice [A, B, C, D, E].) a. Apply sterile gauze dressing. b. Document wound appearance. c. Apply silver sulfadiazine cream. d. Administer IV fentanyl (Sublimaze). e. Clean wound with saline-soaked gauze.

ANS: D, E, C, A, B Because partial-thickness burns are very painful, the nurse's first action should be to administer pain medications. The wound will then be cleaned, antibacterial cream applied, and covered with a new sterile dressing. The last action should be to document the appearance of the wound. DIF: Cognitive Level: Apply (application) REF: 465 OBJ: Special Questions: Prioritization TOP: Nursing Process: Implementation MSC: NCLEX: Physiological Integrity

The nurse observes a student nurse assigned to initiate oral feedings for a 68-year-old woman with an ischemic stroke. The nurse should intervene if she observes the student nurse: A. giving the patient 8 ounces of ice water to swallow. B. telling the patient to perform a chin tuck before swallowing. C. assisting the patient to sit in a chair before feeding the patient. D. assessing cranial nerves IX and X before the patient attempts to eat.

A. giving the patient 8 ounces of ice water to swallow. The majority of patients after a stroke have dysphagia. The gag reflex and swallowing ability (cranial nerves IX and X) should be assessed before the first oral feeding. To assess swallowing ability, the nurse should elevate the head of the bed to an upright position (unless contraindicated) and give the patient a very small amount (not 8 ounces) of crushed ice or ice water to swallow. The patient should remain in a high Fowler's position, preferably in a chair with the head flexed forward, for the feeding and for 30 minutes following.

After receiving change-of-shift report on the following four patients, which patient should the nurse see first? a. A 60-year-old patient with right-sided weakness who has an infusion of tPA prescribed b. A 50-year-old patient who has atrial fibrillation and a new order for warfarin (Coumadin) c. A 40-year-old patient who experienced a transient ischemic attack yesterday who has a dose of aspirin due d. A 30-year-old patient with a subarachnoid hemorrhage 2 days ago who has nimodipine (Nimotop) scheduled

A. tPA needs to be infused within the first few hours after stroke symptoms start in order to be effective in minimizing brain injury. The other medications should also be given as quickly as possible, but timing of the medications is not as critical.

A patient is admitted to the hospital with a left hemiplegia. To determine the size and location and to ascertain whether a stroke is ischemic or hemorrhagic, the nurse anticipates that the health care provider will request a a. CT scan b. lumbar puncture c. cerebral arteriogram d. positron emission tomography (PET)

A: CT scan- A CT scan is the most commonly used diagnostic test to determine the size and location of the lesion and to differentiate a thrombotic stroke from a hemorrhagic stroke. Positron emission tomography (PET) will show the metabolic activity of the brain and provide a depiction of the extent of tissue damage after a stroke. Lumbar punctures are not performed routinely because of the chance of increased intracranial pressure causing herniation. Cerebral arteriograms are invasive and may dislodge an embolism or cause further hemorrhage; they are performed only when no other test can provide the needed information.

During the acute phase of a stroke, the nurse assesses the patient's vital signs and neurologic status every 4 hours. A cardiovascular sign that the nurse would see as the body attempts to increase cerebral blood flow is a. hypertension b. fluid overload c. cardiac dysrhythmias d. S3 and S4 heart sounds

A: Hypertension- The body responds to the vasopasm and a decreased circulation to the brain that occurs with a stroke by increasing the BP, frequently resulting in hypertension. The other options are important cardiovascular factors to assess, but they do not result from impaired cerebral blood flow.

A newly admitted patient who has suffered a right sided brain stroke has a nursing diagnosis of disturbed visual sensory perception related to homonymous hemianopsia. Early in the care of the patient, the nurse should a. place objects on the right side within the patient's field of vision b. approach the patient from the left side to encourage the patient to turn the head c. place objects on the patient's left side to assess the patient's ability to compensate d. patch the affected eye to encourage the patient to turn the head to scan the environment

A: Place objects on the right side within the patient's field of vision- the presence of homonymous hemianopia in a patient with right-hemisphere brain damage causes a loss of vision in the left field. Early in the care of the patient, objects should be placed on the right side of the patient in the field of vision, and the nurse should approach the patient from the right side. Later in treatment, patients should be taught to turn the head and scan the environment and should be approached from the affected side to encourage head turning. Eye patches are used if patients have diplopia (double vision).

Four days following a stroke, a patient is to start oral fluids and feedings. Before feeding the patient, the nurse should first a. check the patient's gag reflex b. order a soft diet for the patient c. raise the head of the bed to sitting position d. evaluate the patient's ability to swallow small sips of ice water

A: check the patient's gag reflex- the first step in providing oral feedings for a patient with a stroke is ensuring that the patient has an intact gag reflex because oral feedings will not be provided if gag reflex is impaired. The nurse should then evaluate the patient's ability to swallow ice chips or ice water after placing the patient in an upright position

1. A 198-lb patient is to receive a dobutamine infusion at 5 mcg/kg/minute. The label on the infusion bag states: dobutamine 250 mg in 250 mL normal saline. When setting the infusion pump, the nurse will set the infusion rate at how many mL per hour?

ANS: 27 In order to administer the dobutamine at the prescribed rate of 5 mcg/kg/minute from a concentration of 250 mg in 250 mL, the nurse will need to infuse 27 mL/hour. DIF: Cognitive Level: Apply (application) REF: 1642 TOP: Nursing Process: Implementation MSC: NCLEX: Physiological Integrity

2. The nurse estimates the extent of a burn using the rule of nines for a patient who has been admitted with deep partial-thickness burns of the anterior trunk and the entire left arm. What percentage of the patient's total body surface area (TBSA) has been injured?

ANS: 27% When using the rule of nines, the anterior trunk is considered to cover 18% of the patient's body and each arm is 9%. DIF: Cognitive Level: Understand (comprehension) REF: 454 TOP: Nursing Process: Assessment MSC: NCLEX: Physiological Integrity

1. An 80-kg patient with burns over 30% of total body surface area (TBSA) is admitted to the burn unit. Using the Parkland formula of 4 mL/kg/%TBSA, what is the IV infusion rate (mL/hour) for lactated Ringer's solution that the nurse will administer during the first 8 hours?

ANS: 600 mL The Parkland formula states that patients should receive 4 mL/kg/%TBSA burned during the first 24 hours. Half of the total volume is given in the first 8 hours and then the last half is given over 16 hours: 4 80 30 = 9600 mL total volume; 9600/2 = 4800 mL in the first 8 hours; 4800 mL/8 hr = 600 mL/hr. DIF: Cognitive Level: Apply (application) REF: 460 TOP: Nursing Process: Implementation MSC: NCLEX: Physiological Integrity

In the cardiac assessment, the ____________, ____________, and ____________ lymph nodes should be palpated.

Epitrochlear, axillary, inguinal

1. The health care provider orders the following interventions for a 67-kg patient who has septic shock with a BP of 70/42 mm Hg and oxygen saturation of 90% on room air. In which order will the nurse implement the actions? (Put a comma and a space between each answer choice [A, B, C, D, E].) a. Obtain blood and urine cultures. b. Give vancomycin (Vancocin) 1 g IV. c. Start norepinephrine (Levophed) 0.5 mcg/min. d. Infuse normal saline 2000 mL over 30 minutes. e. Titrate oxygen administration to keep O2 saturation >95%.

ANS: E, D, C, A, B The initial action for this hypotensive and hypoxemic patient should be to improve the oxygen saturation, followed by infusion of IV fluids and vasopressors to improve perfusion. Cultures should be obtained before administration of antibiotics. DIF: Cognitive Level: Analyze (analysis) REF: 1645 OBJ: Special Questions: Prioritization TOP: Nursing Process: Implementation MSC: NCLEX: Physiological Integrity

Which assessment finding in a patient admitted with acute decompensated heart failure (ADHF) requires the most immediate action by the nurse? a. Oxygen saturation of 88% b. Weight gain of 1 kg (2.2 lb) c. Heart rate of 106 beats/minute d. Urine output of 50 mL over 2 hours

ANS: A A decrease in oxygen saturation to less than 92% indicates hypoxemia. The nurse should administer supplemental oxygen immediately to the patient. An increase in apical pulse rate, 1-kg weight gain, and decreases in urine output also indicate worsening heart failure and require nursing actions, but the low oxygen saturation rate requires the most immediate nursing action

18. A patient arrives in the emergency department with facial and chest burns caused by a house fire. Which action should the nurse take first? a. Auscultate the patient's lung sounds. b. Determine the extent and depth of the burns. c. Infuse the ordered lactated Ringer's solution. d. Administer the ordered hydromorphone (Dilaudid).

ANS: A A patient with facial and chest burns is at risk for inhalation injury, and assessment of airway and breathing is the priority. The other actions will be completed after airway management is assured. DIF: Cognitive Level: Apply (application) REF: 452 | 455 OBJ: Special Questions: Prioritization TOP: Nursing Process: Implementation MSC: NCLEX: Physiological Integrity

11. Norepinephrine (Levophed) has been prescribed for a patient who was admitted with dehydration and hypotension. Which patient data indicate that the nurse should consult with the health care provider before starting the norepinephrine? a. The patient's central venous pressure is 3 mm Hg. b. The patient is in sinus tachycardia at 120 beats/min. c. The patient is receiving low dose dopamine (Intropin). d. The patient has had no urine output since being admitted.

ANS: A Adequate fluid administration is essential before administration of vasopressors to patients with hypovolemic shock. The patient's low central venous pressure indicates a need for more volume replacement. The other patient data are not contraindications to norepinephrine administration. DIF: Cognitive Level: Apply (application) REF: 1644 TOP: Nursing Process: Implementation MSC: NCLEX: Physiological Integrity

22. Which patient is most appropriate for the burn unit charge nurse to assign to a registered nurse (RN) who has floated from the hospital medical unit? a. A 34-year-old patient who has a weight loss of 15% from admission and requires enteral feedings. b. A 67-year-old patient who has blebs under an autograft on the thigh and has an order for bleb aspiration c. A 46-year-old patient who has just come back to the unit after having a cultured epithelial autograft to the chest d. A 65-year-old patient who has twice-daily burn debridements and dressing changes to partial-thickness facial burns

ANS: A An RN from a medical unit would be familiar with malnutrition and with administration and evaluation of response to enteral feedings. The other patients require burn assessment and care that is more appropriate for staff who regularly care for burned patients. DIF: Cognitive Level: Analyze (analysis) REF: 15-16 OBJ: Special Questions: Delegation TOP: Nursing Process: Planning MSC: NCLEX: Safe and Effective Care Environment

Which information about a patient who is receiving cisatracurium (Nimbex) to prevent asynchronous breathing with the positive pressure ventilator requires immediate action by the nurse? a. Only continuous IV opioids have been ordered. b. The patient does not respond to verbal stimulation. c. There is no cough or gag when the patient is suctioned. d. The patient's oxygen saturation fluctuates between 90% to 93%.

ANS: A Because neuromuscular blockade is extremely anxiety provoking, it is essential that patients who are receiving neuromuscular blockade receive concurrent sedation and analgesia. Absence of response to stimuli is expected in patients receiving neuromuscular blockade. The oxygen saturation is adequate.

15. A patient with septic shock has a BP of 70/46 mm Hg, pulse 136, respirations 32, temperature 104° F, and blood glucose 246 mg/dL. Which intervention ordered by the health care provider should the nurse implement first? a. Give normal saline IV at 500 mL/hr. b. Give acetaminophen (Tylenol) 650 mg rectally. c. Start insulin drip to maintain blood glucose at 110 to 150 mg/dL. d. Start norepinephrine (Levophed) to keep systolic blood pressure >90 mm Hg.

ANS: A Because of the low systemic vascular resistance (SVR) associated with septic shock, fluid resuscitation is the initial therapy. The other actions also are appropriate, and should be initiated quickly as well. DIF: Cognitive Level: Apply (application) REF: 1644-1645 OBJ: Special Questions: Prioritization TOP: Nursing Process: Implementation MSC: NCLEX: Physiological Integrity

The nurse is caring for a 78-year-old patient with aortic stenosis. Which assessment data obtained by the nurse would be most important to report to the health care provider? a. The patient complains of chest pressure when ambulating. b. A loud systolic murmur is heard along the right sternal border. c. A thrill is palpated at the second intercostal space, right sternal border. d. The point of maximum impulse (PMI) is at the left midclavicular line.

ANS: A Chest pressure (or pain) occurring with aortic stenosis is caused by cardiac ischemia, and reporting this information would be a priority. A systolic murmur and thrill are expected in a patient with aortic stenosis. A PMI at the left midclavicular line is normal

The nurse is caring for a 64-year-old patient admitted with mitral valve regurgitation. Which information obtained by the nurse when assessing the patient should be communicated to the health care provider immediately? a. The patient has bilateral crackles. b. The patient has bilateral, 4+ peripheral edema. c. The patient has a loud systolic murmur across the precordium. d. The patient has a palpable thrill felt over the left anterior chest.

ANS: A Crackles that are audible throughout the lungs indicate that the patient is experiencing severe left ventricular failure with pulmonary congestion and needs immediate interventions such as diuretics. A systolic murmur and palpable thrill would be expected in a patient with mitral regurgitation. Although 4+ peripheral edema indicates a need for a change in therapy, it does not need to be addressed urgently

A patient is admitted to the hospital with possible acute pericarditis. The nurse should plan to teach the patient about the purpose of a. echocardiography. b. daily blood cultures. c. cardiac catheterization. d. 24-hour Holter monitor.

ANS: A Echocardiograms are useful in detecting the presence of the pericardial effusions associated with pericarditis. Blood cultures are not indicated unless the patient has evidence of sepsis. Cardiac catheterization and 24-hour Holter monitor is not a diagnostic procedure for pericarditis

A nurse is caring for a patient who is orally intubated and receiving mechanical ventilation. To decrease the risk for ventilator-associated pneumonia, which action will the nurse include in the plan of care? a. Elevate head of bed to 30 to 45 degrees. b. Suction the endotracheal tube every 2 to 4 hours. c. Limit the use of positive end-expiratory pressure. d. Give enteral feedings at no more than 10 mL/hr.

ANS: A Elevation of the head decreases the risk for aspiration. Positive end-expiratory pressure is frequently needed to improve oxygenation in patients receiving mechanical ventilation. Suctioning should be done only when the patient assessment indicates that it is necessary. Enteral feedings should provide adequate calories for the patient's high energy needs.

6. A patient has just been admitted with a 40% total body surface area (TBSA) burn injury. To maintain adequate nutrition, the nurse should plan to take which action? a. Insert a feeding tube and initiate enteral feedings. b. Infuse total parenteral nutrition via a central catheter. c. Encourage an oral intake of at least 5000 kcal per day. d. Administer multiple vitamins and minerals in the IV solution.

ANS: A Enteral feedings can usually be initiated during the emergent phase at low rates and increased over 24 to 48 hours to the goal rate. During the emergent phase, the patient will be unable to eat enough calories to meet nutritional needs and may have a paralytic ileus that prevents adequate nutrient absorption. Vitamins and minerals may be administered during the emergent phase, but these will not assist in meeting the patient's caloric needs. Parenteral nutrition increases the infection risk, does not help preserve gastrointestinal function, and is not routinely used in burn patients. DIF: Cognitive Level: Apply (application) REF: 463 TOP: Nursing Process: Planning MSC: NCLEX: Physiological Integrity

Which action will the public health nurse take to reduce the incidence of epidemic encephalitis in a community? a. Encourage the use of effective insect repellents during mosquito season. b. Remind patients that most cases of viral encephalitis can be cared for at home. c. Teach about the importance of prophylactic antibiotics after exposure to encephalitis. d. Arrange for screening of school-age children for West Nile virus during the school year.

ANS: A Epidemic encephalitis is usually spread by mosquitoes and ticks. Use of insect repellent is effective in reducing risk. Encephalitis frequently requires that the patient be hospitalized in an intensive care unit during the initial stages. Antibiotic prophylaxis is not used to prevent encephalitis because most encephalitis is viral. West Nile virus is most common in adults over age 50 during the summer and early fall

1. A 78-kg patient with septic shock has a urine output of 30 mL/hr for the past 3 hours. The pulse rate is 120/minute and the central venous pressure and pulmonary artery wedge pressure are low. Which order by the health care provider will the nurse question? a. Give PRN furosemide (Lasix) 40 mg IV. b. Increase normal saline infusion to 250 mL/hr. c. Administer hydrocortisone (Solu-Cortef) 100 mg IV. d. Titrate norepinephrine (Levophed) to keep systolic BP >90 mm Hg.

ANS: A Furosemide will lower the filling pressures and renal perfusion further for the patient with septic shock. The other orders are appropriate. DIF: Cognitive Level: Apply (application) REF: 1640 TOP: Nursing Process: Implementation MSC: NCLEX: Physiological Integrity

While caring for a patient who has been admitted with a pulmonary embolism, the nurse notes a change in the patient's oxygen saturation (SpO2) from 94% to 88%. Which action should the nurse take next? a. Increase the oxygen flow rate. b. Suction the patient's oropharynx. c. Instruct the patient to cough and deep breathe. d. Help the patient to sit in a more upright position.

ANS: A Increasing oxygen flow rate will usually improve oxygen saturation in patients with ventilation-perfusion mismatch, as occurs with pulmonary embolism. Because the problem is with perfusion, actions that improve ventilation, such as deep breathing and coughing, sitting upright, and suctioning, are not likely to improve oxygenation.

When admitting a patient with possible respiratory failure with a high PaCO2, which assessment information should be immediately reported to the health care provider? a. The patient is somnolent. b. The patient complains of weakness. c. The patient's blood pressure is 164/98. d. The patient's oxygen saturation is 90%.

ANS: A Increasing somnolence will decrease the patient's respiratory rate and further increase the PaCO2 and respiratory failure. Rapid action is needed to prevent respiratory arrest. An SpO2 of 90%, weakness, and elevated blood pressure all require ongoing monitoring but are not indicators of possible impending respiratory arrest.

A patient in the intensive care unit with acute decompensated heart failure (ADHF) complains of severe dyspnea and is anxious, tachypneic, and tachycardic. All of the following medications have been ordered for the patient. The nurse's priority action will be to a. give IV morphine sulfate 4 mg. b. give IV diazepam (Valium) 2.5 mg. c. increase nitroglycerin (Tridil) infusion by 5 mcg/min. d. increase dopamine (Intropin) infusion by 2 mcg/kg/min.

ANS: A Morphine improves alveolar gas exchange, improves cardiac output by reducing ventricular preload and afterload, decreases anxiety, and assists in reducing the subjective feeling of dyspnea. Diazepam may decrease patient anxiety, but it will not improve the cardiac output or gas exchange. Increasing the dopamine may improve cardiac output, but it will also increase the heart rate and myocardial oxygen consumption. Nitroglycerin will improve cardiac output and may be appropriate for this patient, but it will not directly reduce anxiety and will not act as quickly as morphine to decrease dyspnea

Which action should the nurse include in the plan of care when caring for a patient admitted with acute decompensated heart failure (ADHF) who is receiving nesiritide (Natrecor)? a. Monitor blood pressure frequently. b. Encourage patient to ambulate in room. c. Titrate nesiritide slowly before stopping. d. Teach patient about home use of the drug.

ANS: A Nesiritide is a potent arterial and venous dilator, and the major adverse effect is hypotension. Because the patient is likely to have orthostatic hypotension, the patient should not be encouraged to ambulate. Nesiritide does not require titration and is used for ADHF but not in a home setting

A nurse is caring for an obese patient with right lower lobe pneumonia. Which position will be best to improve gas exchange? a. On the left side b. On the right side c. In the tripod position d. In the high-Fowler's position

ANS: A The patient should be positioned with the "good" lung in the dependent position to improve the match between ventilation and perfusion. The obese patient's abdomen will limit respiratory excursion when sitting in the high-Fowler's or tripod positions.

____________ is at the left 3rd intercostal space, you hear S1 and S2 equally.

Erb's point

A 42-year-old patient who has bacterial meningitis is disoriented and anxious. Which nursing action will be included in the plan of care? a. Encourage family members to remain at the bedside. b. Apply soft restraints to protect the patient from injury. c. Keep the room well-lighted to improve patient orientation. d. Minimize contact with the patient to decrease sensory input.

ANS: A Patients with meningitis and disorientation will be calmed by the presence of someone familiar at the bedside. Restraints should be avoided because they increase agitation and anxiety. The patient requires frequent assessment for complications. The use of touch and a soothing voice will decrease anxiety for most patients. The patient will have photophobia, so the light should be dim

A patient who has bacterial meningitis is disoriented and anxious. Which nursing action will be included in the plan of care? a. Encourage family members to remain at the bedside. b. Apply soft restraints to protect the patient from injury. c. Keep the room well-lighted to improve patient orientation. d. Minimize contact with the patient to decrease sensory input.

ANS: A Patients with meningitis and disorientation will be calmed by the presence of someone familiar at the bedside. Restraints should be avoided because they increase agitation and anxiety. The patient requires frequent assessment for complications; the use of touch and a soothing voice will decrease anxiety for most patients. The patient will have photophobia, so the light should be dim. DIF: Cognitive Level: Application REF: 1453-1455

The nurse suspects cardiac tamponade in a patient who has acute pericarditis. To assess for the presence of pulsus paradoxus, the nurse should a. note when Korotkoff sounds are auscultated during both inspiration and expiration. b. subtract the diastolic blood pressure (DBP) from the systolic blood pressure (SBP). c. check the electrocardiogram (ECG) for variations in rate during the respiratory cycle. d. listen for a pericardial friction rub that persists when the patient is instructed to stop breathing.

ANS: A Pulsus paradoxus exists when there is a gap of greater than 10 mm Hg between when Korotkoff sounds can be heard during only expiration and when they can be heard throughout the respiratory cycle. The other methods described would not be useful in determining the presence of pulsus paradoxus

While caring for a patient with aortic stenosis, the nurse identifies a nursing diagnosis of acute pain related to decreased coronary blood flow. A priority nursing intervention for this patient would be to a. promote rest to decrease myocardial oxygen demand. b. teach the patient about the need for anticoagulant therapy. c. teach the patient to use sublingual nitroglycerin for chest pain. d. raise the head of the bed 60 degrees to decrease venous return.

ANS: A Rest is recommended to balance myocardial oxygen supply and demand and to decrease chest pain. The patient with aortic stenosis requires higher preload to maintain cardiac output, so nitroglycerin and measures to decrease venous return are contraindicated. Anticoagulation is not recommended unless the patient has atrial fibrillation

25. The charge nurse observes the following actions being taken by a new nurse on the burn unit. Which action by the new nurse would require an intervention by the charge nurse? a. The new nurse uses clean latex gloves when applying antibacterial cream to a burn wound. b. The new nurse obtains burn cultures when the patient has a temperature of 95.2° F (35.1° C). c. The new nurse administers PRN fentanyl (Sublimaze) IV to a patient 5 minutes before a dressing change. d. The new nurse calls the health care provider for a possible insulin order when a nondiabetic patient's serum glucose is elevated.

ANS: A Sterile gloves should be worn when applying medications or dressings to a burn. Hypothermia is an indicator of possible sepsis, and cultures are appropriate. Nondiabetic patients may require insulin because stress and high calorie intake may lead to temporary hyperglycemia. Fentanyl peaks 5 minutes after IV administration, and should be used just before and during dressing changes for pain management. DIF: Cognitive Level: Apply (application) REF: 461 OBJ: Special Questions: Delegation TOP: Nursing Process: Implementation MSC: NCLEX: Safe and Effective Care Environment

A nurse is caring for a patient with acute respiratory distress syndrome (ARDS) who is receiving mechanical ventilation using synchronized intermittent mandatory ventilation (SIMV). The settings include fraction of inspired oxygen (FIO2) 80%, tidal volume 450, rate 16/minute, and positive end-expiratory pressure (PEEP) 5 cm. Which assessment finding is most important for the nurse to report to the health care provider? a. Oxygen saturation 99% b. Respiratory rate 22 breaths/minute c. Crackles audible at lung bases d. Heart rate 106 beats/minute

ANS: A The FIO2 of 80% increases the risk for oxygen toxicity. Because the patient's O2 saturation is 99%, a decrease in FIO2 is indicated to avoid toxicity. The other patient data would be typical for a patient with ARDS and would not need to be urgently reported to the health care provider.

The nurse is caring for a 33-year-old patient who arrived in the emergency department with acute respiratory distress. Which assessment finding by the nurse requires the most rapid action? a. The patient's PaO2 is 45 mm Hg. b. The patient's PaCO2 is 33 mm Hg. c. The patient's respirations are shallow. d. The patient's respiratory rate is 32 breaths/minute.

ANS: A The PaO2 indicates severe hypoxemia and respiratory failure. Rapid action is needed to prevent further deterioration of the patient. Although the shallow breathing, rapid respiratory rate, and low PaCO2 also need to be addressed, the most urgent problem is the patient's poor oxygenation.

19. During change-of-shift report, the nurse is told that a patient has been admitted with dehydration and hypotension after having vomiting and diarrhea for 4 days. Which finding is most important for the nurse to report to the health care provider? a. New onset of confusion b. Heart rate 112 beats/minute c. Decreased bowel sounds d. Pale, cool, and dry extremities

ANS: A The changes in mental status are indicative that the patient is in the progressive stage of shock and that rapid intervention is needed to prevent further deterioration. The other information is consistent with compensatory shock. DIF: Cognitive Level: Apply (application) REF: 1639 OBJ: Special Questions: Prioritization TOP: Nursing Process: Assessment MSC: NCLEX: Physiological Integrity

9. A patient with circumferential burns of both legs develops a decrease in dorsalis pedis pulse strength and numbness in the toes. Which action should the nurse take? a. Notify the health care provider. b. Monitor the pulses every 2 hours. c. Elevate both legs above heart level with pillows. d. Encourage the patient to flex and extend the toes on both feet.

ANS: A The decrease in pulse in a patient with circumferential burns indicates decreased circulation to the legs and the need for an escharotomy. Monitoring the pulses is not an adequate response to the decrease in circulation. Elevating the legs or increasing toe movement will not improve the patient's circulation. DIF: Cognitive Level: Apply (application) REF: 458 TOP: Nursing Process: Implementation MSC: NCLEX: Physiological Integrity

14. Which data collected by the nurse caring for a patient who has cardiogenic shock indicate that the patient may be developing multiple organ dysfunction syndrome (MODS)? a. The patient's serum creatinine level is elevated. b. The patient complains of intermittent chest pressure. c. The patient's extremities are cool and pulses are weak. d. The patient has bilateral crackles throughout lung fields.

ANS: A The elevated serum creatinine level indicates that the patient has renal failure as well as heart failure. The crackles, chest pressure, and cool extremities are all consistent with the patient's diagnosis of cardiogenic shock. DIF: Cognitive Level: Apply (application) REF: 1649 | 1633 TOP: Nursing Process: Assessment MSC: NCLEX: Physiological Integrity

18. A patient is admitted to the emergency department (ED) for shock of unknown etiology. The first action by the nurse should be to a. administer oxygen. b. obtain a 12-lead electrocardiogram (ECG). c. obtain the blood pressure. d. check the level of consciousness.

ANS: A The initial actions of the nurse are focused on the ABCs—airway, breathing, and circulation—and administration of oxygen should be done first. The other actions should be accomplished as rapidly as possible after oxygen administration. DIF: Cognitive Level: Apply (application) REF: 1641 OBJ: Special Questions: Prioritization TOP: Nursing Process: Implementation MSC: NCLEX: Physiological Integrity

After receiving change-of-shift report on a heart failure unit, which patient should the nurse assess first? a. A patient who is cool and clammy, with new-onset confusion and restlessness b. A patient who has crackles bilaterally in the lung bases and is receiving oxygen. c. A patient who had dizziness after receiving the first dose of captopril (Capoten) d. A patient who is receiving IV nesiritide (Natrecor) and has a blood pressure of 100/62

ANS: A The patient who has "wet-cold" clinical manifestations of heart failure is perfusing inadequately and needs rapid assessment and changes in management. The other patients also should be assessed as quickly as possible but do not have indications of severe decreases in tissue perfusion.

Two days after an acute myocardial infarction (MI), a patient complains of stabbing chest pain that increases with a deep breath. Which action will the nurse take first? a. Auscultate the heart sounds. b. Check the patient's temperature. c. Notify the patient's health care provider. d. Give the PRN acetaminophen (Tylenol).

ANS: A The patient's clinical manifestations and history are consistent with pericarditis, and the first action by the nurse should be to listen for a pericardial friction rub. Checking the temperature and notifying the health care provider are also appropriate actions but would not be done before listening for a rub. It is not stated for what symptom (e.g., headache) or finding (e.g., increased temperature) the PRN acetaminophen (Tylenol) is ordered.

2. A nurse is caring for a patient with shock of unknown etiology whose hemodynamic monitoring indicates BP 92/54, pulse 64, and an elevated pulmonary artery wedge pressure. Which collaborative intervention ordered by the health care provider should the nurse question? a. Infuse normal saline at 250 mL/hr. b. Keep head of bed elevated to 30 degrees. c. Hold nitroprusside (Nipride) if systolic BP <90 mm Hg. d. Titrate dobutamine (Dobutrex) to keep systolic BP >90 mm Hg.

ANS: A The patient's elevated pulmonary artery wedge pressure indicates volume excess. A saline infusion at 250 mL/hr will exacerbate the volume excess. The other actions are appropriate for the patient. DIF: Cognitive Level: Apply (application) REF: 1633 TOP: Nursing Process: Planning MSC: NCLEX: Physiological Integrity

During a visit to a 78-year-old with chronic heart failure, the home care nurse finds that the patient has ankle edema, a 2-kg weight gain over the past 2 days, and complains of "feeling too tired to get out of bed." Based on these data, the best nursing diagnosis for the patient is a. activity intolerance related to fatigue. b. disturbed body image related to weight gain. c. impaired skin integrity related to ankle edema. d. impaired gas exchange related to dyspnea on exertion.

ANS: A The patient's statement supports the diagnosis of activity intolerance. There are no data to support the other diagnoses, although the nurse will need to assess for other patient problems

13. The nurse caring for a patient admitted with burns over 30% of the body surface assesses that urine output has dramatically increased. Which action by the nurse would best ensure adequate kidney function? a. Continue to monitor the urine output. b. Monitor for increased white blood cells (WBCs). c. Assess that blisters and edema have subsided. d. Prepare the patient for discharge from the burn unit.

ANS: A The patient's urine output indicates that the patient is entering the acute phase of the burn injury and moving on from the emergent stage. At the end of the emergent phase, capillary permeability normalizes and the patient begins to diurese large amounts of urine with a low specific gravity. Although this may occur at about 48 hours, it may be longer in some patients. Blisters and edema begin to resolve, but this process requires more time. White blood cells may increase or decrease, based on the patient's immune status and any infectious processes. The WBC count does not indicate kidney function. The patient will likely remain in the burn unit during the acute stage of burn injury. DIF: Cognitive Level: Understand (comprehension) REF: 464 TOP: Nursing Process: Application MSC: NCLEX: Physiological Integrity

A patient with acute respiratory distress syndrome (ARDS) is placed in the prone position. When prone positioning is used, which information obtained by the nurse indicates that the positioning is effective? a. The patient's PaO2 is 89 mm Hg, and the SaO2 is 91%. b. Endotracheal suctioning results in clear mucous return. c. Sputum and blood cultures show no growth after 48 hours. d. The skin on the patient's back is intact and without redness.

ANS: A The purpose of prone positioning is to improve the patient's oxygenation as indicated by the PaO2 and SaO2. The other information will be collected but does not indicate whether prone positioning has been effective.

21. Which patient should the nurse assess first? a. A patient with smoke inhalation who has wheezes and altered mental status b. A patient with full-thickness leg burns who has a dressing change scheduled c. A patient with abdominal burns who is complaining of level 8 (0 to 10 scale) pain d. A patient with 40% total body surface area (TBSA) burns who is receiving IV fluids at 500 mL/hour

ANS: A This patient has evidence of lower airway injury and hypoxemia and should be assessed immediately to determine the need for oxygen or intubation. The other patients should also be assessed as rapidly as possible, but they do not have evidence of life-threatening complications. DIF: Cognitive Level: Apply (application) REF: 452 OBJ: Special Questions: Multiple Patients TOP: Nursing Process: Assessment MSC: NCLEX: Safe and Effective Care Environment

16. An employee spills industrial acids on both arms and legs at work. What is the priority action that the occupational health nurse at the facility should take? a. Remove nonadherent clothing and watch. b. Apply an alkaline solution to the affected area. c. Place cool compresses on the area of exposure. d. Cover the affected area with dry, sterile dressings.

ANS: A With chemical burns, the initial action is to remove the chemical from contact with the skin as quickly as possible. Remove nonadherent clothing, shoes, watches, jewelry, glasses, or contact lenses (if face was exposed). Flush chemical from wound and surrounding area with copious amounts of saline solution or water. Covering the affected area or placing cool compresses on the area will leave the chemical in contact with the skin. Application of an alkaline solution is not recommended. DIF: Cognitive Level: Apply (application) REF: 455 | 457 OBJ: Special Questions: Prioritization TOP: Nursing Process: Implementation MSC: NCLEX: Physiological Integrity

2. Which preventive actions by the nurse will help limit the development of systemic inflammatory response syndrome (SIRS) in patients admitted to the hospital (select all that apply)? a. Use aseptic technique when caring for invasive lines or devices. b. Ambulate postoperative patients as soon as possible after surgery. c. Remove indwelling urinary catheters as soon as possible after surgery. d. Advocate for parenteral nutrition for patients who cannot take oral feedings. e. Administer prescribed antibiotics within 1 hour for patients with possible sepsis.

ANS: A, B, C, E Because sepsis is the most frequent etiology for SIRS, measures to avoid infection such as removing indwelling urinary catheters as soon as possible, use of aseptic technique, and early ambulation should be included in the plan of care. Adequate nutrition is important in preventing SIRS. Enteral, rather than parenteral, nutrition is preferred when patients are unable to take oral feedings because enteral nutrition helps maintain the integrity of the intestine, thus decreasing infection risk. Antibiotics should be administered within 1 hour after being prescribed to decrease the risk of sepsis progressing to SIRS. DIF: Cognitive Level: Analyze (analysis) REF: 1649 TOP: Nursing Process: Planning MSC: NCLEX: Physiological Integrity

1. A patient with suspected neurogenic shock after a diving accident has arrived in the emergency department. A cervical collar is in place. Which actions should the nurse take (select all that apply)? a. Prepare to administer atropine IV. b. Obtain baseline body temperature. c. Infuse large volumes of lactated Ringer's solution. d. Provide high-flow oxygen (100%) by non-rebreather mask. e. Prepare for emergent intubation and mechanical ventilation.

ANS: A, B, D, E All of the actions are appropriate except to give large volumes of lactated Ringer's solution. The patient with neurogenic shock usually has a normal blood volume, and it is important not to volume overload the patient. In addition, lactated Ringer's solution is used cautiously in all shock situations because the failing liver cannot convert lactate to bicarbonate. DIF: Cognitive Level: Apply (application) REF: 1646 TOP: Nursing Process: Implementation MSC: NCLEX: Physiological Integrity

Based on the Joint Commission Core Measures for patients with heart failure, which topics should the nurse include in the discharge teaching plan for a patient who has been hospitalized with chronic heart failure (select all that apply)? a. How to take and record daily weight b. Importance of limiting aerobic exercise c. Date and time of follow-up appointment d. Symptoms indicating worsening heart failure e. Actions and side effects of prescribed medications

ANS: A, C, D, E The Joint Commission Core Measures state that patients should be taught about prescribed medications, follow-up appointments, weight monitoring, and actions to take for worsening symptoms. Patients with heart failure are encouraged to begin or continue aerobic exercises such as walking, while self-monitoring to avoid excessive fatigue

24. After change-of-shift report in the progressive care unit, who should the nurse care for first? a. Patient who had an inferior myocardial infarction 2 days ago and has crackles in the lung bases b. Patient with suspected urosepsis who has new orders for urine and blood cultures and antibiotics c. Patient who had a T5 spinal cord injury 1 week ago and currently has a heart rate of 54 beats/minute d. Patient admitted with anaphylaxis 3 hours ago who now has clear lung sounds and a blood pressure of 108/58 mm Hg

ANS: B Antibiotics should be administered within the first hour for patients who have sepsis or suspected sepsis in order to prevent progression to systemic inflammatory response syndrome (SIRS) and septic shock. The data on the other patients indicate that they are more stable. Crackles heard only at the lung bases do not require immediate intervention in a patient who has had a myocardial infarction. Mild bradycardia does not usually require atropine in patients who have a spinal cord injury. The findings for the patient admitted with anaphylaxis indicate resolution of bronchospasm and hypotension. DIF: Cognitive Level: Analyze (analysis) REF: 1644 | 1646 OBJ: Special Questions: Prioritization; Multiple Patients TOP: Nursing Process: Assessment MSC: NCLEX: Safe and Effective Care Environment

Which of these findings is the best indicator that the fluid resuscitation for a patient with hypovolemic shock has been successful? a. Hemoglobin is within normal limits. b. Urine output is 60 mL over the last hour. c. Pulmonary artery wedge pressure (PAWP) is normal. d. Mean arterial pressure (MAP) is 65 mm Hg.

ANS: B Assessment of end organ perfusion, such as an adequate urine output, is the best indicator that fluid resuscitation has been successful. The hemoglobin level, PAWP, and MAP are useful in determining the effects of fluid administration, but they are not as useful as data indicating good organ perfusion. DIF: Cognitive Level: Application REF: 1733-1735 TOP: Nursing Process: Evaluation MSC: NCLEX: Physiological Integrity

9. Which finding is the best indicator that the fluid resuscitation for a patient with hypovolemic shock has been effective? a. Hemoglobin is within normal limits. b. Urine output is 60 mL over the last hour. c. Central venous pressure (CVP) is normal. d. Mean arterial pressure (MAP) is 72 mm Hg.

ANS: B Assessment of end organ perfusion, such as an adequate urine output, is the best indicator that fluid resuscitation has been successful. The hemoglobin level, CVP, and MAP are useful in determining the effects of fluid administration, but they are not as useful as data indicating good organ perfusion. DIF: Cognitive Level: Apply (application) REF: 1642 TOP: Nursing Process: Evaluation MSC: NCLEX: Physiological Integrity

17. The nurse is caring for a patient who has septic shock. Which assessment finding is most important for the nurse to report to the health care provider? a. Blood pressure (BP) 92/56 mm Hg b. Skin cool and clammy c. Oxygen saturation 92% d. Heart rate 118 beats/minute

ANS: B Because patients in the early stage of septic shock have warm and dry skin, the patient's cool and clammy skin indicates that shock is progressing. The other information will also be reported, but does not indicate deterioration of the patient's status. DIF: Cognitive Level: Apply (application) REF: 1638 OBJ: Special Questions: Prioritization TOP: Nursing Process: Assessment MSC: NCLEX: Physiological Integrity

A patient admitted with acute respiratory failure has a nursing diagnosis of ineffective airway clearance related to thick, secretions. Which action is a priority for the nurse to include in the plan of care? a. Encourage use of the incentive spirometer. b. Offer the patient fluids at frequent intervals. c. Teach the patient the importance of ambulation. d. Titrate oxygen level to keep O2 saturation >93%.

ANS: B Because the reason for the poor airway clearance is the thick secretions, the best action will be to encourage the patient to improve oral fluid intake. Patients should be instructed to use the incentive spirometer on a regular basis (e.g., every hour) in order to facilitate the clearance of the secretions. The other actions may also be helpful in improving the patient's gas exchange, but they do not address the thick secretions that are causing the poor airway clearance.

A patient who has just been admitted with septic shock has a BP of 70/46, pulse 136, respirations 32, temperature 104.0° F, and blood glucose 246 mg/dl. Which order will the nurse accomplish first? a. Start insulin drip to maintain blood glucose at 110 to 150 mg/dl. b. Give normal saline IV at 500 ml/hr. c. Titrate norepinephrine (Levophed) to keep MAP at 65 to 70 mm Hg. d. Infuse drotrecogin- (Xigris) 24 mcg/kg.

B

Which statement by the nurse when explaining the purpose of positive end-expiratory pressure (PEEP) to the family members of a patient with ARDS is accurate? a. "PEEP will push more air into the lungs during inhalation." b. "PEEP prevents the lung air sacs from collapsing during exhalation." c. "PEEP will prevent lung damage while the patient is on the ventilator." d. "PEEP allows the breathing machine to deliver 100% oxygen to the lungs."

ANS: B By preventing alveolar collapse during expiration, PEEP improves gas exchange and oxygenation. PEEP will not prevent lung damage (e.g., fibrotic changes that occur with ARDS), push more air into the lungs, or change the fraction of inspired oxygen (FIO2) delivered to the patient.

26. Which nursing action is a priority for a patient who has suffered a burn injury while working on an electrical power line? a. Obtain the blood pressure. b. Stabilize the cervical spine. c. Assess for the contact points. d. Check alertness and orientation.

ANS: B Cervical spine injuries are commonly associated with electrical burns. Therefore stabilization of the cervical spine takes precedence after airway management. The other actions are also included in the emergent care after electrical burns, but the most important action is to avoid spinal cord injury. DIF: Cognitive Level: Apply (application) REF: 452 | 456 OBJ: Special Questions: Prioritization TOP: Nursing Process: Implementation MSC: NCLEX: Physiological Integrity

The nurse identifies the nursing diagnosis of decreased cardiac output related to valvular insufficiency for the patient with infective endocarditis (IE) based on which assessment finding(s)? a. Fever, chills, and diaphoresis b. Urine output less than 30 mL/hr c. Petechiae on the inside of the mouth and conjunctiva d. Increase in heart rate of 15 beats/minute with walking

ANS: B Decreased renal perfusion caused by inadequate cardiac output will lead to decreased urine output. Petechiae, fever, chills, and diaphoresis are symptoms of IE, but are not caused by decreased cardiac output. An increase in pulse rate of 15 beats/minute is normal with exercise

To maintain a positive nitrogen balance in a major burn, the patient must: A. increase normal caloric intake by about 3 times B. eat a high-protein, low-fat, high-carbohydrate diet C. eat at least 1500 calories per day in small, frequent meals D. eat rice and whole wheat for the chemical effect on nitrogen balance

B

22. The following interventions are ordered by the health care provider for a patient who has respiratory distress and syncope after eating strawberries. Which will the nurse complete first? a. Start a normal saline infusion. b. Give epinephrine (Adrenalin). c. Start continuous ECG monitoring. d. Give diphenhydramine (Benadryl).

ANS: B Epinephrine rapidly causes peripheral vasoconstriction, dilates the bronchi, and blocks the effects of histamine and reverses the vasodilation, bronchoconstriction, and histamine release that cause the symptoms of anaphylaxis. The other interventions are also appropriate but would not be the first ones completed. DIF: Cognitive Level: Apply (application) REF: 1645 OBJ: Special Questions: Prioritization TOP: Nursing Process: Implementation MSC: NCLEX: Physiological Integrity

A patient with acute respiratory distress syndrome (ARDS) and acute kidney injury has the following medications ordered. Which medication should the nurse discuss with the health care provider before giving? a. Pantoprazole (Protonix) 40 mg IV b. Gentamicin (Garamycin) 60 mg IV c. Sucralfate (Carafate) 1 g per nasogastric tube d. Methylprednisolone (Solu-Medrol) 60 mg IV

ANS: B Gentamicin, which is one of the aminoglycoside antibiotics, is potentially nephrotoxic, and the nurse should clarify the drug and dosage with the health care provider before administration. The other medications are appropriate for the patient with ARDS.

After receiving report on the following patients, which patient should the nurse assess first? a. Patient with rheumatic fever who has sharp chest pain with a deep breath b. Patient with acute aortic regurgitation whose blood pressure is 86/54 mm Hg c. Patient with infective endocarditis who has a murmur and splinter hemorrhages d. Patient with dilated cardiomyopathy who has bilateral crackles at the lung bases

ANS: B Hypotension in patients with acute aortic regurgitation may indicate cardiogenic shock. The nurse should immediately assess this patient for other findings such as dyspnea or chest pain. The findings in the other patients are typical of their diagnoses and do not indicate a need for urgent assessment and intervention

To monitor a patient with severe acute pancreatitis for the early organ damage associated with MODS, the most important assessments for the nurse to make are a. stool guaiac and bowel sounds. b. lung sounds and oxygenation status. c. serum creatinine and urinary output. d. serum bilirubin levels and skin color.

B

A 53-year-old patient with Stage D heart failure and type 2 diabetes asks the nurse whether heart transplant is a possible therapy. Which response by the nurse is most appropriate? a. "Because you have diabetes, you would not be a candidate for a heart transplant." b. "The choice of a patient for a heart transplant depends on many different factors." c. "Your heart failure has not reached the stage in which heart transplants are needed." d. "People who have heart transplants are at risk for multiple complications after surgery."

ANS: B Indications for a heart transplant include end-stage heart failure (Stage D), but other factors such as coping skills, family support, and patient motivation to follow the rigorous posttransplant regimen are also considered. Diabetic patients who have well-controlled blood glucose levels may be candidates for heart transplant. Although heart transplants can be associated with many complications, this response does not address the patient's question

The nurse is assessing a patient with myocarditis before administering the scheduled dose of digoxin (Lanoxin). Which finding is most important for the nurse to communicate to the health care provider? a. Leukocytosis b. Irregular pulse c. Generalized myalgia d. Complaint of fatigue

ANS: B Myocarditis predisposes the heart to digoxin-associated dysrhythmias and toxicity. The other findings are common symptoms of myocarditis and there is no urgent need to report these.

During the assessment of a 25-year-old patient with infective endocarditis (IE), the nurse would expect to find a. substernal chest pressure. b. a new regurgitant murmur. c. a pruritic rash on the chest. d. involuntary muscle movement.

ANS: B New regurgitant murmurs occur in IE because vegetations on the valves prevent valve closure. Substernal chest discomfort, rashes, and involuntary muscle movement are clinical manifestations of other cardiac disorders such as angina and rheumatic fever

To assess the patient with pericarditis for evidence of a pericardial friction rub, the nurse should a. listen for a rumbling, low-pitched, systolic murmur over the left anterior chest. b. auscultate by placing the diaphragm of the stethoscope on the lower left sternal border. c. ask the patient to cough during auscultation to distinguish the sound from a pleural friction rub. d. feel the precordial area with the palm of the hand to detect vibrations with cardiac contraction.

ANS: B Pericardial friction rubs are heard best with the diaphragm at the lower left sternal border. The nurse should ask the patient to hold his or her breath during auscultation to distinguish the sounds from a pleural friction rub. Friction rubs are not typically low pitched or rumbling and are not confined to systole. Rubs are not assessed by palpation

The nurse is admitting a patient with possible rheumatic fever. Which question on the admission health history will be most pertinent to ask? a. "Do you use any illegal IV drugs?" b. "Have you had a recent sore throat?" c. "Have you injured your chest in the last few weeks?" d. "Do you have a family history of congenital heart disease?"

ANS: B Rheumatic fever occurs as a result of an abnormal immune response to a streptococcal infection. Although illicit IV drug use should be discussed with the patient before discharge, it is not a risk factor for rheumatic fever, and would not be as pertinent when admitting the patient. Family history is not a risk factor for rheumatic fever. Chest injury would cause musculoskeletal chest pain rather than rheumatic fever.

10. Which intervention will the nurse include in the plan of care for a patient who has cardiogenic shock? a. Check temperature every 2 hours. b. Monitor breath sounds frequently. c. Maintain patient in supine position. d. Assess skin for flushing and itching.

ANS: B Since pulmonary congestion and dyspnea are characteristics of cardiogenic shock, the nurse should assess the breath sounds frequently. The head of the bed is usually elevated to decrease dyspnea in patients with cardiogenic shock. Elevated temperature and flushing or itching of the skin are not typical of cardiogenic shock. DIF: Cognitive Level: Apply (application) REF: 1633 TOP: Nursing Process: Implementation MSC: NCLEX: Physiological Integrity

Which assessment finding in a patient who is admitted with infective endocarditis (IE) is most important to communicate to the health care provider? a. Generalized muscle aching b. Sudden onset right flank pain c. Janeway's lesions on the palms d. Temperature 100.7° F (38.1° C)

ANS: B Sudden onset of flank pain indicates possible embolization to the kidney and may require diagnostic testing such as a renal arteriogram and interventions to improve renal perfusion. The other findings are typically found in IE, but do not require any new interventions

7. A patient with cardiogenic shock has the following vital signs: BP 102/50, pulse 128, respirations 28. The pulmonary artery wedge pressure (PAWP) is increased and cardiac output is low. The nurse will anticipate an order for which medication? a. 5% human albumin b. Furosemide (Lasix) IV c. Epinephrine (Adrenalin) drip d. Hydrocortisone (Solu-Cortef)

ANS: B The PAWP indicates that the patient's preload is elevated, and furosemide is indicated to reduce the preload and improve cardiac output. Epinephrine would further increase heart rate and myocardial oxygen demand. 5% human albumin would also increase the PAWP. Hydrocortisone might be considered for septic or anaphylactic shock. DIF: Cognitive Level: Apply (application) REF: 1645 TOP: Nursing Process: Planning MSC: NCLEX: Physiological Integrity

When developing a community health program to decrease the incidence of rheumatic fever, which action would be most important for the community health nurse to include? a. Vaccinate high-risk groups in the community with streptococcal vaccine. b. Teach community members to seek treatment for streptococcal pharyngitis. c. Teach about the importance of monitoring temperature when sore throats occur. d. Teach about prophylactic antibiotics to those with a family history of rheumatic fever.

ANS: B The incidence of rheumatic fever is decreased by treatment of streptococcal infections with antibiotics. Family history is not a risk factor for rheumatic fever. There is no immunization that is effective in decreasing the incidence of rheumatic fever. Teaching about monitoring temperature will not decrease the incidence of rheumatic fever.

The nurse is caring for a patient who is intubated and receiving positive pressure ventilation to treat acute respiratory distress syndrome (ARDS). Which finding is most important to report to the health care provider? a. Blood urea nitrogen (BUN) level 32 mg/dL b. Red-brown drainage from orogastric tube c. Scattered coarse crackles heard throughout lungs d. Arterial blood gases: pH 7.31, PaCO2 50, PaO2 68

ANS: B The nasogastric drainage indicates possible gastrointestinal bleeding and/or stress ulcer, and should be reported. The pH and PaCO2 are slightly abnormal, but current guidelines advocating for permissive hypercapnia indicate that these would not indicate an immediate need for a change in therapy. The BUN is slightly elevated but does not indicate an immediate need for action. Adventitious breath sounds are commonly heard in patients with ARDS.

While caring for a seriously ill patient, the nurse determines that the patient may be in the compensatory stage of shock on finding a. cold, mottled extremities. b. restlessness and apprehension. c. a heart rate of 120 and cool, clammy skin. d. systolic BP less than 90 mm Hg.

B

After receiving the following information about four patients during change-of-shift report, which patient should the nurse assess first? a. Patient with acute pericarditis who has a pericardial friction rub b. Patient who has just returned to the unit after balloon valvuloplasty c. Patient who has hypertrophic cardiomyopathy and a heart rate of 116 d. Patient with a mitral valve replacement who has an anticoagulant scheduled

ANS: B The patient who has just arrived after balloon valvuloplasty will need assessment for complications such as bleeding and hypotension. The information about the other patients is consistent with their diagnoses and does not indicate any complications or need for urgent assessment or intervention. DIF

A patient with possible viral meningitis is admitted to the nursing unit after lumbar puncture was performed in the emergency department. Which action prescribed by the health care provider should the nurse question? a. Elevate the head of the bed 20 degrees. b. Restrict oral fluids to 1000 mL daily. c. Administer ceftriaxone (Rocephin) 1 g IV every 12 hours. d. Give ibuprofen (Motrin) 400 mg every 6 hours as needed for headache.

ANS: B The patient with meningitis has increased fluid needs, so oral fluids should be encouraged. The other actions are appropriate. Slight elevation of the head of the bed will decrease headache without causing leakage of cerebrospinal fluid from the lumbar puncture site. Antibiotics should be administered until bacterial meningitis is ruled out by the cerebrospinal fluid analysis

4. The oxygen saturation (SpO2) for a patient with left lower lobe pneumonia is 90%. The patient has rhonchi, a weak cough effort, and complains of fatigue. Which action is a priority for the nurse to take? a. Position the patient on the left side. b. Assist the patient with staged coughing. c. Place a humidifier in the patient's room. d. Schedule a 2-hour rest period for the patient.

ANS: B The patient's assessment indicates that assisted coughing is needed to help remove secretions, which will improve oxygenation. A 2-hour rest period at this time may allow the oxygen saturation to drop further. Humidification will not be helpful unless the secretions can be mobilized. Positioning on the left side may cause a further decrease in oxygen saturation because perfusion will be directed more toward the more poorly ventilated lung

3. A patient is admitted to the burn unit with burns to the head, face, and hands. Initially, wheezes are heard, but an hour later, the lung sounds are decreased and no wheezes are audible. What is the best action for the nurse to take? a. Encourage the patient to cough and auscultate the lungs again. b. Notify the health care provider and prepare for endotracheal intubation. c. Document the results and continue to monitor the patient's respiratory rate. d. Reposition the patient in high-Fowler's position and reassess breath sounds.

ANS: B The patient's history and clinical manifestations suggest airway edema and the health care provider should be notified immediately, so that intubation can be done rapidly. Placing the patient in a more upright position or having the patient cough will not address the problem of airway edema. Continuing to monitor is inappropriate because immediate action should occur. DIF: Cognitive Level: Apply (application) REF: 459 TOP: Nursing Process: Implementation MSC: NCLEX: Physiological Integrity

A patient with rheumatic fever has subcutaneous nodules, erythema marginatum, and polyarthritis. Based on these findings, which nursing diagnosis would be most appropriate? a. Pain related to permanent joint fixation b. Activity intolerance related to arthralgia c. Risk for infection related to open skin lesions d. Risk for impaired skin integrity related to pruritus

ANS: B The patient's joint pain will lead to difficulty with activity. The skin lesions seen in rheumatic fever are not open or pruritic. Although acute joint pain will be a problem for this patient, joint inflammation is a temporary clinical manifestation of rheumatic fever and is not associated with permanent joint changes

A patient with respiratory failure has a respiratory rate of 6 breaths/minute and an oxygen saturation (SpO2) of 88%. The patient is increasingly lethargic. Which intervention will the nurse anticipate? a. Administration of 100% oxygen by non-rebreather mask b. Endotracheal intubation and positive pressure ventilation c. Insertion of a mini-tracheostomy with frequent suctioning d. Initiation of continuous positive pressure ventilation (CPAP)

ANS: B The patient's lethargy, low respiratory rate, and SpO2 indicate the need for mechanical ventilation with ventilator-controlled respiratory rate. Administration of high flow oxygen will not be helpful because the patient's respiratory rate is so low. Insertion of a mini-tracheostomy will facilitate removal of secretions, but it will not improve the patient's respiratory rate or oxygenation. CPAP requires that the patient initiate an adequate respiratory rate to allow adequate gas exchange.

After receiving change-of-shift report on a heart failure unit, which patient should the nurse assess first? a. Patient who is taking carvedilol (Coreg) and has a heart rate of 58 b. Patient who is taking digoxin and has a potassium level of 3.1 mEq/L c. Patient who is taking isosorbide dinitrate/hydralazine (BiDil) and has a headache d. Patient who is taking captopril (Capoten) and has a frequent nonproductive cough

ANS: B The patient's low potassium level increases the risk for digoxin toxicity and potentially fatal dysrhythmias. The nurse should assess the patient for other signs of digoxin toxicity and then notify the health care provider about the potassium level. The other patients also have side effects of their medications, but their symptoms do not indicate potentially life-threatening complications

8. A nurse is caring for a patient who has burns of the ears, head, neck, and right arm and hand. The nurse should place the patient in which position? a. Place the right arm and hand flexed in a position of comfort. b. Elevate the right arm and hand on pillows and extend the fingers. c. Assist the patient to a supine position with a small pillow under the head. d. Position the patient in a side-lying position with rolled towel under the neck.

ANS: B The right hand and arm should be elevated to reduce swelling and the fingers extended to avoid flexion contractures (even though this position may not be comfortable for the patient). The patient with burns of the ears should not use a pillow for the head because this will put pressure on the ears, and the pillow may stick to the ears. Patients with neck burns should not use a pillow because the head should be maintained in an extended position in order to avoid contractures. DIF: Cognitive Level: Apply (application) REF: 462 TOP: Nursing Process: Implementation MSC: NCLEX: Physiological Integrity

A nurse is caring for a patient with ARDS who is being treated with mechanical ventilation and high levels of positive end-expiratory pressure (PEEP). Which assessment finding by the nurse indicates that the PEEP may need to be reduced? a. The patient's PaO2 is 50 mm Hg and the SaO2 is 88%. b. The patient has subcutaneous emphysema on the upper thorax. c. The patient has bronchial breath sounds in both the lung fields. d. The patient has a first-degree atrioventricular heart block with a rate of 58.

ANS: B The subcutaneous emphysema indicates barotrauma caused by positive pressure ventilation and PEEP. Bradycardia, hypoxemia, and bronchial breath sounds are all concerns and will need to be addressed, but they are not specific indications that PEEP should be reduced.

28. A young adult patient who is in the rehabilitation phase 6 months after a severe face and neck burn tells the nurse, "I'm sorry that I'm still alive. My life will never be normal again." Which response by the nurse is best? a. "Most people recover after a burn and feel satisfied with their lives." b. "It's true that your life may be different. What concerns you the most?" c. "It is really too early to know how much your life will be changed by the burn." d. "Why do you feel that way? You will be able to adapt as your recovery progresses."

ANS: B This response acknowledges the patient's feelings and asks for more assessment data that will help in developing an appropriate plan of care to assist the patient with the emotional response to the burn injury. The other statements are accurate, but do not acknowledge the anxiety and depression that the patient is expressing. DIF: Cognitive Level: Apply (application) REF: 470-471 OBJ: Special Questions: Prioritization TOP: Nursing Process: Assessment MSC: NCLEX: Psychosocial Integrity

A patient who is receiving chemotherapy is admitted to the hospital with acute dehydration caused by nausea and vomiting. Which action will the nurse include in the plan of care to best prevent the development of shock, systemic inflammatory response syndrome (SIRS), and multiorgan dysfunction syndrome (MODS)? a. Administer all medications through the patient's indwelling central line. b. Place the patient in a private room. c. Restrict the patient to foods that have been well-cooked or processed. d. Insert a nasogastric (NG) tube for enteral feeding.

B

When planning care for a patient hospitalized with a streptococcal infective endocarditis (IE), which intervention is a priority for the nurse to include? a. Monitor labs for streptococcal antibodies. b. Arrange for placement of a long-term IV catheter. c. Teach the importance of completing all oral antibiotics. d. Encourage the patient to begin regular aerobic exercise.

ANS: B Treatment for IE involves 4 to 6 weeks of IV antibiotic therapy in order to eradicate the bacteria, which will require a long-term IV catheter such as a peripherally inserted central catheter (PICC) line. Rest periods and limiting physical activity to a moderate level are recommended during the treatment for IE. Oral antibiotics are not effective in eradicating the infective bacteria that cause IE. Blood cultures, rather than antibody levels, are used to monitor the effectiveness of antibiotic therapy

Which admission order written by the health care provider for a patient admitted with infective endocarditis (IE) and a fever would be a priority for the nurse to implement? a. Administer ceftriaxone (Rocephin) 1 g IV. b. Order blood cultures drawn from two sites. c. Give acetaminophen (Tylenol) PRN for fever. d. Arrange for a transesophageal echocardiogram.

ANS: B Treatment of the IE with antibiotics should be started as quickly as possible, but it is essential to obtain blood cultures before initiating antibiotic therapy to obtain accurate sensitivity results. The echocardiogram and acetaminophen administration also should be implemented rapidly, but the blood cultures (and then administration of the antibiotic) have the highest priority

7. While the patient's full-thickness burn wounds to the face are exposed, what is the best nursing action to prevent cross contamination? a. Use sterile gloves when removing old dressings. b. Wear gowns, caps, masks, and gloves during all care of the patient. c. Administer IV antibiotics to prevent bacterial colonization of wounds. d. Turn the room temperature up to at least 70° F (20° C) during dressing changes.

ANS: B Use of gowns, caps, masks, and gloves during all patient care will decrease the possibility of wound contamination for a patient whose burns are not covered. When removing contaminated dressings and washing the dirty wound, use nonsterile, disposable gloves. The room temperature should be kept at approximately 85° F for patients with open burn wounds to prevent shivering. Systemic antibiotics are not well absorbed into deep burns because of the lack of circulation. DIF: Cognitive Level: Apply (application) REF: 461 TOP: Nursing Process: Implementation MSC: NCLEX: Physiological Integrity

When the nurse is assessing a patient who is receiving a nitroprusside (Nipride) infusion to treat cardiogenic shock, which finding indicates that the medication is effective? a. No heart murmur is audible. b. Skin is warm, pink, and dry. c. Troponin level is decreased. d. Blood pressure is 90/40 mm Hg.

ANS: B Warm, pink, and dry skin indicates that perfusion to tissues is improved. Since nitroprusside is a vasodilator, the blood pressure may be low even if the medication is effective. Absence of a heart murmur and a decrease in troponin level are not indicators of improvement in shock. DIF: Cognitive Level: Application REF: 1721 | 1723 | 1733-1735 TOP: Nursing Process: Evaluation MSC: NCLEX: Physiological Integrity

The nurse will plan discharge teaching about the need for prophylactic antibiotics when having dental procedures for which patient? a. Patient admitted with a large acute myocardial infarction. b. Patient being discharged after an exacerbation of heart failure. c. Patient who had a mitral valve replacement with a mechanical valve. d. Patient being treated for rheumatic fever after a streptococcal infection.

ANS: C Current American Heart Association guidelines recommend the use of prophylactic antibiotics before dental procedures for patients with prosthetic valves to prevent infective endocarditis (IE). The other patients are not at risk for IE

5. After receiving 2 L of normal saline, the central venous pressure for a patient who has septic shock is 10 mm Hg, but the blood pressure is still 82/40 mm Hg. The nurse will anticipate an order for a. nitroglycerine (Tridil). b. norepinephrine (Levophed). c. sodium nitroprusside (Nipride). d. methylprednisolone (Solu-Medrol).

ANS: B When fluid resuscitation is unsuccessful, vasopressor drugs are administered to increase the systemic vascular resistance (SVR) and blood pressure, and improve tissue perfusion. Nitroglycerin would decrease the preload and further drop cardiac output and BP. Methylprednisolone (Solu-Medrol) is considered if blood pressure does not respond first to fluids and vasopressors. Nitroprusside is an arterial vasodilator and would further decrease SVR. DIF: Cognitive Level: Apply (application) REF: 1643 TOP: Nursing Process: Planning MSC: NCLEX: Physiological Integrity

1. When assessing a patient who spilled hot oil on the right leg and foot, the nurse notes that the skin is dry, pale, hard skin. The patient states that the burn is not painful. What term would the nurse use to document the burn depth? a. First-degree skin destruction b. Full-thickness skin destruction c. Deep partial-thickness skin destruction d. Superficial partial-thickness skin destruction

ANS: B With full-thickness skin destruction, the appearance is pale and dry or leathery and the area is painless because of the associated nerve destruction. Erythema, swelling, and blisters point to a deep partial-thickness burn. With superficial partial-thickness burns, the area is red, but no blisters are present. First-degree burns exhibit erythema, blanching, and pain. DIF: Cognitive Level: Understand (comprehension) REF: 454 TOP: Nursing Process: Assessment MSC: NCLEX: Physiological Integrity

Which actions should the nurse initiate to reduce the risk for ventilator-associated pneumonia (VAP) (select all that apply)? a. Obtain arterial blood gases daily. b. Provide a "sedation holiday" daily. c. Elevate the head of the bed to at least 30°. d. Give prescribed pantoprazole (Protonix). e. Provide oral care with chlorhexidine (0.12%) solution daily.

ANS: B, C, D, E All of these interventions are part of the ventilator bundle that is recommended to prevent VAP. Arterial blood gases may be done daily but are not always necessary and do not help prevent VAP.

14. A patient with burns covering 40% total body surface area (TBSA) is in the acute phase of burn treatment. Which snack would be best for the nurse to offer to this patient? a. Bananas b. Orange gelatin c. Vanilla milkshake d. Whole grain bagel

ANS: C A patient with a burn injury needs high protein and calorie food intake, and the milkshake is the highest in these nutrients. The other choices are not as nutrient-dense as the milkshake. Gelatin is likely high in sugar. The bagel is a good carbohydrate choice, but low in protein. Bananas are a good source of potassium, but are not high in protein and calories. DIF: Cognitive Level: Apply (application) REF: 467 TOP: Nursing Process: Planning MSC: NCLEX: Physiological Integrity

The emergency department (ED) receives notification that a patient who has just been in an automobile accident is being transported to your facility with anticipated arrival in 1 minute. In preparation for the patient's arrival, the nurse will obtain a. 500 mL of 5% albumin. b. lactated Ringer's solution. c. two 14-gauge IV catheters. d. dopamine (Intropin) infusion.

ANS: C A patient with multiple trauma may require fluid resuscitation to prevent or treat hypovolemic shock, so the nurse will anticipate the need for 2 large bore IV lines to administer normal saline. Lactated Ringer's solution should be used cautiously and will not be ordered until the patient has been assessed for possible liver abnormalities. Although colloids may sometimes be used for volume expansion, crystalloids should be used as the initial therapy for fluid resuscitation. Vasopressor infusion is not used as the initial therapy for hypovolemic shock. DIF: Cognitive Level: Application REF: 1731 | 1732 | 1733 TOP: Nursing Process: Planning MSC: NCLEX: Physiological Integrity

8. The emergency department (ED) nurse receives report that a patient involved in a motor vehicle crash is being transported to the facility with an estimated arrival in 1 minute. In preparation for the patient's arrival, the nurse will obtain a. hypothermia blanket. b. lactated Ringer's solution. c. two 14-gauge IV catheters. d. dopamine (Intropin) infusion.

ANS: C A patient with multiple trauma may require fluid resuscitation to prevent or treat hypovolemic shock, so the nurse will anticipate the need for 2 large bore IV lines to administer normal saline. Lactated Ringer's solution should be used cautiously and will not be ordered until the patient has been assessed for possible liver abnormalities. Vasopressor infusion is not used as the initial therapy for hypovolemic shock. Patients in shock need to be kept warm not cool. DIF: Cognitive Level: Apply (application) REF: 1641 TOP: Nursing Process: Planning MSC: NCLEX: Physiological Integrity

19. A patient with extensive electrical burn injuries is admitted to the emergency department. Which prescribed intervention should the nurse implement first? a. Assess oral temperature. b. Check a potassium level. c. Place on cardiac monitor. d. Assess for pain at contact points.

ANS: C After an electrical burn, the patient is at risk for fatal dysrhythmias and should be placed on a cardiac monitor. Assessing the oral temperature is not as important as assessing for cardiac dysrhythmias. Checking the potassium level is important. However, it will take time before the laboratory results are back. The first intervention is to place the patient on a cardiac monitor and assess for dysrhythmias, so that they can be treated if occurring. A decreased or increased potassium level will alert the nurse to the possibility of dysrhythmias. The cardiac monitor will alert the nurse immediately of any dysrhythmias. Assessing for pain is important, but the patient can endure pain until the cardiac monitor is attached. Cardiac dysrhythmias can be lethal. DIF: Cognitive Level: Analyze (analysis) REF: 452 | 456 OBJ: Special Questions: Prioritization TOP: Nursing Process: Implementation MSC: NCLEX: Physiological Integrity

During change-of-shift report on a medical unit, the nurse learns that a patient with aspiration pneumonia who was admitted with respiratory distress has become increasingly agitated. Which action should the nurse take first? a. Give the prescribed PRN sedative drug. b. Offer reassurance and reorient the patient. c. Use pulse oximetry to check the oxygen saturation. d. Notify the health care provider about the patient's status.

ANS: C Agitation may be an early indicator of hypoxemia. The other actions may also be appropriate, depending on the findings about oxygen saturation.

15. A patient has just arrived in the emergency department after an electrical burn from exposure to a high-voltage current. What is the priority nursing assessment? a. Oral temperature b. Peripheral pulses c. Extremity movement d. Pupil reaction to light

ANS: C All patients with electrical burns should be considered at risk for cervical spine injury, and assessments of extremity movement will provide baseline data. The other assessment data are also necessary but not as essential as determining the cervical spine status. DIF: Cognitive Level: Apply (application) REF: 452 OBJ: Special Questions: Prioritization TOP: Nursing Process: Assessment MSC: NCLEX: Physiological Integrity

A patient who is receiving dobutamine (Dobutrex) for the treatment of acute decompensated heart failure (ADHF) has the following nursing interventions included in the plan of care. Which action will be most appropriate for the registered nurse (RN) to delegate to an experienced licensed practical/vocational nurse (LPN/LVN)? a. Assess the IV insertion site for signs of extravasation. b. Teach the patient the reasons for remaining on bed rest. c. Monitor the patient's blood pressure and heart rate every hour. d. Titrate the rate to keep the systolic blood pressure >90 mm Hg.

ANS: C An experienced LPN/LVN would be able to monitor BP and heart rate and would know to report significant changes to the RN. Teaching patients, making adjustments to the drip rate for vasoactive medications, and monitoring for serious complications such as extravasation require RN level education and scope of practice

A patient admitted with bacterial meningitis and a temperature of 102° F (38.8° C) has orders for all of these collaborative interventions. Which action should the nurse take first? a. Administer ceftizoxime (Cefizox) 1 g IV. b. Use a cooling blanket to lower temperature. c. Swap the nasopharyngeal mucosa for cultures. d. Give acetaminophen (Tylenol) 650 mg PO.

ANS: C Antibiotic therapy should be instituted rapidly in bacterial meningitis, but cultures must be done before antibiotics are started. As soon as the cultures are done, the antibiotic should be started. Hypothermia therapy and acetaminophen administration are appropriate but can be started after the other actions are implemented. DIF: Cognitive Level: Application REF: 1440-1441

During discharge teaching with a 68-year-old patient who had a mitral valve replacement with a mechanical valve, the nurse instructs the patient on the a. use of daily aspirin for anticoagulation. b. correct method for taking the radial pulse. c. need for frequent laboratory blood testing. d. need to avoid any physical activity for 1 month.

ANS: C Anticoagulation with warfarin (Coumadin) is needed for a patient with mechanical valves to prevent clotting on the valve. This will require frequent international normalized ratio (INR) testing. Daily aspirin use will not be effective in reducing the risk for clots on the valve. Monitoring of the radial pulse is not necessary after valve replacement. Patients should resume activities of daily living as tolerated

To evaluate the effectiveness of ordered interventions for a patient with ventilatory failure, which diagnostic test will be most useful to the nurse? a. Chest x-ray b. Oxygen saturation c. Arterial blood gas analysis d. Central venous pressure monitoring

ANS: C Arterial blood gas (ABG) analysis is most useful in this setting because ventilatory failure causes problems with CO2 retention, and ABGs provide information about the PaCO2 and pH. The other tests may also be done to help in assessing oxygenation or determining the cause of the patient's ventilatory failure

Which diagnostic test will be most useful to the nurse in determining whether a patient admitted with acute shortness of breath has heart failure? a. Serum troponin b. Arterial blood gases c. B-type natriuretic peptide d. 12-lead electrocardiogram

ANS: C B-type natriuretic peptide (BNP) is secreted when ventricular pressures increase, as they do with heart failure. Elevated BNP indicates a probable or very probable diagnosis of heart failure. A twelve-lead electrocardiogram, arterial blood gases, and troponin may also be used in determining the causes or effects of heart failure but are not as clearly diagnostic of heart failure as BNP

Which action by the nurse will determine if the therapies ordered for a patient with chronic constrictive pericarditis are effective? a. Assess for the presence of a paradoxical pulse. b. Monitor for changes in the patient's sedimentation rate. c. Assess for the presence of jugular venous distention (JVD). d. Check the electrocardiogram (ECG) for ST segment changes.

ANS: C Because the most common finding on physical examination for a patient with chronic constrictive pericarditis is jugular venous distention, a decrease in JVD indicates improvement. Paradoxical pulse, ST-segment ECG changes, and changes in sedimentation rates occur with acute pericarditis but are not expected in chronic constrictive pericarditis

23. Which finding about a patient who is receiving vasopressin (Pitressin) to treat septic shock is most important for the nurse to communicate to the health care provider? a. The patient's urine output is 18 mL/hr. b. The patient's heart rate is 110 beats/minute. c. The patient is complaining of chest pain. d. The patient's peripheral pulses are weak.

ANS: C Because vasopressin is a potent vasoconstrictor, it may decrease coronary artery perfusion. The other information is consistent with the patient's diagnosis and should be reported to the health care provider but does not indicate a need for a change in therapy. DIF: Cognitive Level: Apply (application) REF: 1643 OBJ: Special Questions: Prioritization TOP: Nursing Process: Assessment MSC: NCLEX: Physiological Integrity

A patient with heart failure has a new order for captopril (Capoten) 12.5 mg PO. After administering the first dose and teaching the patient about the drug, which statement by the patient indicates that teaching has been effective? a. "I will be sure to take the medication with food." b. "I will need to eat more potassium-rich foods in my diet." c. "I will call for help when I need to get up to use the bathroom." d. "I will expect to feel more short of breath for the next few days."

ANS: C Captopril can cause hypotension, especially after the initial dose, so it is important that the patient not get up out of bed without assistance until the nurse has had a chance to evaluate the effect of the first dose. The angiotensin-converting enzyme (ACE) inhibitors are potassium sparing, and the nurse should not teach the patient to purposely increase sources of dietary potassium. Increased shortness of breath is expected with the initiation of b-adrenergic blocker therapy for heart failure, not for ACE inhibitor therapy. ACE inhibitors are best absorbed when taken an hour before eating

Norepinephrine (Levophed) has been ordered for the patient in hypovolemic shock. Before administering the drug, the nurse ensures that the a. patient's heart rate is less than 100. b. patient has received adequate fluid replacement. c. patient's urine output is within normal range. d. patient is not receiving other sympathomimetic drugs.

B

When caring for a patient with infective endocarditis of the tricuspid valve, the nurse should monitor the patient for the development of a. flank pain. b. splenomegaly. c. shortness of breath. d. mental status changes.

ANS: C Embolization from the tricuspid valve would cause symptoms of pulmonary embolus. Flank pain, changes in mental status, and splenomegaly would be associated with embolization from the left-sided valves

4. A patient with severe burns has crystalloid fluid replacement ordered using the Parkland formula. The initial volume of fluid to be administered in the first 24 hours is 30,000 mL. The initial rate of administration is 1875 mL/hr. After the first 8 hours, what rate should the nurse infuse the IV fluids? a. 350 mL/hour b. 523 mL/hour c. 938 mL/hour d. 1250 mL/hour

ANS: C Half of the fluid replacement using the Parkland formula is administered in the first 8 hours and the other half over the next 16 hours. In this case, the patient should receive half of the initial rate, or 938 mL/hr. DIF: Cognitive Level: Apply (application) REF: 460 TOP: Nursing Process: Implementation MSC: NCLEX: Physiological Integrity

24. The nurse is reviewing laboratory results on a patient who had a large burn 48 hours ago. Which result requires priority action by the nurse? a. Hematocrit 53% b. Serum sodium 147 mEq/L c. Serum potassium 6.1 mEq/L d. Blood urea nitrogen 37 mg/dL

ANS: C Hyperkalemia can lead to fatal dysrhythmias and indicates that the patient requires cardiac monitoring and immediate treatment to lower the potassium level. The other laboratory values are also abnormal and require changes in treatment, but they are not as immediately life threatening as the elevated potassium level. DIF: Cognitive Level: Apply (application) REF: 458 OBJ: Special Questions: Prioritization TOP: Nursing Process: Assessment MSC: NCLEX: Physiological Integrity

A patient has recently started on digoxin (Lanoxin) in addition to furosemide (Lasix) and captopril (Capoten) for the management of heart failure. Which assessment finding by the home health nurse is a priority to communicate to the health care provider? a. Presence of 1 to 2+ edema in the feet and ankles b. Palpable liver edge 2 cm below the ribs on the right side c. Serum potassium level 3.0 mEq/L after 1 week of therapy d. Weight increase from 120 pounds to 122 pounds over 3 days

ANS: C Hypokalemia can predispose the patient to life-threatening dysrhythmias (e.g., premature ventricular contractions), and potentiate the actions of digoxin and increase the risk for digoxin toxicity, which can also cause life-threatening dysrhythmias. The other data indicate that the patient's heart failure requires more effective therapies, but they do not require nursing action as rapidly as the low serum potassium level

A 21-year-old woman is scheduled for percutaneous transluminal balloon valvuloplasty to treat mitral stenosis. Which information should the nurse include when explaining the advantages of valvuloplasty over valve replacement to the patient? a. Biologic valves will require immunosuppressive drugs after surgery. b. Mechanical mitral valves need to be replaced sooner than biologic valves. c. Lifelong anticoagulant therapy will be needed after mechanical valve replacement. d. Ongoing cardiac care by a health care provider is not necessary after valvuloplasty.

ANS: C Long-term anticoagulation therapy is needed after mechanical valve replacement, and this would restrict decisions about career and childbearing in this patient. Mechanical valves are durable and last longer than biologic valves. All valve repair procedures are palliative, not curative, and require lifelong health care. Biologic valves do not activate the immune system, and immunosuppressive therapy is not needed

While caring for a patient who has just been admitted with meningococcal meningitis, the RN observes all of the following. Which one requires action by the RN? a. The bedrails at the head and foot of the bed are both elevated. b. The patient receives a regular diet from the dietary department. c. The nursing assistant goes into the patient's room without a mask. d. The lights in the patient's room are turned off and the blinds are shut.

ANS: C Meningococcal meningitis is spread by respiratory secretions, so it is important to maintain respiratory isolation as well as standard precautions. Because the patient may be confused and weak, bedrails should be elevated at both the food and head of the bed. Low light levels in the room decrease pain caused by photophobia. Nutrition is an important aspect of care in a patient with meningitis. DIF: Cognitive Level: Application REF: 1453-1455

The nurse plans discharge teaching for a patient with chronic heart failure who has prescriptions for digoxin (Lanoxin) and hydrochlorothiazide (HydroDIURIL). Appropriate instructions for the patient include a. limit dietary sources of potassium. b. take the hydrochlorothiazide before bedtime. c. notify the health care provider if nausea develops. d. skip the digoxin if the pulse is below 60 beats/minute.

ANS: C Nausea is an indication of digoxin toxicity and should be reported so that the provider can assess the patient for toxicity and adjust the digoxin dose, if necessary. The patient will need to include potassium-containing foods in the diet to avoid hypokalemia. Patients should be taught to check their pulse daily before taking the digoxin and if the pulse is less than 60, to call their provider before taking the digoxin. Diuretics should be taken early in the day to avoid sleep disruption

3. A 19-year-old patient with massive trauma and possible spinal cord injury is admitted to the emergency department (ED). Which assessment finding by the nurse will help confirm a diagnosis of neurogenic shock? a. Inspiratory crackles. b. Cool, clammy extremities. c. Apical heart rate 45 beats/min. d. Temperature 101.2° F (38.4° C).

ANS: C Neurogenic shock is characterized by hypotension and bradycardia. The other findings would be more consistent with other types of shock. DIF: Cognitive Level: Understand (comprehension) REF: 1634 TOP: Nursing Process: Assessment MSC: NCLEX: Physiological Integrity

4. An older patient with cardiogenic shock is cool and clammy and hemodynamic monitoring indicates a high systemic vascular resistance (SVR). Which intervention should the nurse anticipate doing next? a. Increase the rate for the dopamine (Intropin) infusion. b. Decrease the rate for the nitroglycerin (Tridil) infusion. c. Increase the rate for the sodium nitroprusside (Nipride) infusion. d. Decrease the rate for the 5% dextrose in normal saline (D5/.9 NS) infusion.

ANS: C Nitroprusside is an arterial vasodilator and will decrease the SVR and afterload, which will improve cardiac output. Changes in the D5/.9 NS and nitroglycerin infusions will not directly decrease SVR. Increasing the dopamine will tend to increase SVR. DIF: Cognitive Level: Apply (application) REF: 1644 TOP: Nursing Process: Planning MSC: NCLEX: Physiological Integrity

A patient who has chronic heart failure tells the nurse, "I was fine when I went to bed, but I woke up in the middle of the night feeling like I was suffocating!" The nurse will document this assessment finding as a. orthopnea. b. pulsus alternans. c. paroxysmal nocturnal dyspnea. d. acute bilateral pleural effusion.

ANS: C Paroxysmal nocturnal dyspnea is caused by the reabsorption of fluid from dependent body areas when the patient is sleeping and is characterized by waking up suddenly with the feeling of suffocation. Pulsus alternans is the alternation of strong and weak peripheral pulses during palpation. Orthopnea indicates that the patient is unable to lie flat because of dyspnea. Pleural effusions develop over a longer time period

Which statement by a patient with restrictive cardiomyopathy indicates that the nurse's discharge teaching about self-management has been most effective? a. "I will avoid taking aspirin or other antiinflammatory drugs." b. "I will need to limit my intake of salt and fluids even in hot weather." c. "I will take antibiotics when my teeth are cleaned at the dental office." d. "I should begin an exercise program that includes things like biking or swimming."

ANS: C Patients with restrictive cardiomyopathy are at risk for infective endocarditis and should use prophylactic antibiotics for any procedure that may cause bacteremia. The other statements indicate a need for more teaching by the nurse. Dehydration and vigorous exercise impair ventricular filling in patients with restrictive cardiomyopathy. There is no need to avoid salt (unless ordered), aspirin, or NSAIDs

A patient develops increasing dyspnea and hypoxemia 2 days after heart surgery. To determine whether the patient has acute respiratory distress syndrome (ARDS) or pulmonary edema caused by heart failure, the nurse will plan to assist with a. obtaining a ventilation-perfusion scan. b. drawing blood for arterial blood gases. c. insertion of a pulmonary artery catheter. d. positioning the patient for a chest x-ray.

ANS: C Pulmonary artery wedge pressures are normal in the patient with ARDS because the fluid in the alveoli is caused by increased permeability of the alveolar-capillary membrane rather than by the backup of fluid from the lungs (as occurs in cardiogenic pulmonary edema). The other tests will not help in differentiating cardiogenic from noncardiogenic pulmonary edema.

Which intervention will the nurse include in the plan of care for a patient who has cardiogenic shock? a. Avoid elevating head of bed. b. Check temperature every 2 hours. c. Monitor breath sounds frequently. d. Assess skin for flushing and itching.

ANS: C Since pulmonary congestion and dyspnea are characteristics of cardiogenic shock, the nurse should assess the breath sounds frequently. The head of the bed is usually elevated to decrease dyspnea. Elevated temperature and flushing or itching of the skin are not typical of cardiogenic shock. DIF: Cognitive Level: Application REF: 1721 TOP: Nursing Process: Implementation MSC: NCLEX: Physiological Integrity

IV sodium nitroprusside (Nipride) is ordered for a patient with acute pulmonary edema. During the first hours of administration, the nurse will need to titrate the nitroprusside rate if the patient develops a. ventricular ectopy. b. a dry, hacking cough. c. a systolic BP <90 mm Hg. d. a heart rate <50 beats/minute.

ANS: C Sodium nitroprusside is a potent vasodilator, and the major adverse effect is severe hypotension. Coughing and bradycardia are not adverse effects of this medication. Nitroprusside does not cause increased ventricular ectopy

The nurse working on the heart failure unit knows that teaching an older female patient with newly diagnosed heart failure is effective when the patient states that a. she will take furosemide (Lasix) every day at bedtime. b. the nitroglycerin patch is applied when any chest pain develops. c. she will call the clinic if her weight goes from 124 to 128 pounds in a week. d. an additional pillow can help her sleep if she is feeling short of breath at night.

ANS: C Teaching for a patient with heart failure includes information about the need to weigh daily and notify the health care provider about an increase of 3 pounds in 2 days or 3 to 5 pounds in a week. Nitroglycerin patches are used primarily to reduce preload (not to prevent chest pain) in patients with heart failure and should be used daily, not on an "as needed" basis. Diuretics should be taken earlier in the day to avoid nocturia and sleep disturbance. The patient should call the clinic if increased orthopnea develops, rather than just compensating by further elevating the head of the bed

A patient with chronic heart failure who is taking a diuretic and an angiotensin-converting enzyme (ACE) inhibitor and who is on a low-sodium diet tells the home health nurse about a 5-pound weight gain in the last 3 days. The nurse's priority action will be to a. have the patient recall the dietary intake for the last 3 days. b. ask the patient about the use of the prescribed medications. c. assess the patient for clinical manifestations of acute heart failure. d. teach the patient about the importance of restricting dietary sodium.

ANS: C The 5-pound weight gain over 3 days indicates that the patient's chronic heart failure may be worsening. It is important that the patient be assessed immediately for other clinical manifestations of decompensation, such as lung crackles. A dietary recall to detect hidden sodium in the diet, reinforcement of sodium restrictions, and assessment of medication compliance may be appropriate interventions but are not the first nursing actions indicated.

Which assessment finding obtained by the nurse when assessing a patient with acute pericarditis should be reported immediately to the health care provider? a. Pulsus paradoxus 8 mm Hg b. Blood pressure (BP) of 168/94 c. Jugular venous distention (JVD) to jaw level d. Level 6 (0 to 10 scale) chest pain with a deep breath

ANS: C The JVD indicates that the patient may have developed cardiac tamponade and may need rapid intervention to maintain adequate cardiac output. Hypertension would not be associated with complications of pericarditis, and the BP is not high enough to indicate that there is any immediate need to call the health care provider. A pulsus paradoxus of 8 mm Hg is normal. Level 6/10 chest pain should be treated but is not unusual with pericarditis

The community health nurse is developing a program to decrease the incidence of meningitis in adolescents and young adults. Which nursing action is most important? a. Vaccinate 11- and 12-year-old children against Haemophilus influenzae. b. Emphasize the importance of hand washing to prevent spread of infection. c. Immunize adolescents and college freshman against Neisseria meningitides. d. Encourage adolescents and young adults to avoid crowded areas in the winter.

ANS: C The Neisseria meningitides vaccination is recommended for children ages 11 and 12, unvaccinated teens entering high school, and college freshmen. Hand washing may help decrease the spread of bacteria, but it is not as effective as immunization. Vaccination with Haemophilus influenzae is for infants and toddlers. Because adolescents and young adults are in school or the workplace, avoiding crowds is not realistic. DIF: Cognitive Level: Application REF: 1453-1455

21. The patient with neurogenic shock is receiving a phenylephrine (Neo-Synephrine) infusion through a right forearm IV. Which assessment finding obtained by the nurse indicates a need for immediate action? a. The patient's heart rate is 58 beats/minute. b. The patient's extremities are warm and dry. c. The patient's IV infusion site is cool and pale. d. The patient's urine output is 28 mL over the last hour.

ANS: C The coldness and pallor at the infusion site suggest extravasation of the phenylephrine. The nurse should discontinue the IV and, if possible, infuse the medication into a central line. An apical pulse of 58 is typical for neurogenic shock but does not indicate an immediate need for nursing intervention. A 28-mL urinary output over 1 hour would require the nurse to monitor the output over the next hour, but an immediate change in therapy is not indicated. Warm, dry skin is consistent with early neurogenic shock, but it does not indicate a need for a change in therapy or immediate action. DIF: Cognitive Level: Apply (application) REF: 1643 OBJ: Special Questions: Prioritization TOP: Nursing Process: Assessment MSC: NCLEX: Physiological Integrity

Which topic will the nurse plan to include in discharge teaching for a patient with systolic heart failure and an ejection fraction of 33%? a. Need to begin an aerobic exercise program several times weekly b. Use of salt substitutes to replace table salt when cooking and at the table c. Benefits and side effects of angiotensin-converting enzyme (ACE) inhibitors d. Importance of making an annual appointment with the primary care provider

ANS: C The core measures for the treatment of heart failure established by The Joint Commission indicate that patients with an ejection fraction (EF) <40% receive an ACE inhibitor to decrease the progression of heart failure. Aerobic exercise may not be appropriate for a patient with this level of heart failure, salt substitutes are not usually recommended because of the risk of hyperkalemia, and the patient will need to see the primary care provider more frequently than annually

While admitting an 82-year-old with acute decompensated heart failure to the hospital, the nurse learns that the patient lives alone and sometimes confuses the "water pill" with the "heart pill." When planning for the patient's discharge the nurse will facilitate a a. consult with a psychologist. b. transfer to a long-term care facility. c. referral to a home health care agency. d. arrangements for around-the-clock care.

ANS: C The data about the patient suggest that assistance in developing a system for taking medications correctly at home is needed. A home health nurse will assess the patient's home situation and help the patient develop a method for taking the two medications as directed. There is no evidence that the patient requires services such as a psychologist consult, long-term care, or around-the-clock home care

20. A patient who has been involved in a motor vehicle crash arrives in the emergency department (ED) with cool, clammy skin; tachycardia; and hypotension. Which intervention ordered by the health care provider should the nurse implement first? a. Insert two large-bore IV catheters. b. Initiate continuous electrocardiogram (ECG) monitoring. c. Provide oxygen at 100% per non-rebreather mask. d. Draw blood to type and crossmatch for transfusions.

ANS: C The first priority in the initial management of shock is maintenance of the airway and ventilation. ECG monitoring, insertion of IV catheters, and obtaining blood for transfusions should also be rapidly accomplished but only after actions to maximize oxygen delivery have been implemented. DIF: Cognitive Level: Apply (application) REF: 1641 OBJ: Special Questions: Prioritization TOP: Nursing Process: Implementation MSC: NCLEX: Physiological Integrity

While assessing a 68-year-old with ascites, the nurse also notes jugular venous distention (JVD) with the head of the patient's bed elevated 45 degrees. The nurse knows this finding indicates a. decreased fluid volume. b. jugular vein atherosclerosis. c. increased right atrial pressure. d. incompetent jugular vein valves.

ANS: C The jugular veins empty into the superior vena cava and then into the right atrium, so JVD with the patient sitting at a 45-degree angle reflects increased right atrial pressure. JVD is an indicator of excessive fluid volume (increased preload), not decreased fluid volume. JVD is not caused by incompetent jugular vein valves or atherosclerosis

25. After reviewing the information shown in the accompanying figure for a patient with pneumonia and sepsis, which information is most important to report to the health care provider? Petechiae noted on chest and legs Cracks heard bilaterally in lung bases no redness or swelling at IV site BUN 34 Hemaocrit 30% platelets 50,000/mL Temp 100F Pulse 102 Respirations 26 BP 110/60 O2 93% on 2L O2 NC a. Temperature and IV site appearance b. Oxygen saturation and breath sounds c. Platelet count and presence of petechiae d. Blood pressure, pulse rate, respiratory rate.

ANS: C The low platelet count and presence of petechiae suggest that the patient may have disseminated intravascular coagulation and that multiple organ dysfunction syndrome (MODS) is developing. The other information will also be discussed with the health care provider but does not indicate that the patient's condition is deteriorating or that a change in therapy is needed immediately. DIF: Cognitive Level: Analyze (analysis) REF: 1640 OBJ: Special Questions: Prioritization TOP: Nursing Process: Assessment MSC: NCLEX: Physiological Integrity

A patient recovering from heart surgery develops pericarditis and complains of level 6 (0 to 10 scale) chest pain with deep breathing. Which ordered PRN medication will be the most appropriate for the nurse to give? a. Fentanyl 1 mg IV b. IV morphine sulfate 4 mg c. Oral ibuprofen (Motrin) 600 mg d. Oral acetaminophen (Tylenol) 650 mg

ANS: C The pain associated with pericarditis is caused by inflammation, so nonsteroidal antiinflammatory drugs (NSAIDs) (e.g., ibuprofen) are most effective. Opioid analgesics are usually not used for the pain associated with pericarditis

An outpatient who has chronic heart failure returns to the clinic after 2 weeks of therapy with metoprolol (Toprol XL). Which assessment finding is most important for the nurse to report to the health care provider? a. 2+ pedal edema b. Heart rate of 56 beats/minute c. Blood pressure (BP) of 88/42 mm Hg d. Complaints of fatigue

ANS: C The patient's BP indicates that the dose of metoprolol may need to be decreased because of hypotension. Bradycardia is a frequent adverse effect of b-adrenergic blockade, but the rate of 56 is not unusual with â-adrenergic blocker therapy. b-Adrenergic blockade initially will worsen symptoms of heart failure in many patients, and patients should be taught that some increase in symptoms, such as fatigue and edema, is expected during the initiation of therapy with this class of drugs

The nurse documents the vital signs for a patient admitted 2 days ago with gram-negative sepsis: temperature 101.2° F, blood pressure 90/56 mm Hg, pulse 92, respirations 34. Which action should the nurse take next? a. Give the scheduled IV antibiotic. b. Give the PRN acetaminophen (Tylenol). c. Obtain oxygen saturation using pulse oximetry. d. Notify the health care provider of the patient's vital signs.

ANS: C The patient's increased respiratory rate in combination with the admission diagnosis of gram-negative sepsis indicates that acute respiratory distress syndrome (ARDS) may be developing. The nurse should check for hypoxemia, a hallmark of ARDS. The health care provider should be notified after further assessment of the patient. Giving the scheduled antibiotic and the PRN acetaminophen will also be done, but they are not the highest priority for a patient who may be developing ARDS.

2. On admission to the burn unit, a patient with an approximate 25% total body surface area (TBSA) burn has the following initial laboratory results: Hct 58%, Hgb 18.2 mg/dL (172 g/L), serum K+ 4.9 mEq/L (4.8 mmol/L), and serum Na+ 135 mEq/L (135 mmol/L). Which action will the nurse anticipate taking now? a. Monitor urine output every 4 hours. b. Continue to monitor the laboratory results. c. Increase the rate of the ordered IV solution. d. Type and crossmatch for a blood transfusion.

ANS: C The patient's laboratory data show hemoconcentration, which may lead to a decrease in blood flow to the microcirculation unless fluid intake is increased. Because the hematocrit and hemoglobin are elevated, a transfusion is inappropriate, although transfusions may be needed after the emergent phase once the patient's fluid balance has been restored. On admission to a burn unit, the urine output would be monitored more often than every 4 hours; likely every 1 hour. DIF: Cognitive Level: Apply (application) REF: 257 TOP: Nursing Process: Planning MSC: NCLEX: Physiological Integrity

When caring for a patient with mitral valve stenosis, it is most important that the nurse assess for a. diastolic murmur. b. peripheral edema. c. shortness of breath on exertion. d. right upper quadrant tenderness.

ANS: C The pressure gradient changes in mitral stenosis lead to fluid backup into the lungs, resulting in hypoxemia and dyspnea. The other findings also may be associated with mitral valve disease but are not indicators of possible hypoxemia

A patient with a history of chronic heart failure is admitted to the emergency department (ED) with severe dyspnea and a dry, hacking cough. Which action should the nurse do first? a. Auscultate the abdomen. b. Check the capillary refill. c. Auscultate the breath sounds. d. Assess the level of orientation.

ANS: C This patient's severe dyspnea and cough indicate that acute decompensated heart failure (ADHF) is occurring. ADHF usually manifests as pulmonary edema, which should be detected and treated immediately to prevent ongoing hypoxemia and cardiac/respiratory arrest. The other assessments will provide useful data about the patient's volume status and also should be accomplished rapidly, but detection (and treatment) of pulmonary complications is the priority

12. A nurse is assessing a patient who is receiving a nitroprusside (Nipride) infusion to treat cardiogenic shock. Which finding indicates that the medication is effective? a. No new heart murmurs b. Decreased troponin level c. Warm, pink, and dry skin d. Blood pressure 92/40 mm Hg

ANS: C Warm, pink, and dry skin indicates that perfusion to tissues is improved. Since nitroprusside is a vasodilator, the blood pressure may be low even if the medication is effective. Absence of a heart murmur and a decrease in troponin level are not indicators of improvement in shock. DIF: Cognitive Level: Apply (application) REF: 1644 TOP: Nursing Process: Evaluation MSC: NCLEX: Physiological Integrity

After receiving 1000 mL of normal saline, the central venous pressure for a patient who has septic shock is 10 mm Hg, but the blood pressure is still 82/40 mm Hg. The nurse will anticipate the administration of a. nitroglycerine (Tridil). b. drotrecogin alpha (Xigris). c. norepinephrine (Levophed). d. sodium nitroprusside (Nipride).

ANS: C When fluid resuscitation is unsuccessful, vasopressor drugs are administered to increase the systemic vascular resistance (SVR) and improve tissue perfusion. Nitroglycerin would decrease the preload and further drop cardiac output and BP. Drotrecogin alpha may decrease inappropriate inflammation and help prevent systemic inflammatory response syndrome, but it will not directly improve blood pressure. Nitroprusside is an arterial vasodilator and would further decrease SVR. DIF: Cognitive Level: Application REF: 1731 | 1733-1735 TOP: Nursing Process: Planning MSC: NCLEX: Physiological Integrity

A patient with chronic obstructive pulmonary disease (COPD) arrives in the emergency department complaining of shortness of breath and dyspnea on minimal exertion. Which assessment finding by the nurse is most important to report to the health care provider? a. The patient has bibasilar lung crackles. b. The patient is sitting in the tripod position. c. The patient's respirations have decreased from 30 to 10 breaths/minute. d. The patient's pulse oximetry indicates an O2 saturation of 91%.

ANS: D A decrease in respiratory rate in a patient with respiratory distress suggests the onset of fatigue and a high risk for respiratory arrest. Therefore immediate action such as positive pressure ventilation is needed. Patients who are experiencing respiratory distress frequently sit in the tripod position because it decreases the work of breathing. Crackles in the lung bases may be the baseline for a patient with COPD. An oxygen saturation of 91% is common in patients with COPD and will provide adequate gas exchange and tissue oxygenation.

Following an acute myocardial infarction, a previously healthy 63-year-old develops clinical manifestations of heart failure. The nurse anticipates discharge teaching will include information about a. digitalis preparations. b. b-adrenergic blockers. c. calcium channel blockers. d. angiotensin-converting enzyme (ACE) inhibitors.

ANS: D ACE inhibitor therapy is currently recommended to prevent the development of heart failure in patients who have had a myocardial infarction and as a first-line therapy for patients with chronic heart failure. Digoxin therapy for heart failure is no longer considered a first-line measure, and digoxin is added to the treatment protocol when therapy with other medications such as ACE-inhibitors, diuretics, and b-adrenergic blockers is insufficient. Calcium channel blockers are not generally used in the treatment of heart failure. The b-adrenergic blockers are not used as initial therapy for new onset heart failure

Pain management for the burn patient is most effective when: A. opioids are administered on a set schedule around the clock B. the patient has as much control over the management of the pain as possible C. there is a flexibility to administer opioids within a dosage and frequency range D. painful dressing changes are delayed until the patient's pain is totally relieved

B

The nurse is obtaining a health history from a 24-year-old patient with hypertrophic cardiomyopathy (HC). Which information obtained by the nurse is most important? a. The patient has a history of a recent upper respiratory infection. b. The patient has a family history of coronary artery disease (CAD). c. The patient reports using cocaine a "couple of times" as a teenager. d. The patient's 29-year-old brother died from a sudden cardiac arrest.

ANS: D About half of all cases of HC have a genetic basis, and it is the most common cause of sudden cardiac death in otherwise healthy young people. The information about the patient's brother will be helpful in planning care (such as an automatic implantable cardioverter-defibrillator [AICD]) for the patient and in counseling other family members. The patient should be counseled against the use of stimulant drugs, but the limited past history indicates that the patient is not at current risk for cocaine use. Viral infections and CAD are risk factors for dilated cardiomyopathy, but not for HC

17. A patient who has burns on the arms, legs, and chest from a house fire has become agitated and restless 8 hours after being admitted to the hospital. Which action should the nurse take first? a. Stay at the bedside and reassure the patient. b. Administer the ordered morphine sulfate IV. c. Assess orientation and level of consciousness. d. Use pulse oximetry to check the oxygen saturation.

ANS: D Agitation in a patient who may have suffered inhalation injury might indicate hypoxia, and this should be assessed by the nurse first. Administration of morphine may be indicated if the nurse determines that the agitation is caused by pain. Assessing level of consciousness and orientation is also appropriate but not as essential as determining whether the patient is hypoxemic. Reassurance is not helpful to reduce agitation in a hypoxemic patient. DIF: Cognitive Level: Apply (application) REF: 458 | 464-465 OBJ: Special Questions: Prioritization TOP: Nursing Process: Implementation MSC: NCLEX: Physiological Integrity

A patient who has just been admitted with pulmonary edema is scheduled to receive the following medications. Which medication should the nurse question before giving? a. Furosemide (Lasix) 60 mg b. Captopril (Capoten) 25 mg c. Digoxin (Lanoxin) 0.125 mg d. Carvedilol (Coreg) 3.125 mg

ANS: D Although carvedilol is appropriate for the treatment of chronic heart failure, it is not used for patients with acute decompensated heart failure (ADHF) because of the risk of worsening the heart failure. The other medications are appropriate for the patient with ADHF

27. Which action will the nurse include in the plan of care for a patient in the rehabilitation phase after a burn injury to the right arm and chest? a. Keep the right arm in a position of comfort. b. Avoid the use of sustained-release narcotics. c. Teach about the purpose of tetanus immunization. d. Apply water-based cream to burned areas frequently.

ANS: D Application of water-based emollients will moisturize new skin and decrease flakiness and itching. To avoid contractures, the joints of the right arm should be positioned in an extended position, which is not the position of comfort. Patients may need to continue the use of opioids during rehabilitation. Tetanus immunization would have been given during the emergent phase of the burn injury. DIF: Cognitive Level: Apply (application) REF: 468-469 TOP: Nursing Process: Planning MSC: NCLEX: Physiological Integrity

When assessing a patient with chronic obstructive pulmonary disease (COPD), the nurse finds a new onset of agitation and confusion. Which action should the nurse take first? a. Notify the health care provider. b. Check pupils for reaction to light. c. Attempt to calm and reorient the patient. d. Assess oxygenation using pulse oximetry.

ANS: D Because agitation and confusion are frequently the initial indicators of hypoxemia, the nurse's initial action should be to assess oxygen saturation. The other actions are also appropriate, but assessment of oxygenation takes priority over other assessments and notification of the health care provider.

A patient with acute respiratory distress syndrome (ARDS) who is intubated and receiving mechanical ventilation develops a right pneumothorax. Which action will the nurse anticipate taking next? a. Increase the tidal volume and respiratory rate. b. Increase the fraction of inspired oxygen (FIO2). c. Perform endotracheal suctioning more frequently. d. Lower the positive end-expiratory pressure (PEEP).

ANS: D Because barotrauma is associated with high airway pressures, the level of PEEP should be decreased. The other actions will not decrease the risk for pneumothorax.

13. Which assessment information is most important for the nurse to obtain to evaluate whether treatment of a patient with anaphylactic shock has been effective? a. Heart rate b. Orientation c. Blood pressure d. Oxygen saturation

ANS: D Because the airway edema that is associated with anaphylaxis can affect airway and breathing, the oxygen saturation is the most critical assessment. Improvements in the other assessments will also be expected with effective treatment of anaphylactic shock. DIF: Cognitive Level: Apply (application) REF: 1646 TOP: Nursing Process: Evaluation MSC: NCLEX: Physiological Integrity

The nurse is caring for a patient who is receiving IV furosemide (Lasix) and morphine for the treatment of acute decompensated heart failure (ADHF) with severe orthopnea. Which clinical finding is the best indicator that the treatment has been effective? a. Weight loss of 2 pounds in 24 hours b. Hourly urine output greater than 60 mL c. Reduction in patient complaints of chest pain d. Reduced dyspnea with the head of bed at 30 degrees

ANS: D Because the patient's major clinical manifestation of ADHF is orthopnea (caused by the presence of fluid in the alveoli), the best indicator that the medications are effective is a decrease in dyspnea with the head of the bed at 30 degrees. The other assessment data also may indicate that diuresis or improvement in cardiac output has occurred, but are not as specific to evaluating this patient's response

The nurse obtains a health history from a 65-year-old patient with a prosthetic mitral valve who has symptoms of infective endocarditis (IE). Which question by the nurse is most appropriate? a. "Do you have a history of a heart attack?" b. "Is there a family history of endocarditis?" c. "Have you had any recent immunizations?" d. "Have you had dental work done recently?"

ANS: D Dental procedures place the patient with a prosthetic mitral valve at risk for infective endocarditis (IE). Myocardial infarction (MI), immunizations, and a family history of endocarditis are not risk factors for IE.

10. Esomeprazole (Nexium) is prescribed for a patient who incurred extensive burn injuries 5 days ago. Which nursing assessment would best evaluate the effectiveness of the medication? a. Bowel sounds b. Stool frequency c. Abdominal distention d. Stools for occult blood

ANS: D H2 blockers and proton pump inhibitors are given to prevent Curling's ulcer in the patient who has suffered burn injuries. Proton pump inhibitors usually do not affect bowel sounds, stool frequency, or appetite. DIF: Cognitive Level: Apply (application) REF: 465 TOP: Nursing Process: Evaluation MSC: NCLEX: Physiological Integrity

Which nursing interventions included in the care of a mechanically ventilated patient with acute respiratory failure can the registered nurse (RN) delegate to an experienced licensed practical/vocational nurse (LPN/LVN) working in the intensive care unit? a. Assess breath sounds every hour. b. Monitor central venous pressures. c. Place patient in the prone position. d. Insert an indwelling urinary catheter.

ANS: D Insertion of indwelling urinary catheters is included in LPN/LVN education and scope of practice and can be safely delegated to an LPN/LVN who is experienced in caring for critically ill patients. Placing a patient who is on a ventilator in the prone position requires multiple staff, and should be supervised by an RN. Assessment of breath sounds and obtaining central venous pressures require advanced assessment skills and should be done by the RN caring for a critically ill patient.

A patient has been admitted with meningococcal meningitis. Which observation by the nurse requires action? a. The bedrails at the head and foot of the bed are both elevated. b. The patient receives a regular diet from the dietary department. c. The lights in the patient's room are turned off and the blinds are shut. d. Unlicensed assistive personnel enter the patient's room without a mask.

ANS: D Meningococcal meningitis is spread by respiratory secretions, so it is important to maintain respiratory isolation as well as standard precautions. Because the patient may be confused and weak, bedrails should be elevated at both the foot and head of the bed. Low light levels in the room decrease pain caused by photophobia. Nutrition is an important aspect of care in a patient with meningitis

When teaching the patient with newly diagnosed heart failure about a 2000-mg sodium diet, the nurse explains that foods to be restricted include a. canned and frozen fruits. b. fresh or frozen vegetables. c. eggs and other high-protein foods. d. milk, yogurt, and other milk products.

ANS: D Milk and yogurt naturally contain a significant amount of sodium, and intake of these should be limited for patients on a diet that limits sodium to 2000 mg daily. Other milk products, such as processed cheeses, have very high levels of sodium and are not appropriate for a 2000-mg sodium diet. The other foods listed have minimal levels of sodium and can be eaten without restriction

11. The nurse is reviewing the medication administration record (MAR) on a patient with partial-thickness burns. Which medication is best for the nurse to administer before scheduled wound debridement? a. Ketorolac (Toradol) b. Lorazepam (Ativan) c. Gabapentin (Neurontin) d. Hydromorphone (Dilaudid)

ANS: D Opioid pain medications are the best choice for pain control. The other medications are used as adjuvants to enhance the effects of opioids. DIF: Cognitive Level: Apply (application) REF: 467 TOP: Nursing Process: Implementation MSC: NCLEX: Physiological Integrity

The nurse caring for a patient in shock notifies the health care provider of the patient's deteriorating status when the patient's ABG results include a. pH 7.48, PaCO2 33 mm Hg. b. pH 7.33, PaCO2 30 mm Hg. c. pH 7.41, PaCO2 50 mm Hg. d. pH 7.38, PaCO2 45 mm Hg.

B

The nurse establishes the nursing diagnosis of ineffective health maintenance related to lack of knowledge regarding long-term management of rheumatic fever when a 30-year-old recovering from rheumatic fever without carditis says which of the following? a. "I will need prophylactic antibiotic therapy for 5 years." b. "I will need to take aspirin or ibuprofen (Motrin) to relieve my joint pain." c. "I will call the doctor if I develop excessive fatigue or difficulty breathing." d. "I will be immune to further episodes of rheumatic fever after this infection."

ANS: D Patients with a history of rheumatic fever are more susceptible to a second episode. Patients with rheumatic fever without carditis require prophylaxis until age 20 and for a minimum of 5 years. The other patient statements are correct and would not support the nursing diagnosis of ineffective health maintenance

16. When the nurse educator is evaluating the skills of a new registered nurse (RN) caring for patients experiencing shock, which action by the new RN indicates a need for more education? a. Placing the pulse oximeter on the ear for a patient with septic shock b. Keeping the head of the bed flat for a patient with hypovolemic shock c. Increasing the nitroprusside (Nipride) infusion rate for a patient with a high SVR d. Maintaining the room temperature at 66° to 68° F for a patient with neurogenic shock

ANS: D Patients with neurogenic shock may have poikilothermia. The room temperature should be kept warm to avoid hypothermia. The other actions by the new RN are appropriate. DIF: Cognitive Level: Apply (application) REF: 1634 | 1636 OBJ: Special Questions: Delegation TOP: Nursing Process: Evaluation MSC: NCLEX: Safe and Effective Care Environment

6. To evaluate the effectiveness of the pantoprazole (Protonix) ordered for a patient with systemic inflammatory response syndrome (SIRS), which assessment will the nurse perform? a. Auscultate bowel sounds. b. Palpate for abdominal pain. c. Ask the patient about nausea. d. Check stools for occult blood.

ANS: D Proton pump inhibitors are given to decrease the risk for stress ulcers in critically ill patients. The other assessments also will be done, but these will not help in determining the effectiveness of the pantoprazole administration. DIF: Cognitive Level: Apply (application) REF: 1646 TOP: Nursing Process: Evaluation MSC: NCLEX: Physiological Integrity

When assessing a 53-year-old patient with bacterial meningitis, the nurse obtains the following data. Which finding should be reported immediately to the health care provider? a. The patient exhibits nuchal rigidity. b. The patient has a positive Kernig's sign. c. The patient's temperature is 101° F (38.3° C). d. The patient's blood pressure is 88/42 mm Hg.

ANS: D Shock is a serious complication of meningitis, and the patient's low blood pressure indicates the need for interventions such as fluids or vasopressors. Nuchal rigidity and a positive Kernig's sign are expected with bacterial meningitis. The nurse should intervene to lower the temperature, but this is not as life threatening as the hypotension

When assessing a patient with bacterial meningitis, the nurse obtains the following data. Which finding should be reported immediately to the health care provider? a. The patient has a positive Kernig's sign. b. The patient complains of having a stiff neck. c. The patient's temperature is 101° F (38.3° C). d. The patient's blood pressure is 86/42 mm Hg.

ANS: D Shock is a serious complication of meningitis, and the patient's low blood pressure indicates the need for interventions such as fluids or vasopressors. Nuchal rigidity and a positive Kernig's sign are expected with bacterial meningitis. The nurse should intervene to lower the temperature, but this is not as life threatening as the hypotension. DIF: Cognitive Level: Application REF: 1452-1453

5. During the emergent phase of burn care, which assessment will be most useful in determining whether the patient is receiving adequate fluid infusion? a. Check skin turgor. b. Monitor daily weight. c. Assess mucous membranes. d. Measure hourly urine output.

ANS: D When fluid intake is adequate, the urine output will be at least 0.5 to 1 mL/kg/hour. The patient's weight is not useful in this situation because of the effects of third spacing and evaporative fluid loss. Mucous membrane assessment and skin turgor also may be used, but they are not as adequate in determining that fluid infusions are maintaining adequate perfusion. DIF: Cognitive Level: Apply (application) REF: 460-461 TOP: Nursing Process: Evaluation MSC: NCLEX: Physiological Integrity

The nurse has identified a nursing diagnosis of acute pain related to inflammatory process for a patient with acute pericarditis. The priority intervention by the nurse for this problem is to a. teach the patient to take deep, slow breaths to control the pain. b. force fluids to 3000 mL/day to decrease fever and inflammation. c. remind the patient to request opioid pain medication every 4 hours. d. place the patient in Fowler's position, leaning forward on the overbed table.

ANS: D Sitting upright and leaning forward frequently will decrease the pain associated with pericarditis. Forcing fluids will not decrease the inflammation or pain. Taking deep breaths will tend to increase pericardial pain. Opioids are not very effective at controlling pain caused by acute inflammatory conditions and are usually ordered PRN. The patient would receive scheduled doses of a nonsteroidal antiinflammatory drug (NSAID).

A patient admitted with acute dyspnea is newly diagnosed with dilated cardiomyopathy. Which information will the nurse plan to teach the patient about managing this disorder? a. A heart transplant should be scheduled as soon as possible. b. Elevating the legs above the heart will help relieve dyspnea. c. Careful compliance with diet and medications will prevent heart failure. d. Notify the doctor about any symptoms of heart failure such as shortness of breath.

ANS: D The patient should be instructed to notify the health care provider about any worsening of heart failure symptoms. Because dilated cardiomyopathy does not respond well to therapy, even patients with good compliance with therapy may have recurrent episodes of heart failure. Elevation of the legs above the heart will worsen symptoms (although this approach is appropriate for a patient with hypertrophic cardiomyopathy). The patient with terminal or end-stage cardiomyopathy may consider heart transplantation

The ____________ prevents excessive atrial impulses from reaching the ventricles, receives current after it has gone through the atria, and if the SA fails it can take over at a rate of 40-60 bpm.

AV node

Seeing or feeling pulsations at the 5 cardiac landmarks is an ____________ finding, unless palpating the apical pulse.

Abnormal

23. A patient who was found unconscious in a burning house is brought to the emergency department by ambulance. The nurse notes that the patient's skin color is bright red. Which action should the nurse take first? a. Insert two large-bore IV lines. b. Check the patient's orientation. c. Assess for singed nasal hair and dark oral mucous membranes. d. Place the patient on 100% oxygen using a non-rebreather mask.

ANS: D The patient's history and skin color suggest carbon monoxide poisoning, which should be treated by rapidly starting oxygen at 100%. The other actions can be taken after the action to correct gas exchange. DIF: Cognitive Level: Apply (application) REF: 455 OBJ: Special Questions: Prioritization TOP: Nursing Process: Implementation MSC: NCLEX: Physiological Integrity

The nurse is caring for a 78-year-old patient who was hospitalized 2 days earlier with community-acquired pneumonia. Which assessment information is most important to communicate to the health care provider? a. Scattered crackles bilaterally in the posterior lung bases. b. Persistent cough that is productive of blood-tinged sputum. c. Temperature of 101.5° F (38.6° C) after 2 days of IV antibiotic therapy. d. Decreased oxygen saturation to 90% with 100% O2 by non-rebreather mask.

ANS: D The patient's low SpO2 despite receiving a high fraction of inspired oxygen (FIO2) indicates the possibility of acute respiratory distress syndrome (ARDS). The patient's blood-tinged sputum and scattered crackles are not unusual in a patient with pneumonia, although they do require continued monitoring. The continued temperature elevation indicates a possible need to change antibiotics, but this is not as urgent a concern as the progression toward hypoxemia despite an increase in O2 flow rate.

20. Eight hours after a thermal burn covering 50% of a patient's total body surface area (TBSA) the nurse assesses the patient. Which information would be a priority to communicate to the health care provider? a. Blood pressure is 95/48 per arterial line. b. Serous exudate is leaking from the burns. c. Cardiac monitor shows a pulse rate of 108. d. Urine output is 20 mL per hour for the past 2 hours.

ANS: D The urine output should be at least 0.5 to 1.0 mL/kg/hr during the emergent phase, when the patient is at great risk for hypovolemic shock. The nurse should notify the health care provider because a higher IV fluid rate is needed. BP during the emergent phase should be greater than 90 systolic, and the pulse rate should be less than 120. Serous exudate from the burns is expected during the emergent phase. DIF: Cognitive Level: Apply (application) REF: 461 OBJ: Special Questions: Prioritization TOP: Nursing Process: Assessment MSC: NCLEX: Physiological Integrity

12. A young adult patient who is in the rehabilitation phase after having deep partial-thickness face and neck burns has a nursing diagnosis of disturbed body image. Which statement by the patient indicates that the problem is resolving? a. "I'm glad the scars are only temporary." b. "I will avoid using a pillow, so my neck will be OK." c. "I bet my boyfriend won't even want to look at me anymore." d. "Do you think dark beige makeup foundation would cover this scar on my cheek?"

ANS: D The willingness to use strategies to enhance appearance is an indication that the disturbed body image is resolving. Expressing feelings about the scars indicates a willingness to discuss appearance, but not resolution of the problem. Because deep partial-thickness burns leave permanent scars, a statement that the scars are temporary indicates denial rather than resolution of the problem. Avoiding using a pillow will help prevent contractures, but it does not address the problem of disturbed body image. DIF: Cognitive Level: Apply (application) REF: 469 TOP: Nursing Process: Evaluation MSC: NCLEX: Psychosocial Integrity

After receiving change-of-shift report on a medical unit, which patient should the nurse assess first? a. A patient with cystic fibrosis who has thick, green-colored sputum b. A patient with pneumonia who has crackles bilaterally in the lung bases c. A patient with emphysema who has an oxygen saturation of 90% to 92% d. A patient with septicemia who has intercostal and suprasternal retractions

ANS: D This patient's history of septicemia and labored breathing suggest the onset of ARDS, which will require rapid interventions such as administration of oxygen and use of positive pressure ventilation. The other patients should also be assessed as quickly as possible, but their assessment data are typical of their disease processes and do not suggest deterioration in their status.

Which action could the nurse delegate to unlicensed assistive personnel (UAP) trained as electrocardiogram (ECG) technicians working on the cardiac unit? a. Select the best lead for monitoring a patient with an admission diagnosis of Dressler syndrome. b. Obtain a list of herbal medications used at home while admitting a new patient with pericarditis. c. Teach about the need to monitor the weight daily for a patient who has hypertrophic cardiomyopathy. d. Check the heart monitor for changes in rhythm while a patient who had a valve replacement ambulates.

ANS: D Under the supervision of registered nurses (RNs), UAP check the patient's cardiac monitor and obtain information about changes in heart rate and rhythm with exercise. Teaching and obtaining information about home medications (prescribed or complementary) and selecting the best leads for monitoring patients require more critical thinking and should be done by the RN

The ____________ determines the patency of radial and ulnar arteries. You should occlude the radial/ulnar arteries, have the patient pump the fist and open the hand, and you should see perfusion. Blood will return via the artery that is not occluded.

Allen's Test

Why would J.F have low hemoglobin and hematocrit?

Anemia associated with endocarditis

A thrill or bruit may be auscultated over an ____________.

Aneurysm

While caring for a 23-year-old patient with mitral valve prolapse (MVP) without valvular regurgitation, the nurse determines that discharge teaching has been effective when the patient states that it will be necessary to a. take antibiotics before any dental appointments. b. limit physical activity to avoid stressing the heart. c. take an aspirin a day to prevent clots from forming on the valve. d. avoid use of over-the-counter (OTC) medications that contain stimulant drugs.

ANS: D Use of stimulant medications should be avoided by patients with MVP because these may exacerbate symptoms. Daily aspirin and restricted physical activity are not needed by patients with mild MVP. Antibiotic prophylaxis is needed for patients with MVP with regurgitation but will not be necessary for this patient

o Vasodilator drugs have been shown to improve survival in HF .•

by increasing venous capacity, improving EF through improved ventricular contraction, slowing the process of ventricular failure, and decreasing heart size

What are signs and symptoms of infective endocarditis?

hyperthermia, enlarged spleen, hearth failure and cardiac murmur.

Which action by the nurse changing the dressings on the client who has burns on the right arm, the left arm, and the upper chest is most effective at preventing auto-contamination? A. Changing gloves after cleaning and dressing one wound area before cleaning and dressing the next wound area. B. Using sterile gloves to remove the old dressings and changing to fresh sterile gloves before applying the new dressings. C. Ensuring that the blood pressure cuff used on another client is thoroughly cleaned before using it on this client. D. Warning the client's family not to bring fresh fruit and vegetables or house plants into the client's environment.

Answer: A Rationale: Auto-contamination is the movement of organisms from one body area on a client to another body area. The use of sterile versus clean gloves for routine wound care varies by agency and is a matter of debate. Regardless of sterility, change gloves when handling wounds on different areas of the body and between handling old and new dressings. So, if the nurse changed to fresh gloves after removing old dressings but kept the fresh gloves on while dressing all the burn wound areas, he or she greatly increases the risk for translocating organisms from one burn wound to another, resulting in auto-contamination. Responses C and D address cross-contamination that occurs between people.

Which client response does the nurse interpret as an indication of fluid resuscitation adequacy? A. Decreasing pulse pressure B. Decreasing urine specific gravity C. Decreasing core body temperature D. Increasing respiratory rate and depth

Answer: B Rationale: Urine output is the most sensitive noninvasive measure of fluid resuscitation adequacy. An increase in urine output is a positive sign; however, so is a decreasing urine specific gravity. As urine output increases, the concentration of the urine decreases, leading to a decreased urine specific gravity. A decreasing pulse pressure often indicates a fall in systolic pressure, which would not indicate fluid resuscitation adequacy. A decreasing core body temperature is related to changes in the inflammatory response or metabolism and not an indication of fluid resuscitation adequacy. An increasing respiratory rate could indicate pulmonary edema but not fluid resuscitation adequacy. The increased respiratory depth may indicate other positive changes but not adequacy of fluid resuscitation.

For which type of burn injury is it most important for the nurse to assess the client for a respiratory injury? A. Hot liquid scald burn B. Liquid chemical burn C. Electrical burn D. Dry heat burn

Answer: D Rationale: Direct injury to the lung from contact with flames, scalding hot liquids, liquid chemicals, or electrical current rarely occurs. Rather, respiratory problems are caused by superheated air, steam, toxic fumes, or smoke. Although it is possible for an electric current to pass through the lungs, it seldom causes injury.

The ____________ should be inspected for contour, pulsations, lifts, heaves, retractions, and palpated for apical pulse.

Anterior chest wall

HF etiology

Any interference with normal regulating mechanisms for CO. Changes lead to decreased ventricular function. 1) preload 2) afterload 3) myocardial contractility 4) heart rate

____________ can produce hoarseness from pressure on the laryngeal nerve, or dysphagia from pressure on the esophagus.

Aortic aneurysms

____________ is inadequate arterial circulation, can cause claudication, diminished pulses, absence of hair on toes, pallor when extremities are elevated, and rubor when extremities are dependent.

Arterial insufficiency

____________ is an abnormal connection between an artery and vein commonly caused by piercing injuries. Characterized by a machinery sounding murmur, bulging veins, decreased BP/increased HR, cyanosis, clubbing.

Arteriovenous fistula

A patient has 25% TBSA burned from a car fire. His wounds have been debrided and covered with a silver-impregnated dressing. The nurse's priority intervention for wound care would be to: A. reapply a new dressing without disturbing the wound bed B. observe the wound for signs of infection during dressing changes C. apply cool compresses for pain relief in between dressing changes D. wash the wound aggressively with soap and water three times a day

B

A patient is treated in the emergency department (ED) for shock of unknown etiology. The first action by the nurse should be to a. check the blood pressure. b. obtain an oxygen saturation. c. attach a cardiac monitor. d. check level of consciousness.

B

Clinical sign of HF (9)

FACES: fatigue, limited activities, chest congestion, edema, SOB 1) fatigue 2) dyspnea 3) tachycardia 4) edema 5) nocturia 6) skin changes 7) behavioral changes 8) chest pain 9) weight changes 10) muscle wasting, thin arms and legs

The nurse would recognize which of the following clinical manifestations as suggestive of sepsis? A) Respiratory rate of seven breaths per minute B) Hyperglycemia in the absence of diabetes C) Sudden diuresis unrelated to drug therapy D) Bradycardia with sudden increase in blood pressure

B Hyperglycemia in patients with no history of diabetes is a diagnostic criterion for sepsis. Oliguria, not diuresis, typically accompanies sepsis along with tachycardia and tachypnea.

What are the symptoms of a stroke?

Facial droop Weakness on one side slurred speech

A patient's localized infection has progressed to the point where septic shock is now suspected. Which of the following is an appropriate treatment modality for this patient? A) Insulin infusion B) Aggressive fluid resuscitation C) Intravenous administration of epinephrine D) Administration of nitrates and β-adrenergic blockers

B Patients in septic shock require large amounts of fluid replacement. Nitrates and β-adrenergic blockers are most often used in the treatment of patients in cardiogenic shock. Epinephrine is indicated in anaphylactic shock, and insulin infusion is not normally necessary in the treatment of septic shock (but can be).

When caring for a patient in acute septic shock, the nurse would anticipate A) Administering osmotic and/or loop diuretics. B) Infusing large amounts of intravenous fluids. C) Administering intravenous diphenhydramine (Benadryl). D) Assisting with insertion of a ventricular assist device (VAD).

B Septic shock is characterized by a decreased circulating blood volume. Volume expansion with the administration of intravenous fluids is the cornerstone of therapy. The administration of diuretics is inappropriate. VADs are useful for cardiogenic shock, not septic shock. Diphenhydramine (Benadryl) may be used for anaphylactic shock, but would not be helpful with septic shock.

A patient with a diagnosis of heart failure has been started on a nitroglycerin patch by his primary care provider. What should this patient be taught to avoid? A High-potassium foods B Drugs to treat erectile dysfunction C Nonsteroidal antiinflammatory drugs D Over-the-counter H2 -receptor blockers

B Drugs to treat erectile dysfunction The use of erectile drugs concurrent with nitrates creates a risk of severe hypotension and possibly death. High-potassium foods, NSAIDs, and H2-receptor blockers do not pose a risk in combination with nitrates.

What should the nurse recognize as an indication for the use of dopamine (Intropin) in the care of a patient with heart failure? A Acute anxiety B Hypotension and tachycardia C Peripheral edema and weight gain D Paroxysmal nocturnal dyspnea (PND)

B Hypotension and tachycardia Dopamine is a β-adrenergic agonist whose inotropic action is used for treatment of severe heart failure accompanied by hemodynamic instability. Such a state may be indicated by tachycardia accompanied by hypotension. PND, anxiety, edema, and weight gain are common signs and symptoms of heart failure, but these do not necessarily warrant the use of dopamine.

Palpate the lower extremities for ____________, ____________, ____________, and ____________ pulses for amplitude.

Femoral, popliteal, posterior tibialis, dorsalis pedis

The home care nurse visits a 73-year-old Hispanic woman with chronic heart failure. Which clinical manifestations, if assessed by the nurse, would indicate acute decompensated heart failure (pulmonary edema)? A Fatigue, orthopnea, and dependent edema B Severe dyspnea and blood-streaked, frothy sputum C Temperature is 100.4o F and pulse is 102 beats/minute D Respirations 26 breaths/minute despite oxygen by nasal cannula

B Severe dyspnea and blood-streaked, frothy sputum Clinical manifestations of pulmonary edema include anxiety, pallor, cyanosis, clammy and cold skin, severe dyspnea, use of accessory muscles of respiration, a respiratory rate > 30 breaths per minute, orthopnea, wheezing, and coughing with the production of frothy, blood-tinged sputum. Auscultation of the lungs may reveal crackles, wheezes, and rhonchi throughout the lungs. The heart rate is rapid, and blood pressure may be elevated or decreased.

The nurse is preparing to administer digoxin to a patient with heart failure. In preparation, laboratory results are reviewed with the following findings: sodium 139 mEq/L, potassium 5.6 mEq/L, chloride 103 mEq/L, and glucose 106 mg/dL. What should the nurse do next? A Withhold the daily dose until the following day. B Withhold the dose and report the potassium level. C Give the digoxin with a salty snack, such as crackers. D Give the digoxin with extra fluids to dilute the sodium level.

B Withhold the dose and report the potassium level The normal potassium level is 3.5 to 5.0 mEq/L. The patient is hyperkalemic, which makes the patient more prone to digoxin toxicity. For this reason, the nurse should withhold the dose and report the potassium level. The physician may order the digoxin to be given once the potassium level has been treated and decreases to within normal range.

A client with a burn injury due to a house fire is admitted to the burn unit. The client's family asks the nurse why the client received a tetanus toxoid injection on admission. What is the nurse's best response to the client's family member? A. "The last tetanus injection was less than 5 years ago." B. "Burn wound conditions promote the growth of Clostridium tetani." C. "The wood in the fire had many nails, which penetrated the skin." D. "The injection was prescribed to prevent infection from Pseudomonas."

B. "Burn wound conditions promote the growth of Clostridium tetani." Burn wound conditions promote the growth of Clostridium tetani, and all burn clients are at risk for this dangerous infection. Tetanus toxoid enhances acquired immunity to C. tetani, so this agent is routinely given when the client is admitted to the hospital. Regardless of when the last tetanus injection is given, it is still given on admission to prevent C. tetani. The fact that there were many nails in the wood in the fire is irrelevant. Tetanus toxoid injection does not prevent Pseudomonas infection.

A patient in the clinic reports a recent episode of dysphasia and left-sided weakness at home that resolved after 2 hours. The nurse will anticipate teaching the patient about a. alteplase (tPA). b. aspirin (Ecotrin). c. warfarin (Coumadin). d. nimodipine (Nimotop).

B. Following a transient ischemic attack (TIA), patients typically are started on medications such as aspirin to inhibit platelet function and decrease stroke risk. tPA is used for acute ischemic stroke. Coumadin is usually used for patients with atrial fibrillation. Nimodipine is used to prevent cerebral vasospasm after a subarachnoid hemorrhage.

The nurse is caring for a client with burns to the face. Which statement by the client requires further evaluation by the nurse? A. "I am getting used to looking at myself." B. "I don't know what I will do when people stare at me." C. "I know that I will never look the way I used to, even after the scars heal." D. "My spouse does not stare at the scars as much now as in the beginning."

B. "I don't know what I will do when people stare at me." The statement about not knowing what to do when people stare indicates that the client is not coping effectively; the nurse should assist the client in exploring coping techniques. Visits from friends and short public appearances before discharge may help the client begin adjusting to this problem. The statement that the client is getting used to looking at himself or herself, the realization that he or she will always look different than before, and stating that the client's spouse doesn't stare at the scars as much all indicate that the client is coping effectively. Community reintegration programs can assist the psychosocial and physical recovery of the client with serious burns.

he nurse is providing discharge teaching to the postoperative client. Which statement by the client indicates the need for further education? A. "I must call my doctor if I develop a fever." B. "I will call my provider if I have any pain." C. "If the incision site begins to bleed, I must immediately notify my doctor." D. "If the site becomes red and swollen, I will call my health care provider."

B. "I will call my provider if I have any pain." (Some pain is normal for the postoperative client.)

Which newly admitted client does the nurse consider to be at highest risk for development of sepsis? A. 75 y/o man with hypertension and early Alzheimer's disease. B. 68 y/o woman 2 days postoperative from bowel surgery C. 80 y/o community-dwelling man with no other health problems undergoing cataract surgery D. 54 y/o woman with moderate asthma and severe degenerative joint disease of the right knee

B. 68 y/o woman 2 days postoperative from bowel surgery (The 68-year-old woman has several risk factors. First she is an older adult, and immune function decreases with age. The greatest risk factor is that she has just had bowel surgery. Not only does major surgery further reduce the immune response, the bowel cannot be "sterilized" for surgery. Therefore the usual bacteria of the bowel have the chance to escape the site and enter the bloodstream when the bowel is disrupted. Reference: p. 822, Safe and Effective Care Environment)

A female patient has left-sided hemiplegia following an ischemic stroke that she experienced 4 days earlier. How should the nurse best promote the health of the patient's integumentary system? A. Position the patient on her weak side the majority of the time. B. Alternate the patient's positioning between supine and side-lying. C. Avoid the use of pillows in order to promote independence in positioning. D. Establish a schedule for the message of areas where skin breakdown emerges.

B. Alternate the patient's positioning between supine and side-lying. A position change schedule should be established for stroke patients. An example is side-back-side, with a maximum duration of 2 hours for any position. The patient should be positioned on the weak or paralyzed side for only 30 minutes. Pillows may be used to facilitate positioning. Areas of skin breakdown should never be massaged.

Situation: A client with septic shock has been started on dopamine (Intropin) at 12 mcg/kg/min. Which response indicates a positive outcome? A. Hourly urine output 10 to 12 mL/hr B. B/P 90/60 and MAP 70 C. Blood glucose 245 D. Serum creatinine 3.6 mg/dl

B. B/P 90/60 and MAP 70 (Dopamine improves blood flow by increasing peripheral resistance, which increases blood pressure - a positive response.)

A patient with massive trauma and possible spinal cord injury is admitted to the ED. Which findings by the nurse will help confirm a diagnosis of neurogenic shock? A. Cool, clammy skin B. Inspiratory crackles C. Apical heart rate 48beats/minute D. Temperature 101.2F (38.4C)

C. Apical heart rate 48beats/minute (Neurogenic shock is characterized by hypotension and bradycardia. The other findings would be more consistent with other types of shock.)

A client with septic shock has been started on dopamine (Intropin) at 12 mcg/kg/min. Which response indicates a positive outcome? A. Hourly urine output 10 to 12 mL/hr B. Blood pressure 90/60 mm Hg and mean arterial pressure 70 mm Hg C. Blood glucose 245 mg/dL D. Serum creatinine 3.6 mg/dL

B. Blood pressure 90/60 mm Hg and mean arterial pressure 70 mm Hg Dopamine improves blood flow by increasing peripheral resistance, which increases blood pressure—a positive response in this case. Urine output less than 30 mL/hr or 0.5 mL/kg/hr and elevations in serum creatinine indicate poor tissue perfusion to the kidney and are a negative consequence of shock, not a positive response. Although a blood glucose of 245 mg/dL is an abnormal finding, dopamine increases blood pressure and myocardial contractility, not glucose levels.

Which problem in the clients below best demonstrates the highest risk for hypovolemic shock? A. Client receiving a blood transfusion B. Client with severe ascites C. Client with myocardial infarction D. Client with syndrome of inappropriate antidiuretic hormone (SIADH) secretion

B. Client with severe ascites Fluid shifts from vascular to intra-abdominal may cause decreased circulating blood volume and poor tissue perfusion. Volume depletion is only one reason why a person may require a blood transfusion; anemia is another. The client receiving a blood transfusion does not have as high a risk as the client with severe ascites. Myocardial infarction results in tissue necrosis in the heart muscle; no blood or fluid losses occur. Owing to excess antidiuretic hormone secretion, the client with SIADH will retain fluid and therefore is not at risk for hypovolemic shock.

Which of the these problems in the clients below best demonstrates highest risk for hypovolemic shock? A. Client receiving a blood transfusion B. Client with severe ascites C. Client with myocardial infarction D. Client with syndrome of inappropriate antipyretic hormone (ADH) secretion (SIADH)

B. Client with severe ascites (Fluid shifts from vascular to intra-abdominal may cause decreased circulating blood volume and poor tissue perfusion.)

Which laboratory result is most indicative of effective therapy for sepsis? A. Decreased hemoglobin B. Decreased segmented neutrophil count C. Increased numbers of monocytes D. Increased platelet count

B. Decreased segmented neutrophil count (A decreased segmented neutrophil count, indicates sepsis is resolving.)

Which laboratory result is seen in late sepsis? A. Decreased serum lactate B. Decreased segmented neutrophil count C. Increased numbers of monocytes D. Increased platelet count

B. Decreased segmented neutrophil count A decreased segmented neutrophil count is indicative of late sepsis. Serum lactate is increased in late sepsis. Monocytosis is usually seen in diseases such as tuberculosis and Rocky Mountain spotted fever. An increased platelet count does not indicate sepsis; late in sepsis, platelets may decrease due to consumptive coagulopathy.

Several weeks after a stroke, a 50-year-old male patient has impaired awareness of bladder fullness, resulting in urinary incontinence. Which nursing intervention will be best to include in the initial plan for an effective bladder training program? a. Limit fluid intake to 1200 mL daily to reduce urine volume. b. Assist the patient onto the bedside commode every 2 hours. c. Perform intermittent catheterization after each voiding to check for residual urine. d. Use an external "condom" catheter to protect the skin and prevent embarrassment.

B. Developing a regular voiding schedule will prevent incontinence and may increase patient awareness of a full bladder. A 1200 mL fluid restriction may lead to dehydration. Intermittent catheterization and use of a condom catheter are appropriate in the acute phase of stroke, but should not be considered solutions for long-term management because of the risks for urinary tract infection (UTI) and skin breakdown.

The nurse plans to administer an antibiotic to the client newly admitted with septic shock. What action should the nurse take? A. Administer the antibiotic immediately. B. Ensure that blood cultures were drawn. C. Obtain signature for informed consent. D. Take the client's vital signs.

B. Ensure that blood cultures were drawn. (Cultures must be taken to identify the organism for more targeted antibiotic treatment before antibiotics are administered.)

The nurse plans to administer an antibiotic to a client newly admitted with septic shock. What action does the nurse take first? A. Administer the antibiotic immediately. B. Ensure that blood cultures were drawn. C. Obtain signature for informed consent. D. Take the client's vital signs.

B. Ensure that blood cultures were drawn. Cultures must be taken to identify the organism for more targeted antibiotic treatment before antibiotics are administered. Antibiotics are not administered until after all cultures are taken. A signed consent is not needed for medication administration. Monitoring the client's vital signs is important, but the antibiotic must be administered within 1 to 3 hours; timing is essential.

The nurse on a burn unit has just received change-of-shift report about these clients. Which client does the nurse assess first? A. Adult client admitted a week ago with deep partial-thickness burns over 35% of the body who is reporting pain B. Firefighter with smoke inhalation and facial burns who has just arrived on the unit and whispers, "I can't catch my breath!" C. An electrician who suffered external burn injuries a month ago and is asking the nurse to contact the health care provider immediately about discharge plans D. Older adult client admitted yesterday with partial- and full-thickness burns over 40% of the body who is receiving IV fluids at 250 mL/hr

B. Firefighter with smoke inhalation and facial burns who has just arrived on the unit and whispers, "I can't catch my breath!" Smoke inhalation and facial burns are associated with airway inflammation and obstruction; the client with difficulty breathing needs immediate assessment and intervention. Although the client admitted a week ago with deep partial-thickness burns is reporting pain, this client does not require immediate assessment. The electrician who suffered burn injuries a month ago is stable and has been in the burn unit for a month, so the client's condition does not warrant that the nurse should assess this client first. The older adult client admitted yesterday with burns over 40% of the body is stable; he is receiving IV fluids and does not need to be assessed first.

A bounding carotid pulse may indicate ____________.

Fever

Classic symptoms of bacterial meningitis include a. papilledema and psychomotor seizures b. high fever, nuchal rigidity, and severe headache c. behavioral changes with memory loss and lethargy d. positive Kernig's and Brudzinski's signs and hemiparesis

B. High fever, severe headache, nuchal rigidity, and positive Brudzinski's and Kernig's signs are such classic symptoms of meningitis that they are usually considered diagnostic for meningitis. Other symptoms, such as papilledema, generalized seizures, hemiparesis, and decreased LOC, may occur as complications of increased ICP and cranial nerve dysfunction.

The nurse would expect to find what clinical manifestation in a patient admitted with a left-sided stroke? A. Impulsivity B. Impaired speech C. Left-side neglect D. Short attention span

B. Impaired speech Clinical manifestations of left-sided brain damage include right hemiplegia, impaired speech/language, impaired right/left discrimination, and slow and cautious performance. Impulsivity, left-sided neglect, and short attention span are all manifestations of right-sided brain damage.

The physician orders alteplase (Activase) for a 58-year-old man diagnosed with an acute ischemic stroke. Which nursing action is most appropriate? A. Administer the medication by an IV route at 15 mL/hr for 24 hours. B. Insert two or three large-bore IV catheters before administering the medication. C. If gingival bleeding occurs, discontinue the medication and notify the physician. D. Reduce the medication infusion rate for a systolic blood pressure above 180 mm Hg.

B. Insert two or three large-bore IV catheters before administering the medication. Before giving alteplase, the nurse should start two or three large bore IVs. Bleeding is a major complication with fibrinolytic therapy, and venipunctures should not be attempted after alteplase is administered. Altepase is administered IV with an initial bolus dose followed by an infusion of the remaining medication within the next 60 minutes. Gingival bleeding is a minor complication and may be controlled with pressure or ice packs. Control of blood pressure is critical prior to altepase administration and for the following 24 hours. Before administering altepase, a systolic pressure above 180 mm Hg or diastolic pressure above 110 mm Hg requires aggressive blood pressure treatment to reduce the risk of cerebral hemorrhage.

A client is in the resuscitation phase of burn injury. Which route does the nurse use to administer pain medication to the client? A. Intramuscular B. Intravenous C. Sublingual D. Topical

B. Intravenous During the resuscitation phase, the IV route is used for giving opioid drugs because of problems with absorption from the muscle and stomach. When these agents are given by the intramuscular or subcutaneous route, they remain in the tissue spaces and do not relieve pain. In addition, when edema is present, all doses are rapidly absorbed at once when the fluid shift is resolving. This delayed absorption can result in lethal blood levels of analgesics. The sublingual route may not be effective, and because the skin is too damaged, the topical route is not indicated for administering drugs to the client in the resuscitation phase of burn injury.

How does the nurse caring for a client with septic shock recognize that severe tissue hypoxia is present? A. PaCO2 58 mm Hg B. Lactate 9.0 mmol/L C. Partial thromboplastin time 64 seconds D. Potassium 2.8 mEq/L

B. Lactate 9.0 mmol/L Poor tissue oxygenation at the cellular level causes anaerobic metabolism, with the by-product of lactic acid. Elevated partial pressure of carbon dioxide occurs with hypoventilation, which may be related to respiratory muscle fatigue, secretions, and causes other than hypoxia. Coagulation times reflect the ability of the blood to clot, not oxygenation at the cellular level. Elevation in potassium appears in septic shock due to acidosis; this value is decreased and is not consistent with septic shock.

How does the nurse caring for the client with septic shock recognize that severe tissue hypoxia is present? A. PaCO2 58 mm Hg B. Lactate level 9.0 mmol/L C. Partial thromboplastin time (PTT) 64 seconds D. Potassium 2.8 mEq/L

B. Lactate level 9.0 mmol/L (Poor tissue oxygenation at the cellular level causes anaerobic metabolism, with the by-product of lactic acid production.)

Which clinical symptoms in a postoperative client indicate early sepsis with an excellent recovery rate if treated? A. Localized erythema and edema B. Low-grade fever and mild hypotension C. Low oxygen saturation rate and decreased cognition D. Reduced urinary output and increased respiratory rate

B. Low-grade fever and mild hypotension Low-grade fever and mild hypotension indicate very early sepsis, but with treatment, the probability of recovery is high. Localized erythema and edema indicate local infection. A low oxygen saturation rate and decreased cognition indicate active (not early) sepsis. Reduced urinary output and increased respiratory rate indicate severe sepsis.

The nurse teaches the client and family that which clinical symptoms in the postoperative client indicate early sepsis with an excellent recovery rate if treated? A. Localized erythema and edema B. Low-grade fever and mild hypotension C. Low oxygen saturation rate and decreased cognition D. Reduced urinary output and an increased respiratory rate

B. Low-grade fever and mild hypotension (Reduced urinary output and an increased respiratory rate indicate severe sepsis.)

How does the nurse recognize that a positive outcome has occurred when administering plasma protein fraction (Plasmanate)? A. Urine output 20 to 30 mL/hr for the last 4 hours. B. MAP 70 C. Albumin level 3.5 D. Hemoglobin 7.6

B. MAP 70 (Plasmanate expands the blood volume and helps maintain MAP greater than 65, which is a desired outcome in shock.)

How does the nurse recognize that a positive outcome has occurred when administering plasma protein fraction (Plasmanate)? A. Urine output 20 to 30 mL/hr for the last 4 hours B. Mean arterial pressure (MAP) 70 mm Hg C. Albumin 3.5 g/dL D. Hemoglobin 7.6 g/dL

B. Mean arterial pressure (MAP) 70 mm Hg Plasmanate expands the blood volume and helps maintain MAP greater than 65 mm Hg, which is a desired outcome in shock. Urine output should be 0.5 mL/kg/hr, or greater than 30 mL/hr. Albumin levels reflect nutritional status, which may be poor in shock states due to an increased need for calories. Plasmanate expands blood volume by exerting increasing colloid osmotic pressure in the bloodstream, pulling fluid into the vascular space; this does not improve an abnormal hemoglobin.

The nurse prepares to administer digoxin (Lanoxin) 0.125 mg to an 82-year-old man admitted with influenza and a history of chronic heart failure. What should the nurse assess before giving the medication? A Prothrombin time B Urine specific gravity C Serum potassium level D Hemoglobin and hematocrit

C Serum potassium level Serum potassium should be monitored because hypokalemia increases the risk for digoxin toxicity. Changes in prothrombin time, urine specific gravity, and hemoglobin or hematocrit would not require holding the digoxin dose.

A postoperative client is admitted to the intensive care unit with hypovolemic shock. Which nursing action does the nurse delegate to an experienced nursing assistant? A. Obtain vital signs every 15 minutes. B. Measure hourly urine output. C. Check oxygen saturation. D. Assess level of alertness.

B. Measure hourly urine output. Monitoring hourly urine output is included in nursing assistant education and does not require special clinical judgment; the nurse evaluates the results. Obtaining vital signs, monitoring oxygen saturation, and assessing mental status in critically ill clients requires the clinical judgment of the critical care nurse because immediate intervention may be needed.

Which wound assessment characteristics suggest a superficial partial-thickness burn injury? A. Black-brown coloration B. Painful C. Moderate to severe edema D. Absence of blisters

B. Painful Characteristics of a superficial partial-thickness burn injury include pink to red coloration, mild to moderate edema, pain, and blisters. A black-brown coloration is more suggestive of full-thickness burn injury. Moderate to severe edema and absence of blisters may be present with deep partial-thickness to full-thickness burn injuries.

A nursing student is caring for a client with open-wound burns. Which nursing interventions does the nursing student provide for this client? (Select all that apply.) A. Provides cushions and rugs for comfort B. Performs frequent handwashing C. Places plants in the client's room D. Performs gloved dressing changes E. Uses disposable dishes

B. Performs frequent handwashing D. Performs gloved dressing changes E. Uses disposable dishes Handwashing is the most effective technique for preventing infection. Gloves should be worn when changing dressings to reduce the risk for infection. Equipment is not shared with other clients to prevent the risk for infection. Disposable items (e.g., pillows, dishes) are used as much as possible. Cushions and rugs are difficult to clean and may harbor organisms, and so are not provided. To avoid exposure to Pseudomonas, having plants or flowers in the room is prohibited.

The nurse is caring for a patient who has just returned after having left carotid artery angioplasty and stenting. Which assessment information is of most concern to the nurse? a. The pulse rate is 102 beats/min. b. The patient has difficulty speaking. c. The blood pressure is 144/86 mm Hg. d. There are fine crackles at the lung bases.

B. Small emboli can occur during carotid artery angioplasty and stenting, and the aphasia indicates a possible stroke during the procedure. Slightly elevated pulse rate and blood pressure are not unusual because of anxiety associated with the procedure. Fine crackles at the lung bases may indicate atelectasis caused by immobility during the procedure. The nurse should have the patient take some deep breaths.

A client has been admitted to the hospital with a diagnosis of suspected bacterial endocarditis. The complication the nurse will constantly observe for is: A. Presence of heart murmur B. Systemic Emboli C. Fever D. Congestive Heart Failure

B. Systemic Emboli

The home health nurse is caring for an 81-year-old who had a stroke 2 months ago. Based on information shown in the accompanying figure from the history, physical assessment, and physical therapy/occupational therapy, which nursing diagnosis is the highest priority for this patient? a. Impaired transfer ability b. Risk for caregiver role strain c. Ineffective health maintenance d. Risk for unstable blood glucose level

B. The spouse's household and patient care responsibilities, in combination with chronic illnesses, indicate a high risk for caregiver role strain. The nurse should further assess the situation and take appropriate actions. The data about the control of the patient's diabetes indicates that ineffective health maintenance and risk for unstable blood glucose are not priority concerns at this time. Because the patient is able to ambulate with a cane, the nursing diagnosis of impaired transfer ability is not supported.

Which information about the patient who has had a subarachnoid hemorrhage is most important to communicate to the health care provider? a. The patient complains of having a stiff neck. b. The patient's blood pressure (BP) is 90/50 mm Hg. c. The patient reports a severe and unrelenting headache. d. The cerebrospinal fluid (CSF) report shows red blood cells (RBCs).

B. To prevent cerebral vasospasm and maintain cerebral perfusion, blood pressure needs to be maintained at a level higher than 90 mm Hg systolic after a subarachnoid hemorrhage. A low BP or drop in BP indicates a need to administer fluids and/or vasopressors to increase the BP. An ongoing headache, RBCs in the CSF, and a stiff neck are all typical clinical manifestations of a subarachnoid hemorrhage and do not need to be rapidly communicated to the health care provider.

Several clients have been brought to the emergency department after an office building fire. Which client is at greatest risk for inhalation injury? A. Middle-aged adult who is frantically explaining to the nurse what happened B. Young adult who suffered burn injuries in a closed space C. Adult with burns to the extremities D. Older adult with thick, tan-colored sputum

B. Young adult who suffered burn injuries in a closed space The client who suffered burn injuries in a closed space is at greatest risk for inhalation injury because the client breathed a greater concentration of confined smoke. Clients who experienced a fire typically have some type of respiratory distress. However, the client talking without difficulty demonstrates minimal respiratory distress. Extensive burns to the hands and face, not the extremities, would be a greater risk. Sputum would be carbonaceous, not tan, if the client had suffered inhalation injury.

A diagnosis of a ruptured cerebral aneurysm has been made in a patient with manifestations of a stroke. The nurse anticipates that treatment options that would be evaluated for the patient include a. hyperventilation therapy b. surgical clipping of the aneurysm c. administration of hyperosmotic agents d. administration of thrombolytic therapy

B: Surgical clipping of they aneurysm- Surgical management with clipping of an aneurysm to decrease re bleeding and vasospasm is an option for a stroke cause by rupture of a cerebral aneurysm. Placement of coils into the lumens of the aneurysm by intercentional radiologists is increasing in popularity. Hyperventilation therapy would increase vasodilation and the potential for hemorrhage. Thrombolytic therapy would be absolutely contraindicated, and if a vessel is patent, osmotic diuretics may leak into tissue, pulling fluid out of the vessel and increasing edema.

To promote communication during rehabilitation of the patient with aphasia, an appropriate nursing intervention is to a. use gestures, pictures, and music to stimulate patient responses b. talk about activities of daily living (ADLs) that are familiar to the patient c. structure statements so that patient does not have to respond verbally d. use flashcards with simple words and pictures to promote language recall

B: Talk about ADLs that are familiar to the patient- during rehabilitation, the patient with aphasia needs frequent, meaningful verbal stimulation that has relevance for him. Conversation by the nurse and family should address ADLs that are familiar to the patient. Gestures, pictures, and simple statements are more appropriate in the acute phase, when patients may be overwhelmed with verbal stimuli. Flashcards are often perceived by the patient as childish and meaningless.

S3 and S4 are low pitched sounds best auscultated with the ____________ of the stethoscope over the apex of the heart.

Bell

Left ventricular thrombus

Blood pooling in LV Decreased ejection fraction -complication of HF

The ____________ and ____________ pulses should be palpated for rate, rhythm, and amplitude.

Brachial and radial

In the cardiac assessment, ____________ should be even, regular, and unlabored.

Breathing

When auscultating the carotid pulse, you hear a whooshing sound. This is a ____________, and indicates an obstruction such as narrowing of the vessel due to cholesterol, or an increased cardiac output as seen with hyperthyroidism.

Bruit

The ____________ is directly after the AV node, and here the impulse is slowed 0.1 seconds before passing onto the bundle branches.

Bundle of his

A patient with hypovolemic shock has a urinary output of 15 ml/hr. The nurse understands that the compensatory physiologic mechanism that leads to altered urinary output is a. activation of the sympathetic nervous system (SNS), causing vasodilation of the renal arteries. b. stimulation of cardiac -adrenergic receptors, leading to increased cardiac output. c. release of aldosterone and antidiuretic hormone (ADH), which cause sodium and water retention. d. movement of interstitial fluid to the intravascular space, increasing renal blood flow.

C

Fluid and electrolyte shifts that occur during the early emergent phase include: A. adherence of albumin to vascular walls B. movement of potassium into the vascular space C. sequestering of sodium and water into interstitial fluid D. hemolysis of red blood cells from the large volumes of rapidly administered fluid

C

Knowing the most common causes of household fires, which of the following prevention strategies would the nurse focus on when teaching about fire safety? A. set hot water temperature at !$)*F (60*C) B. use only hardwired smoke detectors C. encourage regular home fire exit drills D. never permit older adults to cook unattended

C

The triage nurse receives a call from a community member who is driving an unconscious friend with multiple injuries after a motorcycle accident to the hospital. The caller states that they will be arriving in 1 minute. In preparation for the patient's arrival, the nurse will obtain a. a liter of lactated Ringer's solution. b. 500 ml of 5% albumin. c. two 14-gauge IV catheters. d. a retention catheter.

C

When caring for a patient with cardiogenic shock and possible MODS, which information obtained by the nurse will help confirm the diagnosis of MODS? a. The patient has crackles throughout both lung fields. b. The patient complains of 8/10 crushing chest pain. c. The patient has an elevated ammonia level and confusion. d. The patient has cool extremities and weak pedal pulses.

C

When the nurse is caring for a patient in cardiogenic shock who is receiving dobutamine (Dobutrex) and nitroglycerin (Tridil) infusions, the best evidence that the medications are effective is that the a. systolic BP increases to greater than 100 mm Hg. b. cardiac monitor shows sinus rhythm at 96 beats/min. c. PAWP drops to normal range. d. troponin and creatine kinase levels decrease.

C

The ____________ should be inspected for clubbing, and color abnormalities.

Fingernails

A 68-year-old man with suspected bacterial meningitis has just had a lumbar puncture in which cerebrospinal fluid was obtained for culture. Which medication should the nurse administer first? A. Codeine B. Phenytoin (Dilantin) C. Ceftriaxone (Rocephin) D. Acetaminophen (Tylenol)

C Bacterial meningitis is a medical emergency. When meningitis is suspected, antibiotic therapy (e.g., ceftriaxone) is instituted immediately after the collection of specimens for cultures, and even before the diagnosis is confirmed. Dexamethasone may also be prescribed before or with the first dose of antibiotics. The nurse should collaborate with the health care provider to manage the headache (with codeine), fever (with acetaminophen), and seizures (with phenytoin).

Which of the following laboratory findings fits with a diagnosis of cardiogenic shock? A) Decreased liver enzymes B) Increased white blood cells C) Increased blood urea nitrogen and creatinine levels D) Decreased red blood cells, hemoglobin, and hematocrit

C The renal hypoperfusion that accompanies cardiogenic shock results in increased BUN and creatinine levels. Impaired perfusion of the liver results in increased liver enzymes while red blood cell indices are typically normal because of relative hypovolemia. White blood cell levels do not typically rise in cardiogenic shock.

A 54-year-old male patient who had bladder surgery 2 days ago develops acute decompensated heart failure (ADHF) with severe dyspnea. Which action by the nurse would be indicated first? A Perform a bladder scan to assess for urinary retention. B Restrict the patient's oral fluid intake to 500 mL per day. C Assist the patient to a sitting position with arms on the overbed table. D Instruct the patient to use pursed-lip breathing until the dyspnea subsides.

C Assist the patient to a sitting position with arms on the overbed table. The nurse should place the patient with ADHF in a high Fowler's position with the feet horizontal in the bed or dangling at the bedside. This position helps decrease venous return because of the pooling of blood in the extremities. This position also increases the thoracic capacity, allowing for improved ventilation. Pursed-lip breathing helps with obstructive air trapping but not with acute pulmonary edema. Restricting fluids takes considerable time to have an effect.

What is the priority assessment by the nurse caring for a patient receiving IV nesiritide (Natrecor) to treat heart failure? A Urine output B Lung sounds C Blood pressure D Respiratory rate

C Blood pressure Although all identified assessments are appropriate for a patient receiving IV nesiritide, the priority assessment would be monitoring for hypotension, the main adverse effect of nesiritide.

A 63-year-old patient who began experiencing right arm and leg weakness is admitted to the emergency department. In which order will the nurse implement these actions included in the stroke protocol? a. Obtain computed tomography (CT) scan without contrast. b. Infuse tissue plasminogen activator (tPA). c. Administer oxygen to keep O2 saturation >95%. d. Use National Institute of Health Stroke Scale to assess patient.

C, D, A, B The initial actions should be those that help with airway, breathing, and circulation. Baseline neurologic assessments should be done next. A CT scan will be needed to rule out hemorrhagic stroke before tPA can be administered.

The nurse is caring for a client with burns. Which question does the nurse ask the client and family to assess their coping strategies? A. "Do you support each other?" B. "How do you plan to manage this situation?" C. "How have you handled similar situations before?" D. "Would you like to see a counselor?"

C. "How have you handled similar situations before?" Asking how the client and family have handled similar situations in the past assesses whether the client's and the family's coping strategies may be effective. "Yes-or-no" questions such as "Do you support each other?" are not very effective in extrapolating helpful information. The client and family in this situation probably are overwhelmed and may not know how they will manage; asking them how they plan to manage the situation does not assess coping strategies. Asking the client and the family if they would like to see a counselor also does not assess their coping strategies.

A client who was the sole survivor of a house fire says, "I feel so guilty. Why did I survive?" What is the best response by the nurse? A. "Do you want to pray about it?" B. "I know, and you will have to learn to adapt to a new body image." C. "Tell me more." D. "There must be a reason."

C. "Tell me more." Asking the client to tell the nurse more encourages therapeutic grieving. Offering to pray with the client assumes that prayer is important to the client and does not allow for grieving; the nurse should never assume that the client is religious. The response, "I know, and you will have to learn to adapt to a new body image" only serves to add stress to the client's situation. The response, "There must be a reason" minimizes the grieving process by not allowing the client to express his or her concerns.

A 68-year-old patient is being admitted with a possible stroke. Which information from the assessment indicates that the nurse should consult with the health care provider before giving the prescribed aspirin? a. The patient has dysphasia. b. The patient has atrial fibrillation. c. The patient reports that symptoms began with a severe headache. d. The patient has a history of brief episodes of right-sided hemiplegia.

C. A sudden onset headache is typical of a subarachnoid hemorrhage, and aspirin is contraindicated. Atrial fibrillation, dysphasia, and transient ischemic attack (TIA) are not contraindications to aspirin use, so the nurse can administer the aspirin.

The nurse is caring for a patient who has been experiencing stroke symptoms for 60 minutes. Which action can the nurse delegate to a licensed practical/vocational nurse (LPN/LVN)? a. Assess the patient's gag and cough reflexes. b. Determine when the stroke symptoms began. c. Administer the prescribed short-acting insulin. d. Infuse the prescribed IV metoprolol (Lopressor).

C. Administration of subcutaneous medications is included in LPN/LVN education and scope of practice. The other actions require more education and scope of practice and should be done by the registered nurse (RN).

____________ results from inflammation of the pericardial sac, and is heard best when the patient sits forward.

Friction rub

Ultrafiltration (aquapheresis)

HF pt with fluid overload, resistant to diuretics -get the pt out of crisis

A 72-year-old patient who has a history of a transient ischemic attack (TIA) has an order for aspirin 160 mg daily. When the nurse is administering medications, the patient says, "I don't need the aspirin today. I don't have a fever." Which action should the nurse take? a. Document that the aspirin was refused by the patient. b. Tell the patient that the aspirin is used to prevent a fever. c. Explain that the aspirin is ordered to decrease stroke risk. d. Call the health care provider to clarify the medication order.

C. Aspirin is ordered to prevent stroke in patients who have experienced TIAs. Documentation of the patient's refusal to take the medication is an inadequate response by the nurse. There is no need to clarify the order with the health care provider. The aspirin is not ordered to prevent aches and pains.

The nurse identifies the nursing diagnosis of imbalanced nutrition: less than body requirements related to impaired self-feeding ability for a left-handed patient with left-sided hemiplegia. Which intervention should be included in the plan of care? a. Provide a wide variety of food choices. b. Provide oral care before and after meals. c. Assist the patient to eat with the right hand. d. Teach the patient the "chin-tuck" technique.

C. Because the nursing diagnosis indicates that the patient's imbalanced nutrition is related to the left-sided hemiplegia, the appropriate interventions will focus on teaching the patient to use the right hand for self-feeding. The other interventions are appropriate for patients with other etiologies for the imbalanced nutrition.

The nurse is caring for a client in the refractory stage of cardiogenic shock. Which intervention does the nurse consider? A. Admission to rehabilitation hospital for ambulatory retraining B. Collaboration with home care agency for return to home C. Discussion with family and provider regarding palliative care D. Enrollment in a cardiac transplantation program

C. Discussion with family and provider regarding palliative care In this irreversible phase, therapy is not effective in saving the client's life, even if the cause of shock is corrected and mean arterial pressure temporarily returns to normal. A discussion on palliative care should be considered. Rehabilitation or returning home is unlikely. The client with sustained tissue hypoxia is not a candidate for organ transplantation.

The nurse is caring for a client in the refractory stage of cardiogenic shock. Which intervention should the nurse consider? A. Admission to rehab hospital for ambulatory retraining B. Collaboration with home care agency for return to home. C. Discussion with family and provider regarding palliative care. D. Enrollment in a cardiac transplantation program.

C. Discussion with family and provider regarding palliative care. (In this irreversible phase, therapy is not effective in saving the client's life, even if the cause of shock is corrected and mean MAP temporarily returns to normal. A discussion on palliative care should be considered.)

Which strategy does the nurse include when teaching a college student about fire prevention in the dormitory room? A. Use space heaters to reduce electrical costs. B. Check water temperature before bathing. C. Do not smoke in bed. D. Wear sunscreen.

C. Do not smoke in bed. Smoking in bed increases the risk for fire because the person could fall asleep. Use of space heaters may increase the risk for fire, especially if they are knocked over and left unattended. Checking water temperature does not prevent fires, but it should be checked if the client has reduced sensation in the hands or feet. Sunscreen is advised to prevent sunburn.

When caring for a patient with a new right-sided homonymous hemianopsia resulting from a stroke, which intervention should the nurse include in the plan of care? a. Apply an eye patch to the right eye. b. Approach the patient from the right side. c. Place objects needed on the patient's left side. d. Teach the patient that the left visual deficit will resolve.

C. During the acute period, the nurse should place objects on the patient's unaffected side. Because there is a visual defect in the right half of each eye, an eye patch is not appropriate. The patient should be approached from the left side. The visual deficit may not resolve, although the patient can learn to compensate for the defect.

A client with burn injuries is being admitted. Which priority does the nurse anticipate within the first 24 hours? A. Range-of-motion exercises B. Emotional support C. Fluid resuscitation D. Sterile dressing changes

C. Fluid resuscitation The client will require fluid resuscitation because fluid does not stay in the vessels after a burn injury. Range-of-motion exercise is not the priority for this client. Although emotional support and sterile dressing changes are important, they are not the priority during the resuscitation phase of burn injury.

Which stroke risk factor for a 48-year-old male patient in the clinic is most important for the nurse to address? a. The patient is 25 pounds above the ideal weight. b. The patient drinks a glass of red wine with dinner daily. c. The patient's usual blood pressure (BP) is 170/94 mm Hg. d. The patient works at a desk and relaxes by watching television.

C. Hypertension is the single most important modifiable risk factor. People who drink more than 1 (for women) or 2 (for men) alcoholic beverages a day may increase risk for hypertension. Physical inactivity and obesity contribute to stroke risk but not as much as hypertension.

Another term for Diastolic HF

HF with normal EF

Which problem places a client at highest risk for sepsis? A. Pernicious anemia B. Pericarditis C. Post kidney transplant D. Client owns an iguana

C. Post kidney transplant The post-kidney transplant client will need to take lifelong immune suppressant therapy and is at risk for infection from internal and external organisms. Pernicious anemia is related to lack of vitamin B12, not to bone marrow failure (aplastic anemia), which would place the client at risk for infection. Inflammation of the pericardial sac is an inflammatory condition that does not pose a risk for septic shock. Although owning pets, especially cats and reptiles, poses a risk for infection, the immune-suppressed kidney transplant client has a very high risk for infection, sepsis, and death.

The nurse is caring for a group of clients at risk for sepsis. Which problem places the client at highest risk? A. Pernicious anemia B. Pericarditis C. Post-kidney transplant D. Client owns an iguana

C. Post-kidney transplant This client will need to take lifelong immune suppressant therapy and is at risk for infection from internal and external organisms.

In assessing a client in the rehabilitative phase of burn therapy, which priority problem does the nurse anticipate? A. Intense pain B. Potential for inadequate oxygenation C. Reduced self-image D. Potential for infection

C. Reduced self-image In the rehabilitative phase of burn therapy, the client is discharged and his or her life is not the same. A priority problem of reduced self-image is expected. Intense pain and potential for inadequate oxygenation are relevant in the resuscitation phase of burn injury. Potential for infection is relevant in the acute phase of burn injury.

The nurse is planning psychosocial support for the patient and family of the patient who suffered a stroke. What factor will most likely have the greatest impact on positive family coping with the situation? A. Specific patient neurologic deficits B. The patient's ability to communicate C. Rehabilitation potential of the patient D. Presence of complications of a stroke

C. Rehabilitation potential of the patient Although a patient's neurologic deficit might initially be severe after a stroke, the ability of the patient to recover is most likely to positively impact the family's coping with the situation. Providing explanations and emotional support beginning in the acute phase through the rehabilitation phase will facilitate coping. Emphasizing successes will offer the most realistic hope for the patient's rehabilitation and helps maintain hope for the patient's future abilities.

Which clinical manifestation is indicative of wound healing for a client in the acute phase of burn injury? A. Pale, boggy, dry, or crusted granulation tissue B. Increasing wound drainage C. Scar tissue formation D. Sloughing of grafts

C. Scar tissue formation Indicators of wound healing include the presence of granulation, re-epithelialization, and scar tissue formation. Pale, boggy, dry, or crusted granulation tissue is indicative of infection, as are increasing wound drainage and sloughing of grafts.

Which assessment is the nurse's highest priority in caring for a client in the acute phase of burn injury? A. Bowel sounds B. Muscle strength C. Signs of infection D. Urine output

C. Signs of infection The client with burn injury is at risk for infection as a result of open wounds and reduced immune function. Burn wound sepsis is a serious complication of burn injury, and infection is the leading cause of death during the acute phase of recovery. Assessing bowel sounds, assessing muscle strength, and assessing urine output are important but not the priority during the acute phase of burn injury.

Which sensory-perceptual deficit is associated with left-sided stroke (right hemiplegia)? A. Overestimation of physical abilities. B. Difficulty judging position and distance. C. Slow and possibly fearful performance of tasks. D. Impulsivity and impatience at performing tasks.

C. Slow and possibly fearful performance of tasks. Patients with a left-sided stroke (right hemiplegia) commonly are slower in organization and performance of tasks and may have a fearful, anxious response to a stroke. Overconfidence, spatial disorientation, and impulsivity are more commonly associated with a right-sided stroke.

What typical sign/symptom indicates the early stage of septic shock? A. Pallor and cool skin B. Blood pressure 84/50 mm Hg C. Tachypnea and tachycardia D. Respiratory acidosis

C. Tachypnea and tachycardia Signs of systemic inflammatory response syndrome, which precedes sepsis, include rapid respiratory rate, leukocytosis, and tachycardia. In the early stage of septic shock, the client is usually warm and febrile. Hypotension does not develop until later in septic shock due to compensatory mechanisms. Respiratory alkalosis occurs early in shock because of an increased respiratory rate.

How does the nurse recognize that the client is in the early stage of septic shock? A. Pallor and cool skin B. B/P 84/50 C. Tachypnea and tachycardia D. Respiratory acidosis

C. Tachypnea and tachycardia (Signs of systemic inflammatory response syndrome, which precede sepsis, include rapid respiratory rate, leukocytosis, and tachycardia,.)

A male patient who has right-sided weakness after a stroke is making progress in learning to use the left hand for feeding and other activities. The nurse observes that when the patient's wife is visiting, she feeds and dresses him. Which nursing diagnosis is most appropriate for the patient? a. Interrupted family processes related to effects of illness of a family member b. Situational low self-esteem related to increasing dependence on spouse for care c. Disabled family coping related to inadequate understanding by patient's spouse d. Impaired nutrition: less than body requirements related to hemiplegia and aphasia

C. The information supports the diagnosis of disabled family coping because the wife does not understand the rehabilitation program. There are no data supporting low self-esteem, and the patient is attempting independence. The data do not support an interruption in family processes because this may be a typical pattern for the couple. There is no indication that the patient has impaired nutrition.

A 70-year-old female patient with left-sided hemiparesis arrives by ambulance to the emergency department. Which action should the nurse take first? a. Monitor the blood pressure. b. Send the patient for a computed tomography (CT) scan. c. Check the respiratory rate and effort. d. Assess the Glasgow Coma Scale score.

C. The initial nursing action should be to assess the airway and take any needed actions to ensure a patent airway. The other activities should take place quickly after the ABCs (airway, breathing, and circulation) are completed.

A 47-year-old patient will attempt oral feedings for the first time since having a stroke. The nurse should assess the gag reflex and then a. order a varied pureed diet. b. assess the patient's appetite. c. assist the patient into a chair. d. offer the patient a sip of juice.

C. The patient should be as upright as possible before attempting feeding to make swallowing easier and decrease aspiration risk. To assess swallowing ability, the nurse should initially offer water or ice to the patient. Pureed diets are not recommended because the texture is too smooth. The patient may have a poor appetite, but the oral feeding should be attempted regardless.

After a patient experienced a brief episode of tinnitus, diplopia, and dysarthria with no residual effects, the nurse anticipates teaching the patient about a. cerebral aneurysm clipping. b. heparin intravenous infusion. c. oral low-dose aspirin therapy. d. tissue plasminogen activator (tPA).

C. The patient's symptoms are consistent with transient ischemic attack (TIA), and drugs that inhibit platelet aggregation are prescribed after a TIA to prevent stroke. Continuous heparin infusion is not routinely used after TIA or with acute ischemic stroke. The patient's symptoms are not consistent with a cerebral aneurysm. tPA is used only for acute ischemic stroke, not for TIA.

Which assessment information about a 60-kg client admitted 12 hours ago with a full-thickness burn over 30% of the total body surface area is of greatest concern to the nurse? A. Bowel sounds are absent. B. The pulse oximetry level is 91%. C. The serum potassium level is 6.1 mEq/L. D. Urine output since admission is 370 mL.

C. The serum potassium level is 6.1 mEq/L. An elevated serum potassium level can cause cardiac dysrhythmias and arrest, and so is of the most concern. Absence of bowel sounds, a pulse oximetry level of 91%, and urine output of 370 mL since admission are normal findings during the resuscitation phase of burn injury.

The nurse is evaluating the effectiveness of fluid resuscitation for a client in the resuscitation phase of burn injury. Which finding does the nurse correlate with clinical improvement? A. Blood urea nitrogen (BUN), 36 mg/dL B. Creatinine, 2.8 mg/dL C. Urine output, 40 mL/hr D. Urine specific gravity, 1.042

C. Urine output, 40 mL/hr Fluid resuscitation is provided at the rate needed to maintain urine output at 30 to 50 mL/hr or 0.5 mL/kg/hr. A BUN of 36 mg/dL is above normal, a creatinine of 2.8 mg/dL is above normal, and a urine specific gravity of 1.042 is above normal.

During the change of shift report a nurse is told that a patient has an occluded left posterior cerebral artery. The nurse will anticipate that the patient may have a. dysphasia. b. confusion. c. visual deficits. d. poor judgment.

C. Visual disturbances are expected with posterior cerebral artery occlusion. Aphasia occurs with middle cerebral artery involvement. Cognitive deficits and changes in judgment are more typical of anterior cerebral artery occlusion.

The nurse on the clinical unit is assigned to four patients. Which patient should she assess first? a. patient with a skull fracture whose nose is bleeding b. elderly patient with a stroke who is confused and whose daughter is present c. patient with meningitis who is suddenly agitated and reporting a HA of 10 on a 0 to 10 scale d. patient who had a craniotomy for a brain tumor who is now 3 days postoperative and has had continued emesis

C. patient with meningitis who is suddenly agitated and reporting a HA of 10 on a 0 to 10 scale

Which intervention should the nurse delegate to the LPN when caring for a patient following an acute stroke? a. assess the patient's neurologic status b. assess the patient's gag reflex before beginning feeding c. administer ordered antihypertensives and platelet inhibitors d. teach the patient's caregivers strategies to minimize unilateral neglect

C: Administer ordered antihypertensives and platelet inhibitors- medication administration is within the scope of practice for an LPN. Assessment and teaching are within the scope of practice for the RN.

A thrombus that develops in a cerebral artery does not always cause a loss of neurologic function because a. the body can dissolve the atherosclerotic plaques as they form b. some tissues of the brain do not require constant blood supply to prevent damage c. circulation through the circle of Willis may provide blood supply to the affected area of the brain d. neurologic deficits occur only when major arteries are occluded by thrombus formation around an atherosclerotic plaque

C: Circulation through the circle of Willis may provide blood supply to the affected area of the brain. The communication between cerebral arteries in the circle of Willing provides a collateral circulation, which may maintain circulation to an area of the brain if its original blood supply is obstructed. ALl areas of the brain require constant blood supply, and atherosclerotic plaques are not readily reversed. Neurologic deficits can result from ischemia cause by many factors.

The incidence of ischemic stroke in patients with TIAs and other risk factors is reduced with administration of a. furosemide (Lasix) b. lovastatin (Mevacor) c. daily low dose aspirin d. nimodipine (Nimotop)

C: Daily low dose aspirin- the administration of antiplatelet agents, such as aspirin, dipyridamole (Persantine), and ticlopdipine (Ticlid), reduces the incidence of stroke in those at risk. Anticoagulants are also used for prevention of embolic strokes but increase the risk for hemorrhage. Diuretics are not indicated for stroke prevention other than for their role in controlling BP, and antilipemic agents have bot been found to have a significant effect on stroke prevention. The calcium channel blocker nimodipine is used in patients with subarachnoid hemorrhage to decrease the effects of vasospasm and minimize tissue damage. P.S. I freaking love you and good luck on the final!!

According to the NYHA functional classification of heart disease, the patient belongs to . In . In

Class 1, ordinary physical activity does not cause fatigue or dyspnea, and there is no limitation of physical activity Class 2, where there are no symptoms at rest. The patient can perform daily routine activities but tires easily and experiences palpitations and dyspnea. In Class 3, there is inability to perform daily chores like dressing oneself. The patient may be comfortable at rest Class 4, the patient has symptoms even at rest and is unable to carry out any activity without discomfort.

A patient with a stroke has a right sided hemiplegia. The nurse prepares family members to help control behavior changes seen with this type of stroke by teaching them to a. ignore undesirable behaviors manifested by the patient b. provide directions to the patient verbally in small steps c. distract the patient from inappropriate emotional responses d. supervise all activities before allowing the patient to pursue them independently

C: Distract the patient from inappropriate emotional responses- patients with left-sided brain damage from stroke often experience emotional lability, inappropriate emotional responses, mood swings, and uncontrolled tears or laughter disproportionate or out of context with the situation. The behavior is upsetting and embarrassing to both the patient and the family, and the patient should be distracted to minimize its presence. Patients with right-brain damage often have impulsive, rapid behavior that supervision and direction.

In promoting health maintenance for prevention of strokes, the nurse understands that the highest risk for the most common type of stroke is present in a. African Americans b. women who smoke c.individuals with hypertension and diabetes d. those who are obese with high dietary fat intake

C: Individuals with hypertension and diabetes- The highest risk factors for thrombotic stroke are hypertension and diabetes. African Americans have a higher risk for stroke than do white persons but probably because they have a greater incidence of hypertension. Factors such as obesity, diet high in saturated fats and cholesterol, cigarette smoking, and excessive alcohol use are also risk factors but carry less risk than hypertension.

A carotid endarterectomy is being considered as a treatment for a patient who has had several TIAs. The nurse explains to the patient that this surgery a. is used to restore blood to the brain following an obstruction of a cerebral artery b. involves intracranial surgery to join a superficial extracranial artery to an intracranial artery c. involves removing an atherosclerotic plaque in the carotid artery to prevent an impending stroke d. is sued to open a stenosis in a carotid artery with a balloon and stent to restore cerebral circulation

C: Involves removing an atherosclerotic plaque in the carotid artery to prevent an impending stroke- An endarterectomy is a removal of an atherosclerotic plaque, and plaque in the carotid artery may impair circulation enough to cause a stroke. A carotid endarterectomy is performed to prevent a cerebrovascular accident (CVA), as are most other surgical procedures. An extacranial-intracranial bypass involves cranial surgery to bypass a sclerotic intacranial artery. Percutaneous transluminal angioplasty uses a balloon to compress stenotic areas in the carotid and vertebrobasilar arteries and often includes inserting a stent to hold the artery open.

A patient with right hemisphere stroke has a nursing diagnosis of unilateral neglect related to sensory perceptual deficits. During the patient's rehabilitation, it is important for the nurse to a. avoid positioning the patient on the affected side b. place all objects for care on the patient's unaffected side c. teach the patient to care consciously for the affected side d. protect the affected side from injury with pillows and supports

C: Teach the patient to care consciously for the affected side- unilateral neglect, or neglect syndrome, occurs when the patient with a stroke is unaware of the affected side of the body, which puts the patient at risk for injury. During the acute phase, the affected side is cared for by the nurse with positioning and support, during rehabilitation the patient is taught to care consciously for and attend to the affected side of the body to protect it from injury. Patients may be positioned on the affected side for up to 30 minutes.

The neurologic functions that are affected by a stroke are primarily related to a. the amount of tissue area involved b. the rapidity of onset of symptoms c. the brain area perfused by the affected artery d. the presence or absence of collateral circulation

C: The brain area perfused by the affected artery- clinical manifestation of altered neurologic function differ, depending primarily on the specific cerebral artery involved and the area of the brain that is perfused by the artery. The degree of impairment depends on rapidity of onset, the size of the lesion, and the presence of collateral circulation.

An appropriate food for a patient with a stroke who has mild dysphagia is a. fruit juices b. pureed meat c. scrambled eggs d. fortified milkshakes

C: scrambled eggs- soft foods that provide enough texture, flavor, and bulk to stimulate swallowing should be used for the patient with dysphasia. Thin liquids are difficult to swallow, and patients may not be able to control them in the mouth. Pureed foods are often too bland and to smooth, and milk products should be avoided because they tend to increase the viscosity of mucus and increase salivation.

Dilated Cardiomyopathy

Can cause mixed systolic and diastolic HF -poor systolic function worsened by dilated ventricle walls that are unable to relax -extremely low EF -high pulmonary pressure -biventricular failure

CI

Cardiac Index 2 - 4 L/min (CO adjusted to body size)

CO

Cardiac Output 4- 8 L/min

You should never palpate both sides of the ____________ at the same time because it can stimulate the baroreceptors and cause a drop in blood pressure or pulse.

Carotid pulse

Anemia (HF)

Caused by poor nutrition, renal disease or drug therapy

CVP

Central Venous Pressure 0 - 8 ICU = 8 - 12 measures volume

CerPP

Cerebral Perfusion 70 - 90 mmHg (MAP - ICP)

52 year old patient who just spent time visiting her family in Brazil has come to your office complaining of chest pain... What are you thinking based off her ethnicity?

Chaga's disease trypanosoma cruzi

What are the symptoms of a Myocardial Infarction?

Chest pain Chest tightness

27. A patient with extensive electrical burn injuries is admitted to the emergency department. Which of these health care provider orders should the nurse implement first? a. Place on cardiac monitor. b. Start 2 large bore IVs. c. Assess for pain at contact points. d. Apply dressings to burned areas.

Correct Answer: A Rationale: After an electrical burn, the patient is at risk for fatal dysrhythmias and should be placed on a cardiac monitor. The other actions should be accomplished in the following order: Start 2 IVs, assess for pain, and apply dressings.

15. Which of these patients is most appropriate for the burn unit charge nurse to assign to an RN staff nurse who has floated from the hospital medical unit? a. A 34-year-old patient who has a weight loss of 15% from admission and requires enteral feedings and parenteral nutrition (PN) b. A 45-year-old patient who has just come back to the unit after having a cultured epithelial autograft to the chest c. A 60-year-old patient who has twice-daily burn débridements and dressing changes to partial-thickness facial burns d. A 63-year-old patient who has blebs under an autograft on the thigh and has an order for bleb aspiration

Correct Answer: A Rationale: An RN from a medical unit would be familiar with malnutrition and with administration and evaluation of response to enteral feedings and PN. The other patients require burn assessment and care that is more appropriate for staff who regularly care for burned patients.

7. During the early emergent phase of burns, the nurse will anticipate giving opioid analgesics by the IV route so that a. the medications will be rapidly effective. b. less frequent administration is needed. c. larger doses of medications can be given. d. respiratory depression can be easily treated.

Correct Answer: A Rationale: Because medications administered by the oral or IM routes will not be rapidly effective, the nurse should administer medications IV. IV medications are rapidly metabolized, and frequent administration may be necessary. The dosage is not determined by the route, but by its effectiveness. The ease with which respiratory depression can be corrected is not a factor in choosing the route for medication administration in a burn patient.

23. Two weeks after admission for major burns, a patient is continuing to lose weight despite a high-carbohydrate, high-protein diet. The nurse identifies the nursing diagnosis of imbalanced nutrition: less than body requirements. In planning nursing interventions, the best between-meal snack for the patient would be a. eggnog. b. bagel. c. nuts. d. crackers and cheese.

Correct Answer: A Rationale: Eggnog has the highest protein, calorie, and carbohydrates of the choices. Because the patient with a major burn is likely to be anorexic, it is important that the patient's foods be nutrient dense.

22. A patient in the acute phase of burn injury requires frequent hydrotherapy sessions for wound débridement. To evaluate for complications of hydrotherapy, the nurse will plan to closely monitor a. serum sodium level. b. lung sounds. c. pulse quality. d. daily urine output.

Correct Answer: A Rationale: Hydrotherapy leads to loss of sodium from open burn areas into the bath water, which is hypotonic. Lung sounds, pulse quality, and urine output are not directly affected by hydrotherapy, although these assessments are also part of patient care.

13. A patient with burns of the upper thorax and circumferential burns of both arms develops decreased radial pulses and loss of sensation in the fingers. The most appropriate action by the nurse is to a. notify the health care provider. b. increase the IV rate and re-evaluate (ROM) on the patient's arms. c. elevate the arms on pillows.

Correct Answer: A Rationale: The decrease in pulses in a patient with circumferential burns indicates decreased circulation to the arms and the need for escharotomy. Increasing the IV rate may increase the blood flow to the arms slightly, but it will not address the cause of the decreased circulation. Passive ROM will not improve circulation to the arms. Elevating the arms on pillows will decrease circulation.

17. A 21-year-old patient who has deep partial-thickness facial and neck burns has a nursing diagnosis of disturbed body image. The nurse evaluates that patient outcomes for this nursing diagnosis are met when the patient a. starts to use make-up to cover up the scars. b. expresses concern about the scar appearance. c. realizes that scarring is temporary. d. avoids using a pillow under the head.

Correct Answer: A Rationale: The willingness to use strategies to enhance appearance is an indication that the problem has resolved. Expressing concern about the scars indicates a willingness to discuss the scars but does not indicate that the disturbed body image is resolved. Scarring from deep partial-thickness burns is permanent, although some improvement in scar appearance may occur. Avoiding using a pillow will help prevent contractures, but it does not address the problem of disturbed body image.

26. Which action should the nurse take first when caring for a patient who has just arrived in the emergency department with facial and chest burns caused by a house fire? a. Ringer's solution b. Lung sounds c. Size and depth d. Ordered opioid

Correct Answer: B Rationale: A patient with facial and chest burns is at risk for inhalation injury, and assessment of airway and breathing is the priority. The other actions will be completed after airway management is assured.

25. Which of these nursing actions should be accomplished first for a patient who has suffered a burn injury while working on an electrical power line? a. Obtain the patient's vital signs. b. Place a cervical collar on the patient. c. Assess for the contact points. d. Place on a cardiac monitor.

Correct Answer: B Rationale: Cervical spine injuries are commonly associated with electrical burns; therefore, stabilization of the cervical spine takes precedence after airway management. The other actions are also included in the emergent care after electrical burns, but the most important action is to avoid spinal cord injury.

3. When assessing an emergency department patient who spilled hot oil from a deep-fat fryer on the right leg and foot, the nurse notes that the leg and foot are red, swollen, and covered with large blisters. The patient states that they are very painful. The nurse will document the injury as a. full-thickness skin destruction. b. deep partial-thickness skin destruction. c. superficial partial-thickness skin destruction. d. deep full-thickness skin destruction.

Correct Answer: B Rationale: The erythema, swelling, and blisters point to a deep partial-thickness burn. With full-thickness skin destruction, the appearance is pale and dry or leathery and the area is painless because of the associated nerve destruction. With superficial partial thickness burns, the area is red, but no blisters are present.

What structure or part of the heart does endocarditis effect the most?

Heart Valves

18. A patient who was found unconscious in a burning bedroom and has burns to the lower legs is assessed by the nurse in the emergency department. The nurse notes that the patient's face is bright red. Which of these actions should the nurse take first? a. Elevate the legs on pillows. b. Place the patient on 100% O2 using a non-breather mask. c. Assess for singed nasal hair and dark oral mucous membranes. d. Insert 2 large-bore IV lines.

Correct Answer: B Rationale: The patient's history and skin color suggest carbon monoxide poisoning, which should be treated by rapidly starting oxygen at 100%. The other actions can be taken after the actions to correct gas exchange.

11. A patient with deep partial-thickness and full-thickness burns of the face and chest has the wounds treated with the open method. The nurse identifies an expected patient outcome of absence of wound infections. An appropriate nursing action to help the patient meet the outcome is to a. restrict all visitors prevent cross-contamination of wounds. b. wear gowns, caps, masks, and gloves during all care of the patient. c. use sterile water for cleansing and debridement in the hydrotherapy tank. d. administer prophylactic antibiotics to prevent bacterial colonization of wounds.

Correct Answer: B Rationale: Use of gowns, caps, masks, and gloves during all patient care will decrease the possibility of wound contamination for a patient whose burns are not covered. Restricting visitors is not necessary and will have adverse psychosocial consequences for the patient. Tap water is used during hydrotherapy, and hydrotherapy tanks are not usually used in burn care because of the risk of cross-contamination. Systemic antibiotics are not well absorbed into deep burns because of the lack of circulation.

1. An employee spilled industrial acids on the arms and legs at work. Before transporting the individual to the hospital, the occupational nurse at the facility should a. cover the affected area with dry, sterile dressings. b. flush the burned area with large amounts of tap water. c. place cool compresses on the area of exposure. d. apply an alkaline solution to the affected area.

Correct Answer: B Rationale: With chemical burns, the initial action is to remove the chemical from contact with the skin as quickly as possible. Covering the affected area or placing cool compresses on the area will leave the chemical in contact with the skin. Application of an alkaline solution is not recommended.

28. A patient who has burns on the back and legs from a house fire has become agitated and restless 9 hours after being admitted to the hospital. Which action should the nurse take first? a. Administer the ordered morphine sulfate IV. b. Assess orientation and level of consciousness. c. Use pulse oximetry to check the oxygen saturation. d. Stay at the bedside and reassure the patient.

Correct Answer: C Rationale: Agitation in a patient who may have suffered inhalation injury might indicate hypoxemia, and this should be assessed by the nurse first. Administration of morphine may be indicated if the nurse determines that the agitation is caused by pain. Assessing loss of consciousness and orientation is also appropriate but not as essential as determining whether the patient is hypoxemic. Reassurance is not helpful to reduce agitation in a hypoxemic patient.

2. A patient is admitted to the emergency department after suffering an electrical burn from exposure to a high-voltage current. In addition to the burn injuries, the most essential assessment is a. blood urea nitrogen (BUN) and creatinine levels. b. pupils' reaction to light. c. extremity movement. d. peripheral pulses.

Correct Answer: C Rationale: All patients with electrical burns should be considered at risk for cervical spine injury, and assessments of extremity movement will provide baseline data. The other assessment data are also necessary but not as essential as determining cervical spine status

8. A patient with severe burns has fluid replacement ordered using the Parkland formula. The initial rate of administration is 1050 ml/hr. The nurse would expect that 18 hours after the burn occurred, the rate of the fluid administration should be _____ ml/hr. a. 263 b. 350 c. 525 d. 1050

Correct Answer: C Rationale: Half of the fluid replacement using the Parkland formula is administered in the first 8 hours and the other half over the next 16 hours. In this case, the patient should receive half of the initial rate, or 525 ml/hr.

20. Which of these laboratory results requires the most rapid action by the nurse who is caring for a patient who suffered a large burn 48 hours ago? a. Serum sodium, 146 mEq/L b. BUN 36 mg/dl c. Serum potassium 6.2 mEq/dl d. Hct 52%

Correct Answer: C Rationale: Hyperkalemia can lead to fatal bradycardias and indicates that the patient requires cardiac monitoring and immediate treatment to lower the potassium level. The other laboratory values are also abnormal and require changes in treatment, but they are not as immediately life-threatening as the elevated potassium level.

16. A patient with burns has the nursing diagnosis of pain related to lack of knowledge of pain-control methods. The most appropriate nursing action for this nursing diagnosis is to a. request that the health care provider order a patient-controlled analgesia machine for the patient. b. administer pain medications on a routine basis so that pain does not become out of control. c. teach the patient how to use ordered analgesics with adjunctive methods such as guided imagery and relaxation. d. use sedative or amnesic drugs in combination with opioids to reduce the perception of the pain experience.

Correct Answer: C Rationale: Since the etiology of the pain is the patient's lack of knowledge about pain control, teaching the patient about how to use adjunctive methods is an appropriate intervention. The other nursing actions may also be used to reduce pain but do not require any behavior change by the patient.

5. On admission to the burn unit, a patient with an approximate 25% total body surface area (TBSA) burn has the following initial laboratory results: Hct 56%, Hb 17.2 mg/dl (172 g/L), serum K+ 4.8 mEq/L (4.8 mmol/L), and serum Na+ 135 mEq/L (135 mmol/L). Based on these findings, the nurse should plan to a. document the findings in the patient's record. b. continue to monitor the laboratory results. c. increase the rate of the ordered IV solution. d. type and crossmatch for a blood transfusion.

Correct Answer: C Rationale: The patient's lab data show hemoconcentration, which may lead to a decrease in blood flow to the microcirculation unless fluid intake is increased. Documentation and continuing to monitor are inadequate responses to the data. Since the hematocrit and hemoglobin are elevated, a transfusion is inappropriate.

In the cardiac assessment, the ____________ should be inspected and palpated for symmetry, skin integrity, color, temp, hair distribution, cap refill, toe nails, and superficial veins.

Lower extremities

S1 is the ____________ sound heard when the AV valves close because the ventricles have been filled.

Lub

4. Six hours after a thermal burn injury involving the anterior and posterior chest and both arms, the nurse obtains all of these data when assessing a patient. Which information is most important to communicate to the health care provider? a. Blood pressure is 94/46 per arterial line. b. Cardiac monitor shows a pulse rate of 104. c. Urine output is 20 to 30 ml per hour. d. Serous exudate is leaking from the burns.

Correct Answer: C Rationale: The urine output should be at least 30 to 50 ml/hour during the emergent phase, when the patient is at great risk for hypovolemic shock. The nurse should notify the health care provider because a higher IV rate is needed. BP during the emergent phase should be greater than 90 systolic, and the pulse rate should be less than 120. Serous exudate from the burns is expected during the emergent phase.

9. The nurse determines that fluid replacement for a patient with major burns is adequate, based on the finding of a. daily weight unchanged from admission. b. BP of 90/58. c. urinary output of 40 ml/hr. d. total fluid intake equal to urinary output.

Correct Answer: C Rationale: When fluid intake is adequate, the urine output will be at least 30 to 50 ml/hr. The patient's weight and ratio of intake to output are not useful in this situation because of the effects of third spacing and evaporative fluid loss. A BP of 90/58 is an indication of hypovolemia in a patient who has had a major burn injury.

19. The nurse caring for a patient admitted with burns over 30% of the body surface will recognize that the patient has moved from the emergent to the acute phase of the burn injury when a. the patient has been hospitalized for 48 hours. b. blisters and edema have subsided. c. white blood cell levels decrease. d. the patient has large quantities of pale urine.

Correct Answer: D Rationale: At the end of the emergent phase, capillary permeability normalizes and the patient begins to diurese large amounts of urine with a low specific gravity. Although this may occur at about 48 hours, it may be longer in some patients. Blisters and edema begin to resolve, but this process requires more time. White blood cells may increase or decrease based on the patient's immune status and any infectious processes

24. To provide wound care for the patient with deep partial-thickness and full-thickness burns, the nurse plans to a. apply sterile wet-to-dry dressings to burned areas bid. b. clean and scrub the wounds twice a week to remove eschar. c. immerse the patient in a hydrotherapy tank for 30 minutes 4 times daily. d. shower or bathe the patient daily to remove loose, necrotic skin.

Correct Answer: D Rationale: Daily showers or baths are commonly used for wound debridement. Wet-to-dry dressings are not usually used because the dressings will pull away new dermis and epidermis. Although some dressings can remain in place for 3 days, the wounds are gently cleaned, not scrubbed. Hydrotherapy can lead to sodium loss and decreased body temperature and is not ordered 4 times daily.

10. The nurse admitting a patient with an extensive burn injury develops a nursing diagnosis of risk for imbalanced nutrition: less than body requirements related to high caloric needs. The initial action by the nurse should be to a. encourage an oral intake of at least 5000 kcal per day. b. administer multiple vitamins and minerals in the IV solution. c. infuse total parenteral nutrition via a central catheter. d. insert a feeding tube and give 20 ml/hr enteral feedings.

Correct Answer: D Rationale: Enteral feedings can usually be initiated during the emergent phase at low rates and increased over 24 to 48 hours to the goal rate. During the emergent phase, the patient will be unable to eat enough calories to meet nutritional needs and may have a paralytic ileus that prevents adequate nutrient absorption. Vitamins and minerals may be administered during the emergent phase, but these will not assist in meeting the patient's caloric needs. Parenteral nutrition increases the infection risk, does not help preserve gastrointestinal function, and is not routinely used in burn patients.

14. Ranitidine (Zantac) is prescribed for a patient who had extensive burn injuries 5 days ago. Which information will the nurse collect to evaluate the effectiveness of the medication? a. Bowel sounds b. Stool frequency c. Stools for occult blood d. Percent of meals eaten

Correct Answer: D Rationale: H2-blockers are given to prevent Curling's ulcer in the patient who has suffered burn injuries. H2-blockers do not impact on bowel sounds, stool frequency, or appetite.

12. When positioning a patient with burns of the head, neck, chest, and right arm and hand, the nurse places the patient a. laterally with a small pillow under the head and the right arm and hand hyperextended. b. supine with no pillow and the right arm and hand flexed in a position of comfort and elevated. c. supine with a small pillow under the head and the right arm and hand elevated on a pillow. d. in a Fowler's position without a pillow with the right arm and hand extended and elevated on a pillow.

Correct Answer: D Rationale: The patient should be placed in Fowler's position to make ventilation easier. Pillows should not be used under the head of a patient with neck burns, and the arms and hands should be extended to avoid flexure contractures, even though this position will not be as comfortable for the patient.

6. A patient is admitted to the burn unit with burns of the head, neck, chest, and back following a garage fire. Upon admission to the unit, the nurse auscultates wheezes in the patient's lungs. One hour later, the wheezes cannot be heard, and lung sounds are decreased. The most appropriate action by the nurse is to a. place the patient in high-Fowler's position. b. encourage the patient to cough and auscultate the lungs again. c. document the results and continue to monitor the ventilation. d. notify the health care provider about the breath sounds.

Correct Answer: D Rationale: The patient with burns of the face and neck and with decreased breath sounds will require intubation and ventilatory assistance, and the health care provider should be notified so that this can be rapidly accomplished. Placing the patient in a more upright position or having the patient cough will not address the problem of airway edema. Continuing to monitor is inappropriate because immediate action should occur.

21. The RN observes all of these actions being taken by a staff nurse who has floated to the unit. Which action requires that the RN intervene? a. The float nurse obtains burn cultures when the patient has a temperature of 95.2° F. b. The float nurse calls the health care provider for an insulin order when a nondiabetic patient has an elevated serum glucose. c. The float nurse administers as-needed fentanyl (Sublimaze) IV to a patient 5 minutes before a dressing change. d. The float nurse lowers room temperature to 76° F during the dressing change of a patient with large burns.

Correct Answer: D Rationale: The patient with large burns requires a room temperature of 85° F degrees during dressing changes to avoid becoming hypothermic. Hypothermia is an indicator of possible sepsis, and cultures are appropriate. Nondiabetic patients may require insulin because stress and high calorie intake may lead to temporary hyperglycemia. Fentanyl peaks 5 minutes after IV administration and should be used just before and during dressing changes for pain management.

Nurse on clinical unit is assigned to four patients. Which patient should she assess first? a. Patient with a skull fracture whose nose is bleeding b. Older patient with a stroke who is confused and whose daughter is present c. Patient with meningitis who is suddenly agitated and reporting a headache of 10 on a 0-10 scale d. Patient who had a craniotomy for a brain tumor who now 3 days postoperative had had continued vomiting

Correct answer: c Rationale: The patient with meningitis should be seen first; patients with meningitis must be observed closely for manifestations of elevated ICP, which is thought to result from swelling around the dura and increased cerebrospinal fluid (CSF) volume. Sudden change in the level of consciousness or change in behavior along with a sudden severe headache may indicate an acute elevation of ICP. The patient who has undergone cranial surgery should be seen second; although nausea and vomiting are common after cranial surgery, it can result in elevations of ICP. Nausea and vomiting should be treated with antiemetics. The patient with a skull fracture needs to be evaluated for CSF leakage occurring with the nose bleed and should be seen third. Confusion after a stroke may be expected; the patient should have a family member present.

A diabetic patient who has had vomiting and diarrhea for the past 3 days is admitted to the hospital with a blood glucose of 748 mg/ml (41.5 mmol/L) and a urinary output of 120 ml in the first hour. The vital signs are blood pressure (BP) 72/62; pulse 128, irregular and thready; respirations 38; and temperature 97° F (36.1° C). The patient is disoriented and lethargic with cold, clammy skin and cyanosis in the hands and feet. The nurse recognizes that the patient is experiencing the a. progressive stage of septic shock. b. compensatory stage of diabetic shock. c. refractory stage of cardiogenic shock. d. progressive stage of hypovolemic shock.

D

A patient is admitted to the burn center with burns of his head and neck, chest and back, and left arm and hand following an explosion and a fire in his garage. On admission to the unit, the nurse auscultates wheezes through the lung fields. On reassessment, the nurse notes that the wheezes are gone and the breath sounds are greatly diminished. Which of the following actions is most appropriate for the nurse to take next? A. place the patient in a high Fowler's position B. encourage the patient to cough and auscultate the lungs again C. document the results and continue to monitor the patient's progress D. anticipate the need for endotracheal intubation and notify the physician

D

A patient is being treated for second and third-degree burns over 30% of his body and is now ready for discharge. The nurse provides discharge instructions related to wound care. What statement indicates that the patient understands the instruction? A. "i can expect occasional periods of low-grade fever and can take tylenol every 4 hours" B. "I must wear my Jobst elastic garment all day and can only remove it when I'm going to bed" C. "I will need to take sponge baths at home to avoid exposing the wounds to unsterile bath water" D. "if any healed areas break open I should cover then with a sterile dressing and immediately report it"

D

The best nursing intervention for a patient in shock who has a nursing diagnosis of fear related to perceived threat of death is to a. arrange for the hospital pastoral care staff to visit the patient. b. ask the health care provider to prescribe a sedative drug for the patient. c. leave the patient alone with family members whenever possible. d. place the patient's call bell where it can be easily reached.

D

The nurse is caring for a patient admitted with a urinary tract infection and sepsis. Which information obtained in the assessment indicates a need for a change in therapy? a. The patient is restless and anxious. b. The patient has a heart rate of 134. c. The patient has hypotonic bowel sounds. d. The patient has a temperature of 94.1° F.

D

When caring for a critically ill patient who is being mechanically ventilated, the nurse will astutely monitor for which of the following clinical manifestations of multiple organ dysfunction syndrome (MODS)? A) Increased gastrointestinal (GI) motility B) Increased serum albumin C) Decreased blood urea nitrogen (BUN)/creatinine ratio D) Decreased respiratory compliance

D Clinical manifestations of MODS include symptoms of respiratory distress, signs and symptoms of decreased renal perfusion, decreased serum albumin and prealbumin, decreased GI motility, acute neurologic changes, myocardial dysfunction, disseminated intravascular coagulation (DIC), and changes in glucose metabolism.

The nurse assesses a patient for signs of meningeal irritation and observes for nuchal rigidity. What indicates the presence of this sign of meningeal irritation? A. Tonic spasms of the legs B. Curling in a fetal position C. Arching of the neck and back D. Resistance to flexion of the neck

D Nuchal rigidity is a clinical manifestation of meningitis. During assessment, the patient will resist passive flexion of the neck by the health care provider. Tonic spasms of the legs, curling in a fetal position, and arching of the neck and back are not related to meningeal irritation.

A massive gastrointestinal bleed has resulted in hypovolemic shock in an elderly patient. Which of the following is a priority nursing diagnosis? A) Acute pain B) Impaired tissue integrity C) Decreased cardiac output D) Ineffective tissue perfusion

D The many deleterious effects of shock are all related to inadequate perfusion and oxygenation of every body system. This nursing diagnosis supersedes the other diagnoses.

The nurse is administering a dose of digoxin (Lanoxin) to a patient with heart failure (HF). The nurse would become concerned with the possibility of digitalis toxicity if the patient reported which symptom(s)? A Muscle aches B Constipation C Pounding headache D Anorexia and nausea

D Anorexia and nausea Anorexia, nausea, vomiting, blurred or yellow vision, and cardiac dysrhythmias are all signs of digitalis toxicity. The nurse would become concerned and notify the health care provider if the patient exhibited any of these symptoms.

A stable patient with acute decompensated heart failure (ADHF) suddenly becomes dyspneic. Before positioning the patient on the bedside, what should the nurse assess first? A Urine output B Heart rhythm C Breath sounds D Blood pressure

D Blood pressure The nurse should evaluate the blood pressure before dangling the patient on the bedside because the blood pressure can decrease as blood pools in the periphery and preload decreases. If the patient's blood pressure is low or marginal, the nurse should put the patient in the semi-Fowler's position and use other measures to improve gas exchange.

Beyond the first year after a heart transplant, the nurse knows that what is a major cause of death? A Infection B Acute rejection C Immunosuppression D Cardiac vasculopathy

D Cardiac vasculopathy Beyond the first year after a heart transplant, malignancy (especially lymphoma) and cardiac vasculopathy (accelerated CAD) are the major causes of death. During the first year after transplant, infection and acute rejection are the major causes of death. Immunosuppressive therapy will be used for posttransplant management to prevent rejection and increases the patient's risk of an infection.

A male patient with a long-standing history of heart failure has recently qualified for hospice care. What measure should the nurse now prioritize when providing care for this patient? A Taper the patient off his current medications. B Continue education for the patient and his family. C Pursue experimental therapies or surgical options. D Choose interventions to promote comfort and prevent suffering.

D Choose interventions to promote comfort and prevent suffering. The central focus of hospice care is the promotion of comfort and the prevention of suffering. Patient education should continue, but providing comfort is paramount. Medications should be continued unless they are not tolerated. Experimental therapies and surgeries are not commonly used in the care of hospice patients.

After having an MI, the nurse notes the patient has jugular venous distention, gained weight, developed peripheral edema, and has a heart rate of 108/minute. What should the nurse suspect is happening? A ADHF B Chronic HF C Left-sided HF D Right-sided HF

D Right-sided HF An MI is a primary cause of heart failure. The jugular venous distention, weight gain, peripheral edema, and increased heart rate are manifestations of right-sided heart failure.

The nurse is teaching a senior citizen's group about signs and symptoms of a stroke. Which statement by the nurse would provide accurate information? A. "Take the person to the hospital if a headache lasts for more than 24 hours." B. "Stroke symptoms usually start when the person is awake and physically active." C. "A person with a transient ischemic attack has mild symptoms that will go away." D. "Call 911 immediately if a person develops slurred speech or difficulty speaking."

D. "Call 911 immediately if a person develops slurred speech or difficulty speaking." Medical assistance should be obtained immediately for someone with signs and symptoms of a stroke such as sudden numbness; weakness; paralysis of the face, arm, or leg (especially on one side of the body); sudden confusion; trouble speaking or understanding; slurred speech; sudden trouble seeing in one or both eyes; sudden trouble walking; dizziness; loss of balance or coordination; or a sudden, severe headache with no known cause. A person with signs and symptoms of a transient ischemic attack should seek medical attention immediately because it is unknown if the symptoms will resolve or persist and progress to a stroke. Onset of signs and symptoms of a stroke vary depending on the type. Onset of an ischemic thrombotic stroke usually occurs at rest. Onset of an ischemic embolic stroke is not related to rest or activity, and onset of a hemorrhagic stroke usually occurs with activity.

Which problem places a person at highest risk for septic shock? A. Kidney failure B. Cirrhosis C. Lung cancer D. 40% burn injury

D. 40% burn injury The skin forms the first barrier to prevent entry of organisms into the body; this client is at very high risk for sepsis and death. Although the client with kidney failure has an increased risk for infection, his skin is intact, unlike the client with burn injury. Although the liver acts as a filter for pathogens, the client with cirrhosis has intact skin, unlike the burned client. The client with lung cancer may be at risk for increased secretions and infection, but risk is not as high as for a client with open skin.

Which problem places a person at highest risk for septic shock? A. Kidney failure B. Cirrhosis C. Lung cancer D. 40% burn injury

D. 40% burn injury (The skin forms the first barrier to prevent entry of organisms into the body; this client is at very high risk for sepsis and death.)

Of the following patients, the nurse recognizes that the one with the highest risk for stroke is a(n): A. obese 45-year old Native American. B. 35-year-old Asian American woman who smokes. C. 32-year-old white woman taking oral contraceptives. D. 65-year-old African American man with hypertension.

D. 65-year-old African American man with hypertension. Nonmodifiable risk factors for stroke include age (older than 65 years), male gender, ethnicity or race (incidence is highest in African Americans; next highest in Hispanics, Native Americans/Alaska Natives, and Asian Americans; and next highest in white people), and family history of stroke or personal history of a transient ischemic attack or stroke. Modifiable risk factors for stroke include hypertension (most important), heart disease (especially atrial fibrillation), smoking, excessive alcohol consumption (causes hypertension), abdominal obesity, sleep apnea, metabolic syndrome, lack of physical exercise, poor diet (high in saturated fat and low in fruits and vegetables), and drug abuse (especially cocaine). Other risk factors for stroke include a diagnosis of diabetes mellitus, increased serum levels of cholesterol, birth control pills (high levels of progestin and estrogen), history of migraine headaches, inflammatory conditions, hyperhomocystinemia, and sickle cell disease.

The nurse in a primary care provider's office is assessing several patients today. Which patient is most at risk for a stroke? A. A 92-year-old female who takes warfarin (Coumadin) for atrial fibrillation. B. A 28-year-old male who uses marijuana after chemotherapy to control nausea. C. A 42-year-old female who takes oral contraceptives and has migraine headaches. D. A 72-year-old male who has hypertension and diabetes mellitus and smokes tobacco.

D. A 72-year-old male who has hypertension and diabetes mellitus and smokes tobacco. Stroke risk increases after 65 years of age. Strokes are more common in men. Hypertension is the single most important modifiable risk factor for stroke. Diabetes mellitus is a significant stroke risk factor; and smoking nearly doubles the risk of a stroke. Other risk factors include drug abuse (especially cocaine), high-dose oral contraception use, migraine headaches, and untreated heart disease such as atrial fibrillation.

A 74-year-old man who has right-sided extremity paralysis related to a thrombotic stroke develops constipation. Which action should the nurse take first? A. Assist the patient to the bathroom every 2 hours. B. Provide incontinence briefs to wear during the day. C. Administer a bisocodyl (Dulcolax) rectal suppository every day. D. Arrange for several servings per day of cooked fruits and vegetables.

D. Arrange for several servings per day of cooked fruits and vegetables. Patients after a stroke frequently have constipation. Dietary management includes the following: fluid intake of 2500 to 3000 mL daily, prune juice (120 mL) or stewed prunes daily, cooked fruit three times daily, cooked vegetables three times daily, and whole-grain cereal or bread three to five times daily. Patients with urinary incontinence should be assisted to the bathroom every 2 hours when appropriate. Suppositories may be ordered for short-term management if the patient does not respond to increased fluid and fiber. Incontinence briefs are indicated as a short-term intervention for urinary incontinence.

A patient admitted with possible stroke has been aphasic for 3 hours and his current blood pressure (BP) is 174/94 mm Hg. Which order by the health care provider should the nurse question? a. Keep head of bed elevated at least 30 degrees. b. Infuse normal saline intravenously at 75 mL/hr. c. Administer tissue plasminogen activator (tPA) per protocol. d. Administer a labetalol (Normodyne) drip to keep BP less than 140/90 mm Hg.

D. Because elevated BP may be a protective response to maintain cerebral perfusion, antihypertensive therapy is recommended only if mean arterial pressure (MAP) is >130 mm Hg or systolic pressure is >220 mm Hg. Fluid intake should be 1500 to 2000 mL daily to maintain cerebral blood flow. The head of the bed should be elevated to at least 30 degrees, unless the patient has symptoms of poor tissue perfusion. tPA may be administered if the patient meets the other criteria for tPA use.

A client with hypovolemic shock has these vital signs: temperature 97.9° F; pulse 122 beats/min; blood pressure 86/48 mm Hg; respirations 24 breaths/min; urine output 20 mL for last 2 hours; skin cool and clammy. Which medication order for this client does the nurse question? A. Dopamine (Intropin) 12 mcg/kg/min B. Dobutamine (Dobutrex) 5 mcg/kg/min C. Plasmanate 1 unit D. Bumetanide (Bumex) 1 mg IV

D. Bumetanide (Bumex) 1 mg IV A diuretic such as bumetanide will decrease blood volume in a client who is already hypovolemic; this order should be questioned because this is not an appropriate action to expand the client's blood volume. The other orders are appropriate for improving blood pressure in shock, and do not need to be questioned.

When caring for an obtunded client admitted with shock of unknown origin, which action should the nurse take first? A. Obtain IV access, and hang prescribed fluid infusions. B. Apply the automatic blood pressure cuff. C. Assess level of consciousness and pupil reaction to light. D. Check the airway and respiratory status.

D. Check the airway and respiratory status. (Although this is an important action, an airway is essential.)

• Drug therapy is a critical part of the care of patients with HF: . .

Diuretics ACE inhibitors ARB's Adrenergic blockers vasodilator digitalis

When caring for an obtunded client admitted with shock of unknown origin, which action does the nurse take first? A. Obtain IV access and hang prescribed fluid infusions. B. Apply the automatic blood pressure cuff. C. Assess level of consciousness and pupil reaction to light. D. Check the airway and respiratory status.

D. Check the airway and respiratory status. When caring for any client, determining airway and respiratory status is the priority. The airway takes priority over obtaining IV access, applying the blood pressure cuff, and assessing for changes in the client's mental status.

When delegating care for clients on the burn unit, which client does the charge nurse assign to an RN who has floated to the burn unit from the intensive care unit? A. Burn unit client who is being discharged after 6 weeks and needs teaching about wound care B. Recently admitted client with a high-voltage electrical burn C. A client who has a 25% total body surface area (TBSA) burn injury, for whom daily wound débridement has been prescribed D. Client receiving IV lactated Ringer's solution at 150 mL/hr

D. Client receiving IV lactated Ringer's solution at 150 mL/hr An RN float nurse will be familiar with administration of IV fluids and with signs of fluid overload, such as shortness of breath, and so could be assigned to the client receiving IV lactated Ringer's solution at 150 mL/hr. The client needing teaching about wound care, the client with a high-voltage electrical burn, and the client with a 25% TBSA burn injury all require specialized knowledge about burn injuries and should be assigned to RNs who have experience caring for clients with burn injuries.

When teaching about clopidogrel (Plavix), the nurse will tell the patient with cerebral atherosclerosis a. to monitor and record the blood pressure daily. b. that Plavix will dissolve clots in the cerebral arteries. c. that Plavix will reduce cerebral artery plaque formation. d. to call the health care provider if stools are bloody or tarry.

D. Clopidogrel (Plavix) inhibits platelet function and increases the risk for gastrointestinal bleeding, so patients should be advised to notify the health care provider about any signs of bleeding. The medication does not lower blood pressure, decrease plaque formation, or dissolve clots.

When teaching a patient with endocarditis how to prevent recurrence of the infection, the nurse instructs the patient to a. start on antibiotic therapy when exposed to person with infections b. take on ASA a day to prevent vegetative lesions from forming around the valves c. always maintain continuous antibiotic therapy to prevent development of any systemic infections d. obtain prophylactic antibiotic therapy before certain invasive medical or dental procedures

D. Obtain prophylactic antibiotic therapy before certain invasive medical or dental procedures

A 58-year-old patient with a left-brain stroke suddenly bursts into tears when family members visit. The nurse should a. use a calm voice to ask the patient to stop the crying behavior. b. explain to the family that depression is normal following a stroke. c. have the family members leave the patient alone for a few minutes. d. teach the family that emotional outbursts are common after strokes.

D. Patients who have left-sided brain stroke are prone to emotional outbursts that are not necessarily related to the emotional state of the patient. Depression after a stroke is common, but the suddenness of the patient's outburst suggests that depression is not the major cause of the behavior. The family should stay with the patient. The crying is not within the patient's control and asking the patient to stop will lead to embarrassment.

S2 is the ____________ sound heard when aortic and pulmonic (semilunar) valves close, and the ventricles emptied.

Dub

A client is in the acute phase of burn injury. For which action does the nurse decide to coordinate with the registered dietitian? A. Discouraging having food brought in from the client's favorite restaurant B. Providing more palatable choices for the client C. Helping the client lose weight D. Planning additions to the standard nutritional pattern

D. Planning additions to the standard nutritional pattern Nutritional requirements for the client with a large burn area can exceed 5000 kcal/day. In addition to a high calorie intake, the burn client requires a diet high in protein for wound healing. Consultation with the dietitian is required to help the client achieve the correct nutritional balance. It is fine for the client with a burn injury to have food brought in from the outside. The hospital kitchen can be consulted to see what other food options may be available to the client. It is not therapeutic for the client with burn injury to lose weight.

Nurses in change-of-shift report are discussing the care of a patient with a stroke who has progressively increasing weakness and decreasing level of consciousness (LOC). Which nursing diagnosis do they determine has the highest priority for the patient? a. Impaired physical mobility related to weakness b. Disturbed sensory perception related to brain injury c. Risk for impaired skin integrity related to immobility d. Risk for aspiration related to inability to protect airway

D. Protection of the airway is the priority of nursing care for a patient having an acute stroke. The other diagnoses are also appropriate, but interventions to prevent aspiration are the priority at this time.

A patient with left-sided weakness that started 60 minutes earlier is admitted to the emergency department and diagnostic tests are ordered. Which test should be done first? a. Complete blood count (CBC) b. Chest radiograph (Chest x-ray) c. 12-Lead electrocardiogram (ECG) d. Noncontrast computed tomography (CT) scan

D. Rapid screening with a noncontrast CT scan is needed before administration of tissue plasminogen activator (tPA), which must be given within 4.5 hours of the onset of clinical manifestations of the stroke. The sooner the tPA is given, the less brain injury. The other diagnostic tests give information about possible causes of the stroke and do not need to be completed as urgently as the CT scan.

A 73-year-old patient with a stroke experiences facial drooping on the right side and right-sided arm and leg paralysis. When admitting the patient, which clinical manifestation will the nurse expect to find? a. Impulsive behavior b. Right-sided neglect c. Hyperactive left-sided tendon reflexes d. Difficulty comprehending instructions

D. Right-sided paralysis indicates a left-brain stroke, which will lead to difficulty with comprehension and use of language. The left-side reflexes are likely to be intact. Impulsive behavior and neglect are more likely with a right-side stroke.

The female patient has been brought to the ED with a sudden onset of a severe headache that is different from any other headache she has had previously. When considering the possibility of a stroke, which type of stroke should the nurse know is most likely occurring? A. TIA B. Embolic stroke C. Thrombotic stroke D. Subarachnoid hemorrhage

D. Subarachnoid hemorrhage Headache is common in a patient who has a subarachnoid hemorrhage or an intracerebral hemorrhage. A TIA is a transient loss of neurologic function usually without a headache. A headache may occur with an ischemic embolic stroke, but severe neurologic deficits are the initial symptoms. The ischemic thrombotic stroke manifestations progress in the first 72 hours as infarction and cerebral edema increase.

The nurse is caring for postoperative clients at risk for hypovolemic shock. Which of these represents early symptoms of shock? A. Hypotension B. Bradypnea C. Heart blocks D. Tachycardia

D. Tachycardia (Heart and respiratory rates increased from the client's baseline level or a slight increase in diastolic blood pressure may be the only objective manifestation of this early stage of shock.)

Which nurse should be assigned to care for an intubated client who has septic shock as the result of a methicillin-resistant Staphylococcus aureus (MRSA) infection? A. The LPN/LVN who has 20 years of experience B. The new RN who recently finished orienting and is working independently with moderately complex clients C. The RN who will also be caring for a client who had coronary artery bypass graft (CABG) surgery 12 hours ago D. The RN with 2 years of experience in intensive care

D. The RN with 2 years of experience in intensive care The RN with current intensive care experience who is not caring for a postoperative client would be an appropriate assignment. Care of the unstable client with intubation and mechanical ventilation is not within the scope of practice for the LPN/LVN. A client who is experiencing septic shock is too complex for the new RN. Although the RN who is also caring for the post-CABG client is experienced, this assignment will put the post-CABG client at risk for MRSA infection.

A 56-year-old patient arrives in the emergency department with hemiparesis and dysarthria that started 2 hours previously, and health records show a history of several transient ischemic attacks (TIAs). The nurse anticipates preparing the patient for a. surgical endarterectomy. b. transluminal angioplasty. c. intravenous heparin administration. d. tissue plasminogen activator (tPA) infusion.

D. The patient's history and clinical manifestations suggest an acute ischemic stroke and a patient who is seen within 4.5 hours of stroke onset is likely to receive tPA (after screening with a CT scan). Heparin administration in the emergency phase is not indicated. Emergent carotid transluminal angioplasty or endarterectomy is not indicated for the patient who is having an acute ischemic stroke.

____________ is unilateral swelling caused by obstructed lymph nodes.

Lymphedema

Why are the manifestations of most types of shock the same regardless of what specific events or condition caused the shock to occur? A. The blood, blood vessels, and heart are directly connected to each other so that when one is affected, all three are affected. B. Because blood loss occurs with all types of shock, the most common first manifestation is hypotension. C. Every type of shock interferes with oxygenation and metabolism of all cells in the same sequence. D. The sympathetic nervous system is triggered by any type of shock and initiates the stress response.

D. The sympathetic nervous system is triggered by any type of shock and initiates the stress response. (Most manifestations of shock are similar regardless of what starts the process or which tissues are affected first. These common manifestations result from physiologic adjustments (compensatory mechanisms) in an attempt to ensure continued oxygenation of vital organs. These adjustment actions are performed by the sympathetic nervous system triggering the stress response and activating the endocrine and cardiovascular systems. Reference: p. 812, Physiological Integrity)

A patient in the emergency department with sudden-onset right-sided weakness is diagnosed with an intracerebral hemorrhage. Which information about the patient is most important to communicate to the health care provider? a. The patient's speech is difficult to understand. b. The patient's blood pressure is 144/90 mm Hg. c. The patient takes a diuretic because of a history of hypertension. d. The patient has atrial fibrillation and takes warfarin (Coumadin).

D. The use of warfarin probably contributed to the intracerebral bleeding and remains a risk factor for further bleeding. Administration of vitamin K is needed to reverse the effects of the warfarin, especially if the patient is to have surgery to correct the bleeding. The history of hypertension is a risk factor for the patient but has no immediate effect on the patient's care. The BP of 144/90 indicates the need for ongoing monitoring but not for any immediate change in therapy. Slurred speech is consistent with a left-sided stroke, and no change in therapy is indicated.

The client is a burn victim who is noted to have increasing edema and decreased urine output as a result of the inflammatory compensation response. What does the nurse do first? A. Administer a diuretic. B. Provide a fluid bolus. C. Recalculate fluid replacement based on time of hospital arrival. D. Titrate fluid replacement.

D. Titrate fluid replacement. The intravenous fluid rate should be adjusted on the basis of urine output plus serum electrolyte values (titration of fluids). A common mistake in treatment is giving diuretics to increase urine output. Giving a diuretic will actually decrease circulating volume and cardiac output by pulling fluid from the circulating blood volume to enhance diuresis. Fluid boluses are avoided because they increase capillary pressure and worsen edema. Fluid replacement formulas are calculated from the time of injury, not from the time of arrival at the hospital.

Which intervention is most appropriate when communicating with a patient suffering from aphasia following a stroke? A. Present several thoughts at once so that the patient can connect the ideas. B. Ask open-ended questions to provide the patient the opportunity to speak. C. Finish the patient's sentences to minimize frustration associated with slow speech. D. Use simple, short sentences accompanied by visual cues to enhance comprehension.

D. Use simple, short sentences accompanied by visual cues to enhance comprehension. When communicating with a patient with aphasia, the nurse should present one thought or idea at a time. Ask questions that can be answered with a "yes," "no," or simple word. Use visual cues and allow time for the individual to comprehend and respond to conversation.

The nurse is caring for postoperative clients at risk for hypovolemic shock. Which condition represents an early symptom of shock? A. Hypotension B. Bradypnea C. Heart blocks D. Tachycardia

D. tachycardia Heart and respiratory rates increased from the client's baseline level or a slight increase in diastolic blood pressure may be the only objective manifestation of this early stage of shock. Catecholamine release occurs early in shock as a compensation for fluid loss; blood pressure will be normal. Early in shock, the client displays rapid, not slow, respirations. Dysrhythmias are a late sign of shock; they are related to lack of oxygen to the heart.

Tachycardia (symptom of HF)

Early clinical symptom Compensate for failing ventricle-> increase HR Decrease CO activates SNS May be reduced or absent in pt on beta-blockers

Fatigue (symptom of HF)

Early symptom Unable to perform usual activities Caused by: decreased CO, impaired perfusion, decreased oxygenation, anemia

Acute Decompensated HF (ADHF)

Early: increased pulmonary venous pressure, increased RR, decreased PaO2 Later: interstitial edema, tachypnea, SOB End: alveolar edema, respiratory acidosis

A nursing intervention is indicated for the patient with hemiplegia is a. the use of a footboard to prevent plantar flexion b. immobilization of the affected arm against the chest with a sling c. positioning the patient in bed with each joint lower than the joint proximal to it d. having the patient perform passive ROM of the affected limb with the unaffected limb

D: Having the patient perform passive ROM of the affected limb with the unaffected limb- active ROM should be initiated on the unaffected side as soon as possible, and passive ROM of the affected side should be started on the first day. Having the patient actively exercise the unaffected side provides the patient with active and passive ROM as needed. Use of footboards is controversial because they stimulate plantar flexion. The unaffected arm should be supported, but immobilization may precipitate a painful shoulder-hand syndrome. The patient should be positioned with each joint higher than the joint proximal to it to prevent dependent edema.

The nurse can assist the patient and the family in coping with the long term effects of a stroke by a. informing family members that the patient will need assistance with almost all ADLs b. explaining that the patient's prestroke behavior will return as improvement progresses c. encouraging the patient and family members to seek assistance from family therapy or stroke support groups d. helping the patient and family understand the significance of residual stroke damage to promote problem solving and planning

D: Helping the patient and family understand the significance of residual stroke damage to promote problem solving and planning- the patient and family need accurate and complete information about the effects of the stroke to problem solve and make plans for chronic care of the patient. It is uncommon for patients with major strokes to return completely to pre stroke function, behaviors, and role, and both the patient and family will mourn these losses. The patient's specific needs for care must be identified, and rehabilitation efforts should be continued at home. Family therapy and support groups may be helpful for some patients and families.

The priority intervention in the emergency department for the patient with a stroke is a. intravenous fluid replacement b. administration of osmotic diuretics to reduce cerebral edema c. initiation of hypothermia to decrease the oxygen needs of the brain d. maintenance of respiratory function with a patent airway and oxygen administration

D: Maintenance of respiratory function with a patent airway and oxygen administration- the first priority in acute management of the patient with a stroke is preservation of life. Because the patient with a stroke may be unconscious or have a reduced gag reflex, it is most important to maintain a patent airway for the patient and provide oxygen if respiratory effort is impaired. IV fluid replacement, treatment with osmotic diuretics, and perhaps hypothermia may be used for further treatment.

A patient's wife asks the nurse why her husband did not receive the clot busting medication (tPA) she has been reading about. Her husband is diagnosed with a hemorrhagic stroke. What should the nurse respond? a. He didn't arrive within the time frame for that therapy b. Not every is eligible for this drug. Has he had surgery lately? c. You should discuss the treatment of your husband with your doctor d. The medication you are talking about dissolves clots and could cause more bleeding in your husband's head

D: The medication you are talking about dissolves clots and could cause more bleeding in your husband's head- tPA dissolves clots and increases the risk for bleeding. It is not used with hemorrhagic strokes. If the patient had a thrombotic/embolic stroke the time frame would be important as well as a history of surgery. The nurse should answer the question as accurately as possible and then encourage the individual to talk with the primary care physician if he or she has further questions.

A patient comes to the emergency department immediately after experiencing numbness of the face and an inability to speak, but while the patient awaits examination, the symptoms disappear and the patient request discharge. The nurse stresses that it is important for the patient to be evaluated primarily because a. the patient has probably experienced an asymptomatic lacunar stroke b. the symptoms are likely to return and progress to worsening neurologic deficit in the next 24 hours c. neurologic deficits that are transient occur most often as a result of small hemorrhages that clot off d. the patient has probably experienced a transient ischemic attack (TIA), which is a sign of progressive cerebral vascular disease

D: The patient has probably experienced a transient ischemic attack (TIA), which is a sign of progressive cerebral vascular disease- A TIA is a temporary focal loss of neurologic function caused by ischemia of an area of the brain, usually lasting only about 3 hours. TIAs may be due to microemboli from heart disease or carotid or cerebral thrombi and are a warning of progressive disease. Evaluation is necessary to determine the cause of the neurologic deficit and provide prophylactic treatment if possible.

Neurohormonal response (compensatory mechanism) RAAS

Decrease in CO-> decrease in blood flow to kidneys -kidneys sense this as decreased volume, juxtaglomerular apparatus -renin-angiotensin-aldosterone-system (RAAS) response causes: *sodium and water retention *peripheral vasoconstriction-> increase in BP

Diuretics

Decrease volume overload, preload Promote sodium and water excretion Monitor K+ levels -Furosemide (Lasix) -Bumetanide (Bumex)

What is the significance of J.F orthostatic hypotension and tachycardia?

Decreased cardiac output

____________ is the occlusion of a deep vein by a blood clot. Characterized by unilateral edema, redness, and heat to the extremity.

Deep vein thrombosis

Frank-Starling Law

Degree of stretch is directly related to the face of the contraction (systole)

The spread of electrical current is called ____________.

Depolarization

Endomyocardial biopsy

Diagnosis of HF -determining new HF that is unresponsive

Echocardiography, nuclear imaging

Diagnosis of HF What is the function? How are the valves working? Measure EF Ischemia, arrhythmias

S1 and S2 are high pitched sounds best auscultated with the ____________ of the stethoscope.

Diaphragm

____________ is when the ventricles relax, begins with the closure of the aortic and pulmonic valves, and ends with the closure of the AV valves.

Diastole

Mixed HF, systolic and diastolic

Diseases with dilated cardiomyopathy (DCM) -poor EF (<35%) -high pulmonary pressure -biventricular failure, dilated, poor filling, emptying capacity

What are six things you will teach J.F. and her family?

How to assess J.F.'s IV site daily for redness, tenderness, warmth, swelling, drainage or other signs of infection. -Infection control and hand hygiene- to prevent infection -J.F. may have a fever for weeks. The fever may cause fatigue. Rest periods should be planned. -Along with the antipyretics (to control the fever) tepid baths or a fan may help keep J.F. comfortable with the fever. If shivering occurs, discontinue. -Oral hygiene for J.F. is very important. J.F. should see a dentist regularly and only use a soft toothbrush with toothpaste to brush her teeth, tongue, mouth and gums twice a day and use an antiseptic mouthwash after brushing. -J.F. should avoid using toothpicks. -J.F. should avoid biting her nails. -J.F. should not use and IUD (intrauterine device), if applicable. -J.F. should not self medicate with antibiotics or stop taking the prescribed antibiotics before they have been completed. - How to prepare TPN -How to administer TPN

How will the vital signs appear with increased intracranial pressure? *increased blood pressure, increased temperature, respirations up then down, increased heart rate

How will the vital signs appear with increased intracranial pressure? *increased blood pressure, increased temperature, respirations up then down, increased heart rate

What are the risk factors for infective carditis?

IV drug use, cardiac malformation

Nursing diagnosis, HF

Impaired gas exchange Decreased cardiac output Excess fluid volume Activity intolerance

Hypertrophy (compensatory mechanism)

Increase in muscle mass and cardiac wall thickness in response to overwork and strain -slow response to increased work and strain Causes: *poor contractility *requires more O2 *poor coronary artery circulation, heart can become ischemic

Positive iontropes

Increase myocardial contractility, therapeutic, sign that HF is worsening -beta-agonists--dopamine, dobutamine -Phosphodiesterase inhibitors -Digitalis

Precipitating causes of HF

Increase workload of the ventricles, acute condition, decreased heart function

Pleural effusion

Increased pressure in pulmonary capillaries -HF complication

Nocturia (symptom of HF)

Increased renal blood flow and diuresis When they lie down to sleep fluid moves from interstitial space back into circulatory system, cardiac workload decreases at rest

What is endocarditis?

Inflammation of the endocardium

Initiate *seizure* precautions

Initiate *seizure* precautions

ICP

Intracranial Pressure 10 mmHg

What is the significance of splinter hemorrhages?

It is a late sign of endocarditis

The ____________ should not be visible when the body is upright, but pulsations can be seen at a 45° angle.

Jugular veins

____________ heart failure may cause precordial movement, displaced apical pulse, a palpable thrill, S3 and murmur at apex.

Left-sided

Pulmonary hypertension

Left-sided HF causes increased pressure in blood vessels of lung, increased right ventricular after load, causes right-sided hypertrophy and HF

____________ causes backup of blood into the pulmonary system resulting in pulmonary edema, frothy sputum, wet lung sounds, and low o2 sat.

Left-sided heart failure

Fluid restriction, HF

Less than 2 L/day

Neurohormonal response (compensatory mechanism) ADH

Low CO-> decrease in cerebral perfusion pressure-> pituitary secretes ADH *increase water reabsorption, H2O retention *increase in blood volume

Nutritional therapy. HF

Low sodium diet, less than 2 g/day DASH

Endothelin

Peptides released by vascular endothelial cells that cause vasoconstriction

What do you need to diagnosis infectious endocarditis?

Positive blood culture positive echocardiogram

Neurohormonal response (compensatory mechanism) Endothelin

Potent vasoconstrictor produced by: -vascular endothelial cells -stimulated by ADH, catecholamines, and angiotensin II Causes: -arterial vasoconstriction -increase in cardiac contractility and hypertrophy

PAD

Pulmonary Artery Pressure Diastolic 8 - 10 mmHg

PAS

Pulmonary Artery Pressure Systolic 5 - 25 mmHg

PAP

Pulmonary Artery Pressures systolic: 20 - 30 mmHg diastolic: 8 - 10 mmHg

Wedge/PA

Pulmonary Artery Wedge Pressure systolic: 15 - 25 mmHg diastolic: 8 - 10 mmHg mean: 10 -20mmHg measures end diastolic pressure

PCWP

Pulmonary Capillary Wedge Pressure 5 - 15 mmHg

PVR

Pulmonary Vascular Resistance 40 - 150 dyne*sec/cm5

The ____________ is at the left 2nd intercostal space, you hear S2.

Pulmonic

____________ is narrowing of the space between the pulmonary artery and the right ventricle. Caused congenitally. Murmur at pulmonic area radiating to neck. Thrill at pulmonic area and erbs point.

Pulmonic stenosis

The ____________ conduct impulses to ventricular myocardial tissue.

Purkinje Fibers

Paroxysmal nocturnal dyspnea (PND)

Reaborption of fluid from dependent body when pt is flat. Awake with panic, feeling of suffocation, impulse to sit or stand up. Cough is first clinical symptom: dry, nonproductive cough not relieved by position change or OTC medication

Vasodilators

Reduce circulating blood volume, improve coronary artery circulation -IV nitroglycerin (ICU setting) -Sodium nitroprusside (Nipride) -Nesiritide (Natrecor)

Nitric oxide (counterregulatory mechanism)

Released from vascular endothelium in response to compensatory mechanisms activated by HF -relaxes arterial smooth muscle -vasodilation -decreased after load

If J.F has a BUN of 85 mg, what could this indicate?

Renal failure

If J.F has a creatine of 3.9 mg, what could this indicate?

Renal failure

____________ is the change in polarity when the stimulated cardiac cells return to their original electrical potential state.

Repolarization

Systolic HF causes

Results from inability to of heart to pump blood effectively -impaired contractile function (MI) -increased afterload (hypertension) -cardiomyopathy -mechanical abnormalities (valvular heart disease)

RAP

Right Atrial Pressure 0 - 8 cm H20 or 0 - 6 mmHg

RVP

Right Ventricular Pressure systolic: 20 -30 mmHg diastolic: 0 - 8 mmHg

Right-sided HF side effects

Right ventricle does not contract effectively, blood backs up into right atrium and venous circulation causing: -jugular vein distention -hepatomegaly -splenomegaly -vascular congestion in GI tract -peripheral edema

____________ heart failure may cause precordial movement at xiphoid process, S3 at lower sternal border, systolic murmur, and weight gain.

Right-sided

____________ causes backup of blood into systemic circulation, is characterized by JVD, HTN, peripheral edema, mental confusion, loss of appetite.

Right-sided heart failure

____________ is the "Ventricular gallop", its vibration of blood filling the ventricles, heard when the semilunar valves close. Fluid overload, heart failure

S3

____________ is the "Atrial gallop", it's atrial contraction and ejection of blood into the ventricles.

S4

The ____________ node is located at the junction of the superior vena cava and the right atrium.

SA

The ____________ initiates electrical impulse in the heart at 60-100 bpm, and is termed the "pacemaker".

SA node

Daily weights, HF

Same time, same clothing, same scale Monitor fluid balance

Where would you put your stethoscope to listen to an aortic valve murmur?

Second intercostal on the right side

What are the symptoms of a Pulmonary Embolism?

Shortness of breath Chest pain

____________ is caused when the closure of the aortic and pulmonic valves are not synchronized normally.

Splitting

what is beck's triad

hypotension muffled heart sounds JVD

According to ACC/AHA stages of HF

Stage A patients are at high risk for heart failure with hypertension, diabetes, or metabolic syndrome. Stage B patients have structural heart disease without any sign or symptom Stage C patients have prior or current symptoms of heart failure associated with a known underlying structural heart disease. Stage D patients have refractory heart failure; they have severe symptoms at rest despite maximal medical therapy and require specialized interventions

The 4 landmarks used in the cardiac assessment are ____________, ____________, ____________, and ____________.

Sternum, clavicles, ribs, intercostal spaces

What can an embolization lead to?

Stroke Myocardial infarction Pulmonary embolism DVT

SV

Stroke Volume 50 - 100 ml/beat (CO X 1000)/HR

SVR

Systemic Vascular Resistance 800 - 1200 dynes*sec/cm5 afterload for LV (resistance of systemic vascular bed)

____________ is when the ventricles contract, begins with the closure of the AV valves, and ends with the closure of the aortic and pulmonic valves.

Systole

The ____________ represents ventricular repolarization.

T wave

If J.F does not have diabetes what could make her blood glucose rise?

TPN

Tell the client not to *strain, cough or sneeze.*

Tell the client not to *strain, cough or sneeze.*

The client will often appear? *restless, agitated, complaining of headaches*

The client will often appear? *restless, agitated, complaining of headaches*

What is the greatest risk for J.F. during the process of rehydration, and what would you monitor to detect its development?

The greatest risk is Fluid Volume excess. J.F. needs to be monitored for weight gain, crackles in her lungs, distended neck veins, a bounding pulse, an increased urine output, increased blood pressure and ascities. These would all be signs of fluid volume excess.

A stable patient with acute decompensated heart failure (ADHF) suddenly becomes dyspneic. Before positioning the patient on the bedside, what should the nurse assess first?

The nurse should evaluate the blood pressure before dangling the patient on the bedside because the blood pressure can decrease as blood pools in the periphery and preload decreases. If the patient's blood pressure is low or marginal, the nurse should put the patient in the semi-Fowler's position and use other measures to improve gas exchange.

A nurse is caring for a patient diagnosed with thyrotoxicosis and suspected to have heart failure. What are precipitating causes of heart failure?

The precipitating causes of heart failure include anemia, where the oxygen-carrying capacity of the blood is reduced. Hypoxemia leads to an increase in cardiac output, which then increases the workload on the heart. In Paget's disease, there is an increase in the vascular bed in the skeletal muscles, which causes an increase in the cardiac workload. In pulmonary embolism, an increase in pulmonary pressure decreases cardiac output. Myocarditis and coronary artery disease are the primary causes of heart failure but are unlikely in this patient

This is the earliest sign of increased intracranial pressure? *decreased level of consciousness*

This is the earliest sign of increased intracranial pressure? *decreased level of consciousness*

____________ are usually asymptomatic, but some patients report deep diffuse chest pain.

Thoracic aneurysms

What are splinter hemorrhages?

Tiny lines of blood under the fingernails where clots have damaged the small capillaries under the nail

The ____________ tests for competence of valves in patients with vericose veins.

Trendelenburgs Test

The ____________ is at the left 4th intercostal space, you hear S1.

Tricuspid valve

____________ are veins that have become dilated and have a diminished rate of blood flow and increased venous pressure.

Vericose veins

What are the common causes of increased intracranial pressure? *trauma, hemorrhage, edema, tumors*

What are the common causes of increased intracranial pressure? *trauma, hemorrhage, edema, tumors*

What do you assess for increased intracranial pressure? *level of conscious; it decreases as increased intracranial pressure increases*

What do you assess for increased intracranial pressure? *level of conscious; it decreases as increased intracranial pressure increases*

What is the normal intracranial pressure range? *5-15 mm Hg*

What is the normal intracranial pressure range? *5-15 mm Hg*

What medications will be prescribed? *Anticonvulsants, blood pressure medications, corticosteroids, diuretics*

What medications will be prescribed? *Anticonvulsants, blood pressure medications, corticosteroids, diuretics*

What will babies physically present with? *bulging fontanelles*

What will babies physically present with? *bulging fontanelles*

What's Cushing's Triad? * 3 things; widening pulse pressure, Cheyne stokes respiration, and bradycardia*

What's Cushing's Triad? * 3 things; widening pulse pressure, Cheyne stokes respiration, and bradycardia*

What's widening pulse pressure and how's it related? *When systolic blood pressure goes up and diastolic continues to go down so that they become further apart; example 135/40 is a bad sign!*

What's widening pulse pressure and how's it related? *When systolic blood pressure goes up and diastolic continues to go down so that they become further apart; example 135/40 is a bad sign!

An intern writes an order for "fasting blood glucose levels daily." Is this order appropriate for J.F.? Explain

Yes, it is appropriate. J.F. is at risk for hyperglycemia. She on TPN and TPN can cause the blood sugar to rise.

The diagnosis of diastolic heart failure is based on the presence of heart failure symptoms with

a normal ejection fraction, which is between 55% and 60%. Diastolic failure is characterized by high filling pressures because of stiff ventricles

subarachnoid hemorrhage

a stroke resulting from intracranial bleeding into the cerebrospinal fluid-filled space between the arachnoid and pia mater membranes on the surface of the brain

embolic stroke

a stroke that occurs when an embolus lodges in and occludes a cerebral artery, resulting in infarction and edema of the area supplied by the involved vessel

hemorrhagic stroke

a stroke that results from bleeding into the brain tissue itself (intracerebral or intraparenchymal hemorrhage) or into the subarachnoid space or ventricles (subarachnoid hemorrhage or intraventricular hemorrhage)

transient ischemic attack (TIA)

a transient episode of neurologic dysfunction caused by focal brain, spinal cord, or retinal ischemia, but without acute infarction of the brain. Clinical symptoms typically last less than 1 hour

intracerebral hemorrhage

a type of hemorrhagic stroke in which bleeding within the brain caused by a rupture of a blood vessel occurs; often caused by hypertension and is associated with increased intracranial pressure

1. Which statements accurately describe heart failure (select all that apply)? a. A common cause of diastolic failure is left ventricular hypertrophy. b. A primary risk factor for heart failure is coronary artery disease (CAD). c. Systolic heart failure results in a normal left ventricular ejection fraction. d. Systolic failure is characterized by abnormal resistance to ventricular filling. e. Hypervolemia precipitates heart failure by decreasing cardiac output and increasing oxygen consumption.

a, b. Diastolic failure is characterized by abnormal resistance to ventricular filling. Coronary artery disease (CAD), advanced age, and hypertension are all risk factors for heart failure (HF). Ejection fraction is decreased in systolic HF. Dysrthythmia precipitates HF with decreased cardiac output (CO) and increased workload and oxygen requirements of the myocardium.

Common psychosocial reactions of the stroke patient to the stroke include (select all that apply) a. depression. b. disassociation. c. intellectualization. d. sleep disturbances. e. denial of severity of stroke.

a, d, e Rationale: The patient with a stroke may experience many losses, including sensory, intellectual, communicative, functional, role behavior, emotional, social, and vocational losses. Some patients experience long-term depression, manifesting symptoms such as anxiety, weight loss, fatigue, poor appetite, and sleep disturbances. The time and energy required to perform previously simple tasks can result in anger and frustration. Frustration and depression are common in the first year after a stroke. A stroke is usually a sudden, extremely stressful event for the patient, caregiver, family, and significant others. The family is often affected emotionally, socially, and financially as their roles and responsibilities change. Reactions vary considerably but may involve fear, apprehension, denial of the severity of stroke, depression, anger, and sorrow.

6. Which initial physical assessment finding would the nurse expect to be present in a patient with acute left-sided heart failure? a. Bubbling crackles and tachycardia b. Hepatosplenomegaly and tachypnea c. Peripheral edema and cool, diaphoretic skin d. Frothy blood-tinged sputum and distended jugular veins

a. Clinical manifestations of acute left-sided heart failure are those of interstitial edema, with bubbling crackles and tachycardia, as well as tachypnea. Later frothy, bloodtinged sputum; severe dyspnea; and orthopnea develop with alveolar edema. Severe tachycardia and cool, clammy skin are present as a result of stimulation of the sympathetic nervous system from hypoxemia. Systemic edema reflected by jugular vein distention, peripheral edema, and hepatosplenomegaly are characteristic of right-sided heart failure.

Carvedilol is a β-adrenergic blocker

and prevents catecholimines from affecting the heart by increasing the rate. β-adrenergic blocking medications block or prevent catecholamines affecting the heart by increasing heart rate.

Strep infection SLE drug hypersensitivity all cause myocarditis via this mechanism

immune-mediated reactions

For a patient who is suspected of having a stroke, one of the most important pieces of information that the nurse can obtain is a. time of the patient's last meal. b. time at which stroke symptoms first appeared. c. patient's hypertension history and management. d. family history of stroke and other cardiovascular diseases.

b Rationale: During initial evaluation, the most important point in the patient's history is the time since onset of stroke symptoms. If the stroke is ischemic, recombinant tissue plasminogen activator (tPA) must be administered within 3 to 4.5 hours of the onset of clinical signs of ischemic stroke; tPA reestablishes blood flow through a blocked artery and prevents brain cell death in patients with acute onset of ischemic stroke.

The nurse explains to the patient with a stroke who is scheduled for angiography that this test is used to determine the a. presence of increased ICP. b. site and size of the infarction. c. patency of the cerebral blood vessels. d. presence of blood in the cerebrospinal fluid.

c Rationale: Angiography provides visualization of cerebral blood vessels and can help estimate perfusion and detect filling defects in the cerebral arteries.

A patient with right-sided hemiplegia and aphasia resulting from a stroke most likely has involvement of the a. brainstem. b. vertebral artery. c. left middle cerebral artery. d. right middle cerebral artery.

c Rationale: If the middle cerebral artery is involved in a stroke, the expected clinical manifestations include aphasia, motor and sensory deficit, and hemianopsia on the dominant side and include neglect, motor and sensory deficit, and hemianopsia on the nondominant side.

A patient experiencing TIAs is scheduled for a carotid endarterectomy. The nurse explains that this procedure is done to a. decrease cerebral edema. b. reduce the brain damage that occurs during a stroke in evolution. c. prevent a stroke by removing atherosclerotic plaques blocking cerebral blood flow. d. provide a circulatory bypass around thrombotic plaques obstructing cranial circulation.

c Rationale: In a carotid endarterectomy, the atheromatous lesion is removed from the carotid artery to improve blood flow.

Bladder training in a male patient who has urinary incontinence after a stroke includes a. limiting fluid intake. b. keeping a urinal in place at all times. c. assisting the patient to stand to void. d. catheterizing the patient every 4 hours.

c Rationale: In the acute stage of stroke, the primary urinary problem is poor bladder control and incontinence. Nurses should promote normal bladder function and avoid the use of indwelling catheters. A bladder retraining program consists of (1) adequate fluid intake, with most fluids administered between 7:00 am and 7:00 pm; (2) scheduled toileting every 2 hours with the use of a bedpan, commode, or bathroom; and (3) noting signs of restlessness, which may indicate the need for urination. Intermittent catheterization may be used for urinary retention (not urinary incontinence). During the rehabilitation phase after a stroke, nursing interventions focused on urinary continence include (1) assessment for bladder distention by palpation; (2) offering the bedpan, urinal, commode, or toilet every 2 hours during waking hours and every 3 to 4 hours at night; (3) using a direct command to help the patient focus on the need to urinate; (4) assistance with clothing and mobility; (5) scheduling most fluid intake between 7:00 am and 7:00 pm; and (6) encouraging the usual position for urinating (i.e., standing for men and sitting for women).

The factor related to cerebral blood flow that most often determines the extent of cerebral damage from a stroke is the a. amount of cardiac output. b. oxygen content of the blood. c. degree of collateral circulation. d. level of carbon dioxide in the blood.

c Rationale: The extent of the stroke depends on the rapidity of onset, size of the lesion, and presence of collateral circulation.

18. A 2400-mg sodium diet is prescribed for a patient with chronic heart failure. The nurse recognizes that additional teaching is necessary when the patient makes which statement? a. "I should limit my milk intake to 2 cups a day." b. "I can eat fresh fruits and vegetables without worrying about sodium content." c. "I can eat most foods as long as I do not add salt when cooking or at the table." d. "I need to read the labels on prepared foods and medicines for their sodium content."

c. All foods that are high in sodium should be eliminated in a 2400-mg sodium diet, in addition to the elimination of salt during cooking. Examples include obviously salted snack foods as well as pickles, processed prepared foods, and many sauces and condiments

What describes the action of the natriuretic peptides and nitric oxide in their counterregulatory processes in response to heart failure (HF)? a. Excretion of potassium b. Increased release of ADH c. Vasodilation and decreased blood pressure d. Decreased glomerular filtration rate and edema

c. Both the natriuretic peptides and nitric oxide contribute to vasodilation, decreased blood pressure, and decreased afterload. The natriuretic peptides also increase excretion of sodium by increasing glomerular filtration rate and diuresis (renal effects) as well as interfere with ADH release and inhibit aldosterone and renin secretion (hormonal effects).

difference b/w constrictive pericarditis & cardiac tamponade is ___

impaired ventricular filling in mid-late diastole (spares early diastole) cardiac tamponade=ALL of diastole

• Nonpharmacologic therapies, including

cardiac resynchronization therapy, biventricular pacing, intra-aortic balloon pump, and ventricular assist devices

most are asymptomatic however lesions can involve the conduction system

cardiac sarcoidosis

noncaseating granulomas with giant cells NO eosinophils NO myocyte necrosis

cardiac sarcoidosis

A 23 year old female was walking her dog when she suddenly collapsed. Autopsy and histological findings revealed no necrosis only noncaseating granulomas with large cells...

cardiac sarcoidosis -no eosinophils -no myocyte necrosis -noncaseating granulomas with giant cells

FACES

clinical signs of HF: -Fatigues -Limitation of Activities -Chest congestion/cough -Edema -SOB

aneurysms

congenital or acquired weakness of the arterial wall resulting in dilation and ballooning of the vessel

see kussmaul's sign in ___

constrictive pericarditis

Most common cause of myocarditis...

coxsachievirus B

Information provided by the patient that would help differentiate a hemorrhagic stroke from a thrombotic stroke includes a. sensory disturbance. b. a history of hypertension. c. presence of motor weakness. d. sudden onset of severe headache.

d Rationale: A hemorrhagic stroke usually causes a sudden onset of symptoms, which include neurologic deficits, headache, nausea, vomiting, decreased level of consciousness, and hypertension. Ischemic stroke symptoms may progress in the first 72 hours as infarction and cerebral edema increase.

Of the following patients, the nurse recognizes that the one with the highest risk for a stroke is a(n) a. obese 45-year-old Native American. b. 35-year-old Asian American woman who smokes. c. 32-year-old white woman taking oral contraceptives. d. 65-year-old African American man with hypertension.

d Rationale: Nonmodifiable risk factors for stroke include age (older than 65 years), male gender, ethnicity or race (incidence is highest in African Americans; next highest in Hispanics, Native Americans/Alaska Natives, and Asian Americans; and next highest in white people), and family history of stroke or personal history of a transient ischemic attack or stroke. Modifiable risk factors for stroke include hypertension (most important), heart disease (especially atrial fibrillation), smoking, excessive alcohol consumption (causes hypertension), abdominal obesity, sleep apnea, metabolic syndrome, lack of physical exercise, poor diet (high in saturated fat and low in fruits and vegetables), and drug abuse (especially cocaine). Other risk factors for stroke include a diagnosis of diabetes mellitus, increased serum levels of cholesterol, birth control pills (high levels of progestin and estrogen), history of migraine headaches, inflammatory conditions, hyperhomocystinemia, and sickle cell disease.

heart failure definition

inadequate pumping and/or filling of the heart -perfusion insufficient to meet O2 needs of tissues characterized by 1. ventricular failure, 2. reduced exercise tolerance, 3. diminished quality of life, 4. shortened life expectancy

kussmaul's sign

inc in JVD during inspiration

Paget's disease causes

increased workload of the heart due to an increased vascular bed in the skeletal muscles

During the first year following a heart transplant, the nurse knows that which is the major cause of death?

infection and acute rejection are the major causes of death.

stroke

death of brain cells that occurs when there is ischemia (inadequate blood flow) to a part of the brain or hemorrhage into the brain

EF in systolic HF

decreases, <45% normally 55-60%

damage to the myocytes themselves can cause myocarditis showing signs of

degeneration and necrosis

dysphasia

difficulty related to the comprehension or use of language

Cor pulmonale is the .

dilation of the right ventricle caused by pulmonary diseases. It manifests as right heart failure

what is ewarts sign

dullness to percussion on angle of L scapula

Pulmonary edema manifests as

dyspnea, orthopnea, anxiety, and cold and clammy skin.

____ is most useful for diagnosis of pericardial tamponade

echo

gold standard for diagnosis of pericardial effusion

echo

Significance of abdominal tenderness, hematuria, joint pain and petechia?

embolization

Cardiac dilation is an

enlargement of the heart chambers, usually the ventricles; it occurs when pressure in the heart chambers is elevated over time

What are the histological features of an immune mediated myocarditis ...

eosinophils penetrating myocardium evident of a drug reaction ex. methyldopa

Dopamine is a β-adrenergic agonist whose inotropic action is used .

for treatment of severe heart failure accompanied by hemodynamic instability. Such a state may be indicated by tachycardia accompanied by hypotension

Severe cause of myocarditis that involves cellular infiltration aggressive high mortality rate

giant cell myocarditis

poor prognosis often with a rapid and lethal clinical course

giant cell myocarditis

tragic younger / middle aged eosinophils macrophages plasma cells lymphocytes "heterogenous infiltrate" poor prognosis

giant cell myocarditis

chest xray: pericardial effusion

globular heart

may see large numbers of eosinophils but will not see giant cells here

hypersensitivity myocarditis

methyldopa is associated with this type of myocarditis

hypersensitivity myocarditis

older age group interstitial cell infiltrates perivascular lymphocytes, macrophages, and large number of eosinophils

hypersensitivity myocarditis

eosinophils think...

hypersensitivity myocarditis giant cell myocarditis (differentiated histologically)

After the first year following a heart transplant, the nurse knows that which is the major cause of death?

malignancy (especially lymphoma) and cardiac vasculopathy (accelerated coronary artery disease [CAD]) are the major causes of death

A 43 year old factory worker who works next to a chemical plant has been complaining of chest pain and what feels like his heart is getting bigger... and he has been fainting a lot

myocarditis due to radiation

loss of muscle giant cells mononuclear inflammatory infiltrate

notice the big ass giant cells, yeah

chagas disease histology

notice the trypanosomes (damn)

congestive heart failure

older term, not all heart failure causes pulmonary congestion new terms: acute vs. chronic heart failure

acute pericarditis: if refractory to NSAIDS give...

oral corticosteroids/colchicine

layers of pericardium

parietal (fibrous) visceral (serosal)

Digitalis glycosides can be used in

patients who remain symptomatic despite being on ACE and β-blocker therapy

chest xray: constrictive pericarditis

pericardial calcification mildly enlarged

cardiac tamponade is caused by ___

pericardial effusion

causes of pericardial tamponade

pericardial effusion

se ewarts sign in____

pericardial effusion

physical findings of acute pericarditis

pericardial friction rub -scratchy -comes/goes -heard during full held exhalation

constrictive pericarditis is ___

pericardial scarring causing restrcited diastolic filling of all 4 chambers *thick fibrotic/calcific shell around heart

see beck's triad in___

pericardial tamponade

definitive treatment: constrictive pericarditis

pericardiectomy

definitive treatment for pericardial tamponade

pericardiocentesis

Cardiac output depends on various factors such as

preload, heart rate, and myocardial contractility. If preload increases, there is volume overload on the right ventricle, which can cause heart failure. An increase in heart rate increases cardiac output. Impaired myocardial contractility decreases cardiac output. Respiratory rate and portal pressure do not alter cardiac output.

• Drug therapy is a critical part of the care of patients with HF: Diuretics are used to .

reduce edema, pulmonary venous pressure, and preload

What are the different causes of myocarditis?

sarcoid SLE drug induced radiation induced giant cell myocarditis

gentleman in 30's crashed his car into a tree, minimal damage to car and he had no injuries on autopsy he found white deposits scattered throughout myocardium looked like ischemic changes...

sarcoidosis -non-caseating granuloma that can cause sudden death in someone without being symptomatic

In anemia, decreased oxygen-carrying capacity of the blood

stimulates an increase in cardiac output to meet the demands of the body.

Which bacteria can cause infective carditis?

streptococcal or staphylococcal bacteria

ischemic stroke

stroke that results from inadequate blood flow to the brain caused by partial or complete occlusion of an artery

cerebrovascular accident (CVA)

term used to describe a stroke

brain attack

term used to describe a stroke; communicates the urgency of recognizing the clinical manifestations of a stroke and treating a medical emergency

Ejection fraction, normal

the amount of blood ejected from the left ventricle with each contraction, normally 55-60%

o Angiotensin-converting enzyme (ACE) inhibitors are

the primary drug of choice in chronic HF patients with systolic failure

eecho: constrictive pericarditis

thickened/calcicif pericardium

allergy asthma drug reactions

think eosinophils

Thyrotoxicosis increases the

tissue metabolic rate, leading to an increase in heart rate and workload of the heart.

What test is used to diagnose endocarditis?

transesophageal echocardiogram

True or False borrelia and neisseria can affect the myocardium

true

myocarditis

varying degrees of CHF wide spectrum of changes most cases are self-limiting but sometimes lead to lethal arrythmias symptoms are vague - flu like symptoms but mostly asymptomatic sometimes will have dilated cardiomyopathy (remember one cause of DMP is viral myocarditis)

How do you tell the difference between a viral induced myocarditis and a bacterial induced myocarditis?

viral = mononuclear cell infiltrate bacterial = neutrophil infiltrate and some necrosis

dressler's syndrome

weeks/months after MI -cause of acute pericarditis

o -Adrenergic blockers in combination

with ACE inhibitors and diuretics have improved survival of patients with HF


Related study sets

Cell Biology Exam Practice Questions

View Set

( 3 ) Task Environment, General Environment, & Ethics

View Set

Lesson 3: Article 1 of the U.S Constitution

View Set

Taylor's Chapter 14: Implementing (Prep U)

View Set

Anatomy Ch. 25- The Urinary System

View Set

Energy Flow in Ecosystems & Organisms

View Set

Fundamentals of Nursing Ch. 4- NCLEX PRACTICE

View Set